Stack Exchange Network

Stack Exchange network consists of 183 Q&A communities including Stack Overflow , the largest, most trusted online community for developers to learn, share their knowledge, and build their careers.

What is the shortest Ph.D. thesis? [closed]

The question is self-explanatory, but I want to make some remarks in order to prevent the responses from going off into undesirable directions.

It seems that every few years I hear someone ask this question; it seems to hold a perennial fascination for research mathematicians, just as quests for short proofs do. The trouble is that it has strong urban-legend tendencies: someone will say, "So-and-so's thesis was only $\epsilon$ pages long!" where $\epsilon \ll 1$ . It will often be very difficult to confirm or disconfirm such claims, since Ph.D. theses are often not even published, let alone readily available online. If you Google around for a while, as I did, you will find many dubious leads and can easily waste a lot of time on wild goose chases. Frankly, I'm a bit fed up with this state of affairs. I am therefore asking this question on MO in the hope that doing so will put this old question to rest, or at least establish provable upper bounds.

I would therefore request that you set yourself a high standard before replying. Don't post a candidate unless you're sure your facts are correct, and please give some indication why you're so sure. Read the meta discussion before posting. (Note that the meta discussion illustrates that even a MathSciNet citation isn't always totally definitive.) Include information about the content and circumstances of the thesis if you know it, but resist the temptation to gossip or speculate.

I'm not making this question community wiki or big-list because it should ideally have a definite answer, though I grant that it's possible that there are some borderline cases out there (perhaps there are theses that were not written in scholarly good faith, or documents that some people would regard as equivalent to a Ph.D. thesis but that others would not, or theses in subjects that are strictly speaking distinct from mathematics but that are arguably indistinguishable from mathematics dissertations).

Finally, to anticipate a possible follow-up question, there is a list of short published papers here (search for "Nelson"). Note that the question of the shortest published paper is not as urban-legendy because the facts are easier to verify. I looked up the short papers listed there myself and found them to be quite interesting. So in addition to trying to settle an urban legend, I am hoping that this question will bring to light some interesting and lesser known mathematics.

  • ho.history-overview
  • 9 $\begingroup$ I think it really should be CW. It makes no sense to me that the shorter the proposed candidate, the more reputation the proposer will get. It will also lower the temptation for people to post gossipy stuff. $\endgroup$ –  Alex B. Commented Feb 8, 2011 at 15:31
  • 3 $\begingroup$ The only reasonable interpretation of the question is extremely short theses in general, because there is more than one measure of the length of a thesis. Moreover in some cases it's debatable whether a particular document really is a thesis or the full thesis. It realy should be CW. $\endgroup$ –  Greg Kuperberg Commented Feb 8, 2011 at 15:40
  • 3 $\begingroup$ How would you like to count? Do all the cover pages, table of contents, abstract, etc. count? How about references? Or do you begin with the introduction and only include the content? $\endgroup$ –  Noah Stein Commented Feb 8, 2011 at 16:33
  • 5 $\begingroup$ -1. This question is terrible. I'm sure I could reformat my thesis in a silly font size to make it have a ludicrously small number of pages. $\endgroup$ –  Peter McNamara Commented Feb 8, 2011 at 19:50
  • 8 $\begingroup$ @Peter McNamara: you probably could, but I'm pretty certain that this is not the issue being discussed here. Anyway, most universities have specific formatting standards and would not let you submit it in this form. $\endgroup$ –  Thierry Zell Commented Feb 8, 2011 at 20:05

9 Answers 9

David Rector's thesis ("An Unstable Adams Spectral Sequence", MIT 1966) is 9 pages, according to the record at the MIT library . I haven't seen the actual thesis for many years, but I'm pretty the actual mathematical content takes about 3 pages total, and is largely identical to the published version in Topology (1966, same title, doi link: https://doi.org/10.1016/0040-9383(66)90025-5 ), which is 3 pages plus bibliography. (Dan Kan, his advisor, likes short papers.)

  • 2 $\begingroup$ Probably not a coincidence. $\endgroup$ –  Tyler Lawson Commented Feb 8, 2011 at 20:25
  • 3 $\begingroup$ Accepted provisionally. Enough people seem instinctively annoyed at this question that it seems likely to be closed soon (despite the fact that I'm asking it on MO in order to prevent its proliferation elsewhere). It doesn't seem likely that a stronger candidate will emerge before then. Ideally I'd like to examine the thesis myself before accepting the answer but I don't feel like purchasing it and it may be a while before my next trip to Boston. $\endgroup$ –  Timothy Chow Commented Feb 9, 2011 at 15:56
  • 4 $\begingroup$ Aside from the library copy, there should be a slightly more accessible copy in the MIT Math reading room. (They used to keep copies of theses there, and I assume they still do.) Maybe somebody reading this could wander down the hall and take a look. :) $\endgroup$ –  Charles Rezk Commented Feb 9, 2011 at 18:55
  • 31 $\begingroup$ I'm in the reading room now. Rector's thesis comprises a title page, an abstract page, a table of contents page, 7 pages of math, a bibliography page (8 refs.), and a biographical note page. The MIT library record's "9 leaves" exclude the title/abstract/contents, which are not numbered. Except for some trivial changes in wording in the intro, the mathematical part is indeed identical to the 4-page Topology paper, vol. 5 (1966), 343-346. The thesis occupies more space since it's manually typed; not including section titles, the 4 sections are respectively 18, 23, 42, and 36 typewritten lines. $\endgroup$ –  Timothy Chow Commented Aug 19, 2011 at 18:44
  • $\begingroup$ 119 typewritten lines! $\endgroup$ –  David Roberts ♦ Commented Oct 14, 2022 at 10:53

John Nash's thesis was 26 pages, and had two references in the bibliography.

Edmund Landau's thesis was 13 pages long.

  • $\begingroup$ There is an English translation here: arxiv.org/PS_cache/arxiv/pdf/0803/0803.3787v2.pdf That document is 17 pages (including title page, etc.). $\endgroup$ –  Zach N Commented Feb 8, 2011 at 18:06
  • 2 $\begingroup$ For a link to a scanned version of Landau's thesis see here gdz.sub.uni-goettingen.de/dms/load/img/?PPN=PPN317979566 The document has 18 pages, of which 2 are completely empty, indeed the catalogue of the libraries of Berlin gives 16 pages as lengths. (the French national library catalogue gives 18). Moreover, one page is a title page, one is a dedication, and one is a vita. So, depending on what one actually counts, 18, 16, or 13. According to library catalogues 16 or 18. $\endgroup$ –  user9072 Commented Feb 8, 2011 at 18:15

I believe the shortest PhD thesis is of Burt Totaro "Milnor K-theory is the simplest part of algebraic K-theory", 12 pages.

Milnor K-theory is the simplest part of algebraic K-theory, Ph.D. thesis, University of California, Berkeley, 1989; published as: K-Theory 6 (1992), 177-189 ( Portico archived version ).

Burt Totaro's webpage at Cambridge , including a pdf of the published version .

  • 1 $\begingroup$ its complete thesis. I gave two references here, Milnor K-theory is the simplest part of algebraic K-theory, Ph.D. thesis, University of California, Berkeley, 1989 and K-Theory 6 (1992), 177-189 $\endgroup$ –  J Verma Commented Feb 8, 2011 at 17:43
  • 2 $\begingroup$ I noticed, but the reference to the actual thesis does not have a page numbers (and it is somewhat surprising that the number of pages did not change from the thesis to K-theory's format) :) $\endgroup$ –  Mariano Suárez-Álvarez Commented Feb 8, 2011 at 17:45
  • 16 $\begingroup$ Totaro's 1989 thesis is titled "K-theory and algebraic cycles" and, according to ProQuest, is 20 pages. If your university library subscribes to ProQuest, you can see a PDF preview of the thesis by searching for "Totaro, Burt" in the Dissertations and Theses database. $\endgroup$ –  Zach N Commented Feb 8, 2011 at 18:02
  • 4 $\begingroup$ You can download it on mathscinet. It has 16 numbered pages, incl. 1 page of bibliography. Definitions start on page 1 though, not much of an introduction. $\endgroup$ –  fherzig Commented Feb 9, 2011 at 2:58
  • 1 $\begingroup$ I downloaded the thesis from ProQuest. It comprises a signature page, a title page, an abstract page, an epigram page, 15 pages of (TeXed) math, and a bibliography page. Short, but not as short as David Rector's thesis. $\endgroup$ –  Timothy Chow Commented Aug 19, 2011 at 19:00

This is not really an answer because these PhD's were never actually written, but anyway: in his book A mathematicians miscellany (in the chapter on math with minimum raw material) Littlewood gave 2 examples that could have been 2-line PhDs:

(1) Cayley's projective definition of length

(2)Theorem: An integral function never 0 or 1 is a constant. Proof: $\exp(i\Omega(f(z)))$ is a bounded integral function. ($\Omega$ is inverse to the elliptic modular function.)

  • 3 $\begingroup$ Richard, perhaps you overlooked that Gerry Myerson already gave this example on the meta discussion? $\endgroup$ –  Timothy Chow Commented Feb 8, 2011 at 15:53
  • 19 $\begingroup$ I don't think it is reasonable to expect people to have read all the meta discussion before posting on a regular thread. This is a sort of fluff question, so it doesn't matter much, but in general I think it should be fine to repost answers from meta, so that the main thread has the most complete record of answers to the question. $\endgroup$ –  David E Speyer Commented Feb 8, 2011 at 16:52
  • 7 $\begingroup$ While I agree with David Speyer in general, I also do not think this should have been posted as an answer to this particular question, given the questioner's emphasis on restricting the scope of the question. $\endgroup$ –  Charles Staats Commented Feb 8, 2011 at 17:27
  • 6 $\begingroup$ @David: I too would agree that in general it’s not reasonable to expect people to read meta discussions on questions before answering them. But this question specifically asks us to, and gives good reasons for it. $\endgroup$ –  Peter LeFanu Lumsdaine Commented Feb 8, 2011 at 20:18
  • $\begingroup$ (2) is a trivial corollary of Picard's little theorem. $\endgroup$ –  tst Commented Jun 13, 2017 at 3:06

I already posted this on meta where there was some discussion of whether the page count was correct. My guess is that it is, so I will post it here too:

MR2615548 Martens, Henrik Herman Buvik A NEW PROOF OF TORELLI'S THEOREM. Thesis (Ph.D.)–New York University. 1962. 12 pp.

  • 8 $\begingroup$ Compared to that, the thesis of his student Kristian Seip was a massive tome, weighing in at 30 pages. $\endgroup$ –  Harald Hanche-Olsen Commented Feb 9, 2011 at 7:56

Kurt Gödel seems to be a good candidate for this "prize".

Let me quote from this review (see Page 74) of Kurt Gödel Collected Works.

The first three works of Godel in this volume are his dissertation of 1929 ( twenty-one pages in English ), a revised and substantially abbreviated version (eleven pages in English) published in 1930, and a brief abstract based on a presentation of Godel's results in Konigsberg on 6 September 1930. Of all of Godel's longer, published writings, his dissertation has been, until now, the most difficult to obtain, and is here translated for the first time into English, by Stefan Bauer-Mengelberg and van Heijenoort.
  • 3 $\begingroup$ The original version of his thesis seems to have 33 pages; see permalink.obvsg.at/AC05181322 (the number next to "Umfangsangabe") $\endgroup$ –  user9072 Commented Feb 8, 2011 at 16:59
  • 2 $\begingroup$ I cannot say anything about the original version (my German skills are null, not almost null). But I have just checked my copy of the Collected Works (unfortunately I have not found any online library to link), and in pages 60-101 we can find Godel's dissertation (even pages match German, while odd ones match English). Thus, the description "21 pages in English" is accurate. $\endgroup$ –  boumol Commented Feb 8, 2011 at 17:14
  • 1 $\begingroup$ I did not want to imply your claim was not accurate. Only, as I understand the question, it is about the actual document the person submitted as a thesis. Thus, I supplemented this information, documenting it by the link to the entry of Goedels thesis in the joint library catalogue of Austrian (academic) libraries. It specifies title, author, year, lengths (that's the Umfangsangabe, S. abbreviates 'Seiten' i.e. pages), the type of document (thesis of University of Vienna (Wien)), and finally the specific libraries where it can be found. $\endgroup$ –  user9072 Commented Feb 8, 2011 at 17:48

According to mathscinet, Eva Kallin's thesis was 14 pages.

  • 3 $\begingroup$ This is promising, but as the question mentions and the meta thread shows, MathSciNet alone is not an authoritative reference. More documentation? $\endgroup$ –  Peter LeFanu Lumsdaine Commented Feb 8, 2011 at 20:12

Barry Mazur's thesis on the proof of the Schoenflies conjecture (and introducing the method of infinite repetition in topology) is 5 pages long.

  • 4 $\begingroup$ According to "Mathematical apocrypha redux" by Krantz, Mazur's thesis was 26 pages long. $\endgroup$ –  Michael Greinecker Commented Feb 8, 2011 at 16:22
  • 2 $\begingroup$ Mathscinet says his thesis is 30 pages. $\endgroup$ –  Jaikrishnan Commented Feb 8, 2011 at 16:26
  • 70 $\begingroup$ Well, it may not be the shortest but it surelyt appears to have the most variable number of pages! $\endgroup$ –  Mariano Suárez-Álvarez Commented Feb 8, 2011 at 16:42
  • 17 $\begingroup$ Let's please heed Timothy's call to do one's homework carefully. "Don't post a candidate unless you're sure your facts are correct, and please give some indication why you're so sure. Read the meta discussion before posting." $\endgroup$ –  Todd Trimble Commented Feb 8, 2011 at 16:47
  • $\begingroup$ Yikes. I had never looked at the thesis, but just the published version in the Bulletin of the AMS which is 5 pages long. $\endgroup$ –  Victor Miller Commented Feb 23, 2011 at 22:12

Not the answer you're looking for? Browse other questions tagged ho.history-overview or ask your own question .

  • Featured on Meta
  • Bringing clarity to status tag usage on meta sites
  • Join Stack Overflow’s CEO and me for the first Stack IRL Community Event in...

shortest math dissertation

The best free cultural &

educational media on the web

  • Online Courses
  • Certificates
  • Degrees & Mini-Degrees
  • Audio Books

John Nash’s Super Short PhD Thesis: 26 Pages & 2 Citations

in Math | July 9th, 2018 2 Comments

nash thesis

When John Nash wrote  “Non Coop­er­a­tive Games,”   his Ph.D. dis­ser­ta­tion at Prince­ton in 1950, the text of his the­sis ( read it online ) was brief. It ran only 26 pages. And more par­tic­u­lar­ly, it was light on cita­tions. Nash’s diss cit­ed two texts:  John von Neu­mann & Oskar Mor­gen­stern’s  The­o­ry of Games and Eco­nom­ic Behav­ior   (1944), which  essen­tial­ly cre­at­ed game the­o­ry and rev­o­lu­tion­ized the field of eco­nom­ics; the oth­er cit­ed text, “Equi­lib­ri­um Points in n‑Person Games,”  was an arti­cle writ­ten by Nash him­self. And it laid the foun­da­tion for his dis­ser­ta­tion, anoth­er sem­i­nal work in the devel­op­ment of game the­o­ry, for which Nash won the Nobel Prize in Eco­nom­ic Sci­ences in 1994 .

The reward of invent­ing a new field is hav­ing a slim bib­li­og­ra­phy.

If you would like to sign up for Open Culture’s free email newslet­ter,  please find it here . Or fol­low our posts on Threads , Face­book , BlueSky or Mastodon .

If you would like to sup­port the mis­sion of Open Cul­ture, con­sid­er mak­ing a dona­tion to our site . It’s hard to rely 100% on ads, and your con­tri­bu­tions will help us con­tin­ue pro­vid­ing the best free cul­tur­al and edu­ca­tion­al mate­ri­als to learn­ers every­where. You can con­tribute through Pay­Pal , Patre­on , and Ven­mo (@openculture). Thanks!

Note: An ear­li­er ver­sion of this post appeared on our site in June, 2015.

Relat­ed Con­tent:

The Short­est-Known Paper Pub­lished in a Seri­ous Math Jour­nal: Two Suc­cinct Sen­tences

The World Record for the Short­est Math Arti­cle: 2 Words

Free Online Math Cours­es

Free Math Text­books

by OC | Permalink | Comments (2) |

shortest math dissertation

Related posts:

Comments (2), 2 comments so far.

Some­times doc­tor­al dis­ser­ta­tions are long on foot­notes and bib­li­og­ra­phy — and short on orig­i­nal think­ing. John Nash reversed the aca­d­e­m­ic trend. Reminds me of the Renais­sance painter who was asked for evi­dence of his abil­i­ty to draw. He drew a near-per­fect cir­cle on a can­vas, and was accept­ed by the mas­ter as an appren­tice.

Excel­lent con­cept and articles.…worth reading.…pl for­ward more read­ing

Thanks and regards

Dr B Vijay Sarthi

Add a comment

Leave a reply.

Name (required)

Email (required)

XHTML: You can use these tags: <a href="" title=""> <abbr title=""> <acronym title=""> <b> <blockquote cite=""> <cite> <code> <del datetime=""> <em> <i> <q cite=""> <s> <strike> <strong>

Click here to cancel reply.

  • 1,700 Free Online Courses
  • 200 Online Certificate Programs
  • 100+ Online Degree & Mini-Degree Programs
  • 1,150 Free Movies
  • 1,000 Free Audio Books
  • 150+ Best Podcasts
  • 800 Free eBooks
  • 200 Free Textbooks
  • 300 Free Language Lessons
  • 150 Free Business Courses
  • Free K-12 Education
  • Get Our Daily Email

shortest math dissertation

Free Courses

  • Art & Art History
  • Classics/Ancient World
  • Computer Science
  • Data Science
  • Engineering
  • Environment
  • Political Science
  • Writing & Journalism
  • All 1700 Free Courses

Receive our Daily Email

Free updates, get our daily email.

Get the best cultural and educational resources on the web curated for you in a daily email. We never spam. Unsubscribe at any time.

FOLLOW ON SOCIAL MEDIA

Free Movies

  • 1150 Free Movies Online
  • Free Film Noir
  • Silent Films
  • Documentaries
  • Martial Arts/Kung Fu
  • Free Hitchcock Films
  • Free Charlie Chaplin
  • Free John Wayne Movies
  • Free Tarkovsky Films
  • Free Dziga Vertov
  • Free Oscar Winners
  • Free Language Lessons
  • All Languages

Free eBooks

  • 700 Free eBooks
  • Free Philosophy eBooks
  • The Harvard Classics
  • Philip K. Dick Stories
  • Neil Gaiman Stories
  • David Foster Wallace Stories & Essays
  • Hemingway Stories
  • Great Gatsby & Other Fitzgerald Novels
  • HP Lovecraft
  • Edgar Allan Poe
  • Free Alice Munro Stories
  • Jennifer Egan Stories
  • George Saunders Stories
  • Hunter S. Thompson Essays
  • Joan Didion Essays
  • Gabriel Garcia Marquez Stories
  • David Sedaris Stories
  • Stephen King
  • Golden Age Comics
  • Free Books by UC Press
  • Life Changing Books

Free Audio Books

  • 700 Free Audio Books
  • Free Audio Books: Fiction
  • Free Audio Books: Poetry
  • Free Audio Books: Non-Fiction

Free Textbooks

  • Free Physics Textbooks
  • Free Computer Science Textbooks
  • Free Math Textbooks

K-12 Resources

  • Free Video Lessons
  • Web Resources by Subject
  • Quality YouTube Channels
  • Teacher Resources
  • All Free Kids Resources

Free Art & Images

  • All Art Images & Books
  • The Rijksmuseum
  • Smithsonian
  • The Guggenheim
  • The National Gallery
  • The Whitney
  • LA County Museum
  • Stanford University
  • British Library
  • Google Art Project
  • French Revolution
  • Getty Images
  • Guggenheim Art Books
  • Met Art Books
  • Getty Art Books
  • New York Public Library Maps
  • Museum of New Zealand
  • Smarthistory
  • Coloring Books
  • All Bach Organ Works
  • All of Bach
  • 80,000 Classical Music Scores
  • Free Classical Music
  • Live Classical Music
  • 9,000 Grateful Dead Concerts
  • Alan Lomax Blues & Folk Archive

Writing Tips

  • William Zinsser
  • Kurt Vonnegut
  • Toni Morrison
  • Margaret Atwood
  • David Ogilvy
  • Billy Wilder
  • All posts by date

Personal Finance

  • Open Personal Finance
  • Amazon Kindle
  • Architecture
  • Artificial Intelligence
  • Comics/Cartoons
  • Current Affairs
  • English Language
  • Entrepreneurship
  • Food & Drink
  • Graduation Speech
  • How to Learn for Free
  • Internet Archive
  • Language Lessons
  • Most Popular
  • Neuroscience
  • Photography
  • Pretty Much Pop
  • Productivity
  • UC Berkeley
  • Uncategorized
  • Video - Arts & Culture
  • Video - Politics/Society
  • Video - Science
  • Video Games

Great Lectures

  • Michel Foucault
  • Sun Ra at UC Berkeley
  • Richard Feynman
  • Joseph Campbell
  • Jorge Luis Borges
  • Leonard Bernstein
  • Richard Dawkins
  • Buckminster Fuller
  • Walter Kaufmann on Existentialism
  • Jacques Lacan
  • Roland Barthes
  • Nobel Lectures by Writers
  • Bertrand Russell
  • Oxford Philosophy Lectures

Sign up for Newsletter

shortest math dissertation

Open Culture scours the web for the best educational media. We find the free courses and audio books you need, the language lessons & educational videos you want, and plenty of enlightenment in between.

Great Recordings

  • T.S. Eliot Reads Waste Land
  • Sylvia Plath - Ariel
  • Joyce Reads Ulysses
  • Joyce - Finnegans Wake
  • Patti Smith Reads Virginia Woolf
  • Albert Einstein
  • Charles Bukowski
  • Bill Murray
  • Fitzgerald Reads Shakespeare
  • William Faulkner
  • Flannery O'Connor
  • Tolkien - The Hobbit
  • Allen Ginsberg - Howl
  • Dylan Thomas
  • Anne Sexton
  • John Cheever
  • David Foster Wallace

Book Lists By

  • Neil deGrasse Tyson
  • Ernest Hemingway
  • F. Scott Fitzgerald
  • Allen Ginsberg
  • Patti Smith
  • Henry Miller
  • Christopher Hitchens
  • Joseph Brodsky
  • Donald Barthelme
  • David Bowie
  • Samuel Beckett
  • Art Garfunkel
  • Marilyn Monroe
  • Picks by Female Creatives
  • Zadie Smith & Gary Shteyngart
  • Lynda Barry

Favorite Movies

  • Kurosawa's 100
  • David Lynch
  • Werner Herzog
  • Woody Allen
  • Wes Anderson
  • Luis Buñuel
  • Roger Ebert
  • Susan Sontag
  • Scorsese Foreign Films
  • Philosophy Films
  • September 2024
  • August 2024
  • February 2024
  • January 2024
  • December 2023
  • November 2023
  • October 2023
  • September 2023
  • August 2023
  • February 2023
  • January 2023
  • December 2022
  • November 2022
  • October 2022
  • September 2022
  • August 2022
  • February 2022
  • January 2022
  • December 2021
  • November 2021
  • October 2021
  • September 2021
  • August 2021
  • February 2021
  • January 2021
  • December 2020
  • November 2020
  • October 2020
  • September 2020
  • August 2020
  • February 2020
  • January 2020
  • December 2019
  • November 2019
  • October 2019
  • September 2019
  • August 2019
  • February 2019
  • January 2019
  • December 2018
  • November 2018
  • October 2018
  • September 2018
  • August 2018
  • February 2018
  • January 2018
  • December 2017
  • November 2017
  • October 2017
  • September 2017
  • August 2017
  • February 2017
  • January 2017
  • December 2016
  • November 2016
  • October 2016
  • September 2016
  • August 2016
  • February 2016
  • January 2016
  • December 2015
  • November 2015
  • October 2015
  • September 2015
  • August 2015
  • February 2015
  • January 2015
  • December 2014
  • November 2014
  • October 2014
  • September 2014
  • August 2014
  • February 2014
  • January 2014
  • December 2013
  • November 2013
  • October 2013
  • September 2013
  • August 2013
  • February 2013
  • January 2013
  • December 2012
  • November 2012
  • October 2012
  • September 2012
  • August 2012
  • February 2012
  • January 2012
  • December 2011
  • November 2011
  • October 2011
  • September 2011
  • August 2011
  • February 2011
  • January 2011
  • December 2010
  • November 2010
  • October 2010
  • September 2010
  • August 2010
  • February 2010
  • January 2010
  • December 2009
  • November 2009
  • October 2009
  • September 2009
  • August 2009
  • February 2009
  • January 2009
  • December 2008
  • November 2008
  • October 2008
  • September 2008
  • August 2008
  • February 2008
  • January 2008
  • December 2007
  • November 2007
  • October 2007
  • September 2007
  • August 2007
  • February 2007
  • January 2007
  • December 2006
  • November 2006
  • October 2006
  • September 2006

©2006-2024 Open Culture, LLC. All rights reserved.

  • Advertise with Us
  • Copyright Policy
  • Privacy Policy
  • Terms of Use

openculture logo

  • Skip to main content
  • Skip to primary sidebar
  • Skip to footer

Doctorandum

PhD: Been There, Done That...

  • The World’s Shortest PhD Dissertations

“David Lee Rector’s Ph.D. Thesis is just nine pages long plus bio and bibliography, not to mention double-spaced.” [Source & Photo Credit: Ali Almossawi ]

You have probably seen thick dissertations, too heavy to lift with one hand… but have you ever thought of how short a PhD dissertation can possibly be?

Well, John Edensor Littlewood once famously inquired “ whether a dissertation of 2 lines could deserve and get a Fellowship ” – and he seems to have meant it.

Interestingly, some of the world’s shortest PhD theses / dissertations also count among the most famous ones at the same time. Here are the Top 5 we could identify:

24 pages – John F. Nash: Non-Cooperative Games (1950)

17 pages – Albert Einstein: Eine neue Bestimmung der Moleküldimensionen (1905) / A New Determination of Molecular Dimensions (1906)

16 pages – Edmund Landau: Neuer Beweis der Gleichung (1899) / New Proof of the Equation (2007)

13 pages – Burt Totaro: Milnor K-Theory is the Simplest Part of Algebraic K-Theory (1992)

9 pages – David Lee Rector: An Unstable Adams Spectral Sequence (1966)

Please drop us a line if you know any shorter dissertations than the ones above!

Recent Posts

  • 250 Words a Day
  • Procrastination is NOT cool
  • What Great Writers Have in Common
  • Sit Down and Write
  • Get Off Your A** and Write Your Thesis
  • No Grad Student is an Island
  • What Are Your Own Metrics?
  • The Real Cost of Graduation
  • Read Fiction
  • Why Proofreading Matters
  • Leaders Are Readers
  • Effective Supervision
  • Stay Healthy
  • What NOT to do in grad school
  • How to Stay Motivated in Grad School?
  • Attention to Detail
  • Your PhD Supervisor and You
  • Are You Ready for a PhD?
  • How to write a dissertation proposal?
  • The Best Research Proposal Template to Start With
  • How to Write a PhD Research Proposal?
  • What is a Research Proposal?
  • How to Find the Right Thesis Advisor

So Many Papers, so Little Time

A blog about scientific publishing and academic productivity

  • Paperpile News
  • Productivity
  • Inspiration

The Shortest Papers Ever Published

A paper consisting of only figures

This blog post is about short papers. It seems out of place writing a long introduction.

If you ever wondered about the shortest papers ever published, or you just want to take the unique opportunity to read several papers in full within one minute, this post is for you.

Math can be short

Math can be hard and tedious resulting in very long papers. The 1995 proof of Fermat’s last Theorem was 108 pages long.

But math can also be short.

Lander and Parkin’s paper about a conjecture by Euler (related to Fermat’s last Theorem), is probably the dream of everyone ever written a paper: It answers an interesting and important question, it’s correct beyond any doubt,  it’s easy to understand and only two sentences long.

A short mathematical proof by a counterexample

Is there a way to beat that? John Conway and Alexander Soifer submitted a paper in 2005 with the goal to  write the shortest math paper ever.

It is only two words long and contains two distinct proofs of their problem in two figures.

The editors were a bit surprised: “The Monthly publishes exposition of mathematics at many levels, and it contains articles both long and short. Your article, however, is a bit too short to be a good Monthly article. . . A line or two of explanation would really help.”

The authors did not give up and could convince the editors: “I respectfully disagree […] What else is there to explain?” Read the full story .

A short figure only proof

Empty pages

Can you  write a meaningful paper shorter than 2 words? Probably not. Still, I want to mention the following case report of severe writer’s block, which contribute important zero words to the literature:

The shortest paper ever?

Also the following zero-word paper (excluding the abstract) makes an important point. Originally submitted to Nature Chemistry it did not make the cut for publication. But the editors liked it so much that they covered it in their blog  and their production team produced a print PDF for it. It finally appeared in 2016  in the journal “ Chemie in unserer Zeit ” (50(2), 144–145)  published by Wiley. It’s a German journal but don’t worry. The empty pages should be easy to read even in German…

Short paper without any content

Short abstracts

Abstracts should be short by definition. But some are shorter than others. These are the shortest we could find:

Paper with two-word abstract

Enough already

Probably the shortest letter to the editor ever

Brought to you by the folks at Paperpile.

We love papers so we blog about it.

Stack Exchange Network

Stack Exchange network consists of 183 Q&A communities including Stack Overflow , the largest, most trusted online community for developers to learn, share their knowledge, and build their careers.

Q&A for work

Connect and share knowledge within a single location that is structured and easy to search.

Short but Substantial Math Papers [closed]

I am looking for short papers that made a significant impact on the mathematics community. I have already seen: interesting-but-short-math-papers and, What is the Shortest Ph.D. Thesis? on math overflow, but these weren't quite what I was looking for (although the intersection of the set of answers to this question with the set of answers to either of the above links is likely to be non-trivial)

I am more interested in short and important works of mathematics, not necessarily Ph.D.s (but not necessarily not Ph.D.s either). Things that changed the course of mathematical history, that sort of thing.

Any suggested readings would be very much appreciated.

I realize I was not clear on what I meant by short. around the 20 page or less mark. If it goes higher, but is really something for its size then that is acceptable too.

  • reference-request
  • soft-question
  • math-history

Community's user avatar

  • 3 $\begingroup$ A classic in short papers is John Nash's Equilibrium Points in N-Person Games with $1$ page(something which today would maybe not be considered a paper anymore). With $26$ pages a little over, but compensating this through its impact, could be Goedel's Ueber formal Unentscheidbare Saetze der Principia Mathematica und verwandter System, I. $\endgroup$ –  blub Commented Aug 31, 2018 at 13:14
  • 3 $\begingroup$ This MO question could also be of interest : mathoverflow.net/questions/7330/… (the first two answers there are the paper by Riemann in Barry Cipra's answer and the Nash one in the comment above). $\endgroup$ –  Arnaud D. Commented Aug 31, 2018 at 13:19
  • $\begingroup$ Levin's paper on universal search problems was only 2 pages long, but introduced the idea of NP-completeness in complexity theory (which might stretch your definition of mathematics a bit). $\endgroup$ –  chepner Commented Aug 31, 2018 at 15:19
  • $\begingroup$ @chepner: Why is it stretching? P=NP is still an open problem that can be written as an arithmetical sentence (namely one that only quantifies over natural numbers), and even got chosen by the Clay Mathematics Institute as 1 of only 7 Millenium Prize Problems. =) $\endgroup$ –  user21820 Commented Aug 31, 2018 at 16:48
  • 1 $\begingroup$ If Riemann hypothesis is wrong and a counterexample is found, the corresponding paper would be pivotal and extremely short : "BTW, here's a non-trivial zero for Riemann zeta function which is outside the critical line : $a+bi$. kthxbye". $\endgroup$ –  Eric Duminil Commented Aug 31, 2018 at 21:40

7 Answers 7

Riemann's short paper, "Über die Anzahl der Primzahlen unter einer gegebenen Grösse," surely qualifies.

  • 1 $\begingroup$ I've made this answer community wiki so that upvotes can indicate agreement rather than reward. $\endgroup$ –  Barry Cipra Commented Aug 31, 2018 at 13:36

Not a paper, but definitely significant, is Russell's paradoxical letter to Frege.

  • $\begingroup$ I've made this answer community wiki so that upvotes can indicate agreement rather than reward. $\endgroup$ –  Barry Cipra Commented Aug 31, 2018 at 13:36

I would recommend Classics of Mathematics , ed. Ronald Calinger. It's got articles from a very broad range of mathematical history, all the way from the Stone Age through 1932 (includes Gödel). Naturally, it does not include later works. Most of the most important ideas in modern mathematics will be found in here somewhere. For more modern topics, I found Love and Math by Edward Frenkel to be excellent.

Adrian Keister's user avatar

Short does not mean a lot of things for me. A paper may be short, but to understand it you may have to read a lot of books. You can read the work of Milnor, he is very well-known for is concise papers which are very well written. Look for example his work on exotic spheres.

Milnor, John W. (1959), "Differentiable structures on spheres", American Journal of Mathematics, 81 (4): 962–972

Tsemo Aristide's user avatar

There is a proof of the non-existence of vector fields on spheres by (Adams and Atiyah ?) that was famously said, could fit on a post card. There is an Indian mathematician named C.P. Ramanujan who wrote a number of very important papers in algebraic geometry and number theory. To the best of my recollection, none of his papers is over 20 pages.

meh's user avatar

The Proceedings of the AMS is all about short papers. You could probably search MathSciNet for publications in the Proceedings and sort by citation number to find some winners.

I'd do this myself, but I don't have a login right now :c

The Noah Sheets helped me a lot in contest math.

Jason Kim's user avatar

  • 1 $\begingroup$ I don't think this qualifies as an answer to the question : it seems to me that OP asks about articles that have had a lot of influence on mathematical research by introducing new ideas. Your link is just a list of known formulae. $\endgroup$ –  Arnaud D. Commented Sep 3, 2018 at 11:49
  • $\begingroup$ Oh... well then I guess I can't contribute... $\endgroup$ –  Jason Kim Commented Sep 3, 2018 at 16:08

Not the answer you're looking for? Browse other questions tagged reference-request soft-question math-history big-list .

  • Featured on Meta
  • Bringing clarity to status tag usage on meta sites
  • Join Stack Overflow’s CEO and me for the first Stack IRL Community Event in...
  • 2024 Election Results: Congratulations to our new moderator!

Hot Network Questions

  • Canonical decomposition as wedge sum up to homotopy equivalence
  • Does this policy mean that my work has flawed password storage?
  • A journal has published an AI-generated article under my name. What to do?
  • 4/4 time change to 6/8 time
  • Why does each leg of my 240V outlet measure 125V to ground, but 217V across the hot wires?
  • Expansion in Latex3 when transforming an input and forwarding it to another function
  • Does the Supremacy Clause allow states to retain abortion rights enshrined in each states' constitution?
  • How would the following changes affect this monster's CR?
  • Getting lost on a Circular Track
  • Inspector tells me that the electrician should have removed green screw from the panel
  • How do I go about writing a tragic ending in a story while making it overall satisfying to the reader?
  • Why does ATSAM3X8E have two separate registers for setting and clearing bits?
  • Remove spaces from the 3rd line onwards in a file on linux
  • What do these expressions mean in NASA's Steve Stitch's brief Starliner undocking statement?
  • What are the steps to write a book?
  • How much does a ma'ah cost in £/$ in today's world?
  • Do images have propositional content?
  • Using "provide" with value of a variable
  • Why were there so many OSes that had the name "DOS" in them?
  • What do you call the act of rebalance an parse tree so simplistic regex are closest to the root node over variables/identifiers?
  • Big Transition of Binary Counting in perspective of IEEE754 floating point
  • Guesstimate a multiple choice exam
  • Torah service on shabbat with no kohanim and not enough yisraelim
  • How solid is the claim that Alfred Nobel founded the Nobel Prize specifically because of his invention of dynamite?

shortest math dissertation

Stack Exchange Network

Stack Exchange network consists of 183 Q&A communities including Stack Overflow , the largest, most trusted online community for developers to learn, share their knowledge, and build their careers.

Q&A for work

Connect and share knowledge within a single location that is structured and easy to search.

Mathematics PhD dissertations that opened a new field of research

I propose this as a companion wiki page to the one about PhD dissertations which contain a solution to an open problem in the style of big-list questions, thinking in terms of the well-known paradigm that splits mathematical research into problem solving and theory building . Theories are at times developed to solve famous open problems, but sometimes the concrete problems they solve are quickly dwarfed by the possibilities that a new theory opens.

Can you name modern mathematicians who already in their PhD theses (or earlier in their career) developed a substantial new theory or laid the foundations of a new field of research?

  • mathematics
  • mathematicians
  • philosophy-of-science

Ali Taghavi's user avatar

  • 9 $\begingroup$ The list will be too long. $\endgroup$ –  Alexandre Eremenko Commented Apr 24, 2018 at 12:11
  • 6 $\begingroup$ en.wikipedia.org/wiki/Tate%27s_thesis $\endgroup$ –  Steve Huntsman Commented Apr 24, 2018 at 12:14
  • 2 $\begingroup$ en.wikipedia.org/wiki/… $\endgroup$ –  Steve Huntsman Commented Apr 24, 2018 at 12:15
  • 2 $\begingroup$ Perhaps Scholze's Perfectoid spaces ? $\endgroup$ –  TKe Commented Apr 24, 2018 at 14:25
  • 2 $\begingroup$ @Steve Huntsman: Shannon's thesis does not qualify: it was a master thesis:-) $\endgroup$ –  Alexandre Eremenko Commented Apr 25, 2018 at 2:52

5 Answers 5

John Forbes Nash Jr. got a Nobel Prize for his.

Nash earned a Ph.D. degree in 1950 with a 28-page dissertation on non-cooperative games. The thesis, written under the supervision of doctoral advisor Albert W. Tucker, contained the definition and properties of the Nash equilibrium, a crucial concept in non-cooperative games. It won Nash the Nobel Memorial Prize in Economic Sciences in 1994.

Community's user avatar

There are many examples. Here are a few that come to mind:

Simon Donaldson's thesis The Yang-Mills equations on Kahler manifolds contains the first major steps in his work on the differential topology of four manifolds. The following paraphrases its abstract. He gave a new proof of a theorem of Narasimhan and Seshadri characterizing those holomorphic bundles over a projective curve that admit a flat connection and used it to prove the simplest interesting case of the conjecture of Hitchin and Kobayashi. He studied the moduli space of self-dual connections on a simply-connected four manifold and used it to deduce obstructions to the realization of a matrix as the intersection pairing on the second cohomology of such a manifold.

John Tate's thesis is another well known example, although I'm not competent even to summarize it. It has its own wikipedia page .

Mikio Sato's doctoral thesis (based on some already published work) introduced the theory of hyperfunctions as boundary values of holomorphic functions. See this survey by P. Schapira and this interview with Sato. (Nothing about Sato's education is standard.)

Dan Fox's user avatar

  • $\begingroup$ I read that article my Mikio Sato. I thought it very interesting. Too bad most famous mathematicians seem to be as unforthcoming as Sato in describing their formative influences. $\endgroup$ –  Mozibur Ullah Commented May 3, 2018 at 4:51

I’ll pick Philippe Delsarte’s 1973 thesis “An algebraic approach to the association schemes of coding theory” which basically expressed classical extremal problems in designs and codes as algebraic questions involving eigenspaces of related association schemes.

Here is a link to a talk on what is now known as “ Delsarte Theory ”.

Maybe not up to the Nash standard, but pretty good for a PhD!

Thomas G. Kurtz ' PhD dissertation at Stanford University (1967) was titled Convergence of operator semigroups with applications to Markov processes . He went on to write with his PhD student Stewart N. Ethier the book Markov Processes: Characterization and Convergence (John Wiley & Sons Inc., 1986), which is "the standard reference for the advanced theory of Markov processes". Its first chapter is Operator semigroups . He made a stellar research career on these foundations. Much of the modern theory of stochastic processes is variations of what he pioneered: "establishing the convergence of Markov processes and characterising the limiting process" (quotes from the Wikipedia page). I do not claim that he single-handedly created the area, but his contribution is immense.

There has been a similar question here: https://www.quora.com/Which-are-the-best-PhD-theses-ever-in-pure-mathematics

I would suggest Kurt Godel. His doctorate thesis proved the completeness theorem, and a year later he published his incompleteness theorems.

MaudPieTheRocktorate's user avatar

Your Answer

Sign up or log in, post as a guest.

Required, but never shown

By clicking “Post Your Answer”, you agree to our terms of service and acknowledge you have read our privacy policy .

Not the answer you're looking for? Browse other questions tagged mathematics mathematicians philosophy-of-science big-list or ask your own question .

  • Featured on Meta
  • Join Stack Overflow’s CEO and me for the first Stack IRL Community Event in...
  • Bringing clarity to status tag usage on meta sites

Hot Network Questions

  • Who was the French detective mentioned in Hitchcock's "Shadow of a Doubt"?
  • Guesstimate a multiple choice exam
  • Why does each leg of my 240V outlet measure 125V to ground, but 217V across the hot wires?
  • sp_blitzlock returns blank data in SQL Managed instance
  • What are the steps to write a book?
  • Why public key is taken in private key in Kyber KEM?
  • Electromagnetic Eigenvalue problem in FEM yielding spurious solutions
  • Engaging students in the beauty of mathematics
  • Please help me identify my Dad's bike collection (80's-2000's)
  • Practice test paper answers all seem incorrect, but provider insists they are ... what am i missing?
  • Key fob frequency filter design
  • How can I get the bounding box of a path (drawn with hobby)?
  • Subtle racism in the lab: how to respond
  • When should I put a biasing resistor - op-amps
  • Is this map real?
  • Why Pythagorean theorem is all about 2?
  • How much could gravity increase before a military tank is crushed
  • Getting lost on a Circular Track
  • Where Does Rashi Mention the Streets of Venice?
  • Why were there so many OSes that had the name "DOS" in them?
  • Using "provide" with value of a variable
  • Why does my Mac's system partition slowly fill up with msu-target-xxxxxxxxx folders in the /private/tmp folder?
  • Torah service on shabbat with no kohanim and not enough yisraelim
  • What is the working justification of this circuit?

shortest math dissertation

The Dramatic and Ultimate Shortening of a Doctoral Dissertation in Mathematics

Taylor & Francis

  • Universidad de Jaén

Discover the world's research

  • 25+ million members
  • 160+ million publication pages
  • 2.3+ billion citations
  • Recruit researchers
  • Join for free
  • Login Email Tip: Most researchers use their institutional email address as their ResearchGate login Password Forgot password? Keep me logged in Log in or Continue with Google Welcome back! Please log in. Email · Hint Tip: Most researchers use their institutional email address as their ResearchGate login Password Forgot password? Keep me logged in Log in or Continue with Google No account? Sign up

shortest math dissertation

  •  | 

Length of the average dissertation

shortest math dissertation

On R is My Friend, as a way to procrastinate on his own dissertation, beckmw took a look at dissertation length via the digital archives at the University of Minnesota.

I’ve selected the top fifty majors with the highest number of dissertations and created boxplots to show relative distributions. Not many differences are observed among the majors, although some exceptions are apparent. Economics, mathematics, and biostatistics had the lowest median page lengths, whereas anthropology, history, and political science had the highest median page lengths. This distinction makes sense given the nature of the disciplines.

I was on the long end of the statistics distribution, around 180 pages. Probably because I had a lot of pictures.

As I was working on my dissertation, people often asked me how many pages I had written and how many pages I had left to write. I never had a good answer, because there’s no page limit or required page count. It’s just whenever you (and your adviser) feel like there’s enough to get a point across. Sometimes that takes 50 pages. Other times it takes 200.

So for those who get that dreaded page-count question, you can wave your finger at this chart and tell people you’re somewhere in the distribution.

  • Revisiting data science, the career
  • Not so likely life of The Simpsons

13 Comments

As a student at Minnesota, I’m not sure our digital archive is representative, as it is optional, and and certain fields are more or less likely (either due to subject matter, or internal politics) to include their works within the archive. Other than that, this is neat!

what do the colors mean? at first glance I thought the colors had to do with subject area, but on closer examination that are same colored major that are very different (i.e. anthropology/aerospace engineering,

The colors look like they are reverse alphabetical.

Kyle, thanks – I figured I was missing something super simple – its been a long day. So the colors are meaningless then?

Yes, the colors are useless. They convey no information that is not already transparently conveyed.

For that matter, the box plot itself is thoroughly obsolete. Read Edward Tufte’s books, specifically chapter 6 of “The Visual Display of Quantitative Information” for the description of the successor to the box plot.

As for the topic of the post, my math dissertation was 88 pages. My advisor’s was (not a typo) 23 pages, double-spaced, including front matter and references.

And Presburger’s was 19 pages or so. But his supervisor, Tarski, would not give him his PhD. He thought the thesis was too short. So there is a lower bound.

would be interesting to see if they have been getting longer over time and by how much by subject

The chart would be more useful if the majors were ordered by median or average dissertatin length. But it is interesting to note that the more mathematical and objective the mayor, the less pages needed. Llongest dissertations: sociology and anthropology. Shortest: biostatistics.

Out of curiosity, Nathan, what was your dissertation on? (Not that I’d understand it anyway!)

Then, there are the departments’ or universities’ format standards. Single spaced? Double spaced? That, of course doubles the plage length. Better, what was the word count in the dissertations?

So with the current trend of dissertation chapters being prepared/formatted as manuscript submissions for journals up front (which have limits to the manuscript length), I would expect to see overall dissertation lengths get shorter over time (to present). Mine started out much longer, but as I started conforming to target journal format standards, I would say the overall length was reduced by a third or so. Is this true outside of ecology as well?

Oh, dear, my dissertation wouldn’t even appear on this chart, but would be off the right edge. And yes, I’m in one of the ‘runs long’ disciplines. Then again, my opus would count as short for, say, a German Habilitation or a French These d’Etat in my field, so it all depends!

This is very interesting. One variable that isn’t accounted for, but varies greatly between institutions and individual authors, is the number of words per page – determined by spacing, margins, font size, and so on. A more helpful plot would count theses by words, instead of pages, as a measure of content.

Become a member. Support an independent site. Make great charts.

See what you get →

Projects by FlowingData See All →

shortest math dissertation

Shifting Incomes for Young People

Compare incomes for young people from the Millennial generation and the baby boomer generation.

shortest math dissertation

Million to One Shot, Doc

Between 2009 and 2014, there were an estimated 17,968 visits to the emergency room for things stuck in a rectum. Here are those things’ stories.

shortest math dissertation

Decade-Long Battle for “Yogurt” vs. “Yoghurt” on Wikipedia

After years of enthusiastic discussions, editors eventually and begrudgingly came to a conclusion.

shortest math dissertation

Income Sources

Most people have a job and receive wages in return, but that starts to change when you get into the higher income groups.

shortest math dissertation

  • Conferences
  • Guide to graduate study
  • The qualifying exam
  • The qualifying exam syllabus
  • How to obtain copies of past PhD theses
  • Birkhoff library
  • Teaching requirements
  • Professional Development
  • Mathematical Job Search Sites
  • Senior faculty research interests
  • Some old qualifying exams
  • Graduation photos
Below is a list of PhD dissertations written by students at the Harvard Department of Mathematics. All scholars can order copies of most Harvard dissertations from 1982 to the present by contacting UMI/ProQuest at 1-800-521-3042. Permission of the author is usually required to copy theses within the last five years. Most PhD dissertations submitted from March 2012 forward are available online in Harvard's central open-access repository. Harvard affiliates with IDs and PINs can access the full text of most Harvard PhD theses since 1990 from the . of the Harvard Mathematics department.
Year Name Advisor Title
2019 Mark Kisin Slopes in eigenvarieties for definite unitary groups
2019 Michael Hopkins Real Orientations of Lubin-Tate Spectra and the Slice Spectral Sequence of a C4-Equivariant Height-4 Theory
2019 Mark Kisin The p-curvature conjecture for the non-abelian Gauss-Manin connection
2019 Curtis McMullen Trees, Berkovich spaces and the barycentric extension in complex dynamics
2019 Horng-Tzer Yau Spectral Statistics of Random d-regular Graphs
2019 Barry Mazur Ramification of the Hilbert Eigenvariety
2019 Joe Harris Stable log surfaces, trigonal covers, and canonical curves of genus 4
2019 Hugh Woodin The Ultrapower Axiom
2019 Martin Nowak Indirect Reciprocity with Optional Interactions and Private Information and Stochastic Evolution of Staying Together.
2019 Shing-Tung Yau Mirror Symmetry, Autoequivalences, and Bridgeland Stability Conditions
2018 Minicozzi Geometric Variational Problems for Mean Curvature
2018 Taubes Several compactness results in gauge theory and low dimensional topology
2018 S-T Yau Picard-Fuchs systems arising from toric and flag varieties
2018 S-T Yau Entire surfaces of prescribed curvature in Minkowski 3-space
2018 Kisin Geometric Properties of Families of Galois Representations
2018 H-T Yau Local statistics of Dyson Brownian motion
2018 M. Hopkins Variations on a Nilpotence Theorem of Hopkins and Mahowald
2018 M.Hopkins Some calculations of cobordism groups and their applications in physics
2018 Curtis McMullen Teichmüller curves in genus two: square-tiled surfaces and modular curves
2018 H.T. Yau Universality of random matrices with dependent entries
2018 Dennis Gaitsgory Nearby cycles of Whittaker sheaves
2017 Mark Kisin The stabilization of the Frobenius-Hecke traces on the intersection cohomology of orthogonal Shimura varieties
2017 Mark Kisin Mod-p isogeny classes on Shimura varieties with parahoric level structure
2017 Mark Kisin The p-curvature conjecture and monodromy about simple closed loops
2017 Michael Hopkins Symmetric Powers and the Equivariant Dual Steenrod Algebra
2017 Jacob Lurie Nilpotence and descent in stable homotopy theory
2017 Mark Kisin Integral canonical models for G-bundles on Shimura varieties of abelian type
2017 Mark Kisin A p-adic Jacquet-Langlands Correspondence
2017 Jacob Lurie The Lubin-Tate Theory of Spectral Lie Algebras
2016 Joe Harris Degenerations, Log K3 Pairs and Low Genus Curves on Algebraic Varieties
2016 Peter Kronheimer On the Framed Singular Instanton Floer Homology from Higher Rank Bundles
2016 Michael Hopkins Equivariant Weiss Calculus and Loop Spaces of Stiefel Manifolds
2016 Mark Kisin Algebraicity criteria and their applications
2016 Joe Harris Derived categories and birational geometry of Gushel-Mukai varieties
2016 Mark Kisin Galois Deformation Ring and Barsotti-Tate Representations in the Relative Case
2016 Alexei Borodin (MIT) q-deformed Interacting Particle Systems, RSKs and Random Polymers
2016 Benedict Gross On the Moy-Prasad filtration and stable vectors
2016 Joe Harris Complete Homogeneous Varieties via Representation Theory
2016 Benedict Gross On the Arithmetic of Hyperelliptic Curves
2015 Dick Gross A formula for some Shalika germs
2015 Clifford Taubes The moduli space of S1-Type zero loci for Z/2 Harmonic spinors in dimension 3
2015 Michael Hopkins Structures on Forms of K-Theory
2015 Dennis Gaitsgory Picard-Lefschetz oscillators for the Drinfeld-Lafforgue-Vinberg compactification
2015 Joe Harris Rational Curves on Hypersurfaces
2015 Dick Gross 2-Selmer groups and Heegner points on eliptic curves
2015 Alexei Borodin Several Theorems about Probabilistic limiting Expressions: The Gaussian free field, symmetric Pearcey process, and strong Szegö asymptotics
2015 Jacob Lurie Goodwillie approximations to higher categories
2015 Joe Harris Covers of an elliptic curve E and curves in E x P1
2015 Richard Taylor Torsion in the Coherent Cohomology of Shimura Varieties and Galois Representations
2015 Joe Harris Interpolation and vector bundles on curves
2015 Jacob Lurie The mod 2 homology of free spectral Lie algebras
2014 S.T Yau Arithmetic Properties of Moduli Spaces and Topological String Partition Functions of Some Calabi-Yau Threefolds
2014 Joe Harris Relative Jacobeans of Linear Systems
2014 Dennis Gatisgory Chiral Principal Series Categories
2014 Joe Harris Regeneration of Elliptic Chains with Exceptional Linear Series
2014 RIchard Taylor Modularity of some elliptic curves over totally real fields
2014 Curt McMullen The geometry of the Weil-Petersson metric in complex dynamics
2014 Marc Kisin The Eigencurve is Proper
2014 Peter Kronheimer Symmetric Spaces and Knot Invariants from Gauge Theory
2014 Curtis McMullen The complex geometry of Teichmuller space
2013 Joe Harris Pencils of quadrics and Jacobians of hyperellipitc curves
2013 Benedict Gross Moduli of Galois Representations
2013 Benedict Gross On Newforms for Split Special Odd Orthogonal Groups
2013 Curt McMullen Entropy, dimension and combinatorial moduli for one-dimensional dynamical systems
2013 Yum-Tong Siu Holomorphically parametrized L2 Cramer's rule and its algebraic geometric applications
2013 Joe Harris The Geometry of Hurwitz Space
2013 Shing-Tung Yau Open Gromov-Witten Invariants on Elliptic K3 Surfaces and Wall-Crossing
2013 Noam Elkies Rational Point Counts for del Pezzo Surfaces over Finite Fields and Coding Theory
2012 Richard Taylor The Arithmetic of Simple Singularities
2012 Peter Kronheimer Towards an Instanton Floer Homology for Tangles
2012 Florian Pop Anabelian Intersection Theory
2012 Shing-Tung Yau Analysis of some PDEs over manifolds
2012 Curtis McMullen Mapping class groups, homology and finite covers of surfaces
2012 Peter Kronheimer Symplectic Rational Blow-Up and Embeddings of Rational Homology Balls
2012 Joe Harris Restrictions of Steiner bundles and divisors on the Hilbert scheme of points in the plane
2012 Joe Harris Alternate Compactifications of Hurwitz spaces
2012 Richard Taylor Local-Global Compatibility and the Action of Monodromy on Nearby Cycles
2012 Martin Nowak Mathematical Models of Cancer
2012 Dennis Gaitsgory D-modules on Spaces of Rational Maps and on Other Generic Data
2011 Pavel Etingof Representations of the rational Cherednik algebras
2011 Clifford Taubes Asymptotic spectral flow for Dirac operators of disjoint Dehn twists
2011 Shing-Tung Yau The Picard-Fuchs systems of Calabi-Yau complete intersections in homogeneous spaces
2011 Dick Gross The Local Langlands Correspondence for Tamely Ramified Groups
2011 Dennis Gaitsgory Twisted geometric Satake equivalence via gerbes on the factorizable grassmannian
2011 Richard Stanley Combinatorial applications of symmetric function theory to certain classes of permutations and truncated tableaux
2011 Shing-Tung Yau Calabi-Yau Geometry and Higer Genus Mirror Symmetry
2011 Barry Mazur On Elliptic Curves, the ABC Conjecture, and Polynomial Threshold Functions
2011 Richard Taylor The Picard-Fuchs systems of Calabi-Yau complete intersections in homogeneous spaces
2011 Shing Tung Yau Existence of Hermitian-Yang-Mills Metrics under Conifold Transitions
2011 Shing-Tung Yau Quasi-local energy in General Relativity
2010 Noam Elkies K3 surfaces of high Picard number and arithmetic applications
2010 Noam Elkies Arithmetic of Elliptic Curves and Surface: Descents and Quadratic Sections
2010 Joseph Harris Moduli of singular curves and crimping
2010 Denis Auroux (MIT) A monoidal structure for the Fukaya category
2010 Horng-Tzer Yau Lower Bound for Ground State Energy of Dilute Bose Gas
2010 Noam Elkies Some combinatorial problems in vector spaces over finite fields
2010 Richard Taylor Modularity Lifting Theorems for Ordinary Galois Representations
2009 Shing-Tung Yau Geometry of complex Monge-Ampère equations
2009 Martin Nowak Evolutionary Dynamics in Structured Populations
2009 Richard Stanley Enumeration of the Distinct Shuffles of Permutations
2009 Cliff Taubes A Number Theoretic Result for Berge's Conjecture
2009 Joe Harris Good Completions of Neron Models
2009 Michael Hopkins Cubical Homotopy Theory and Monoidal Model Categories
2009 Michael Hopkins A generalization of a theorem of Ravenel and Wilson
2009 Barry Mazur Filament Geometry
2009 Richard Taylor Local Universal Deformation Lifting Spaces Of Mod L Galois Representations
2009 Shing-Tung Yau Pseudonorms and Theorems of Torelli Type for Birational Equivalence
2009 Joe Harris Subcanonical Points on Algebraic
2009 Michael Hopkins A Structures on Thom Spectra
2009 Richard Taylor On Potential Automorphy and other topics in Number Theory
2008 Joe Harris Compact Moduli of Singular Curves: A Case Study in Genus One
2008 Barry Mazur Selmer Growth and a Triangulordinary Local Condition
2008 Richard Taylor Arithmetic Compactifications of PEL-type Shimura Varieties
2008 Barry Mazur Algorithms for p-adic Cohomology and p-adic Heights
2008 Gerald Sacks Models with High Scott Rank
2008 Joe Harris Severi Varieties and the Moduli Space of Curves
2008 Joe Harris Covers of Elliptic Curves and Slopes of Effective Divisors on the Moduli Space of Curves
2008 Roman Bezrukavnikov (MIT) Weak Representation of Tangle Categories in Algebraic Geometry
2008 Cliff Taubes Investigation of J-holomorphic curves in M3 x S1
2007 Richard Taylor Counting points on Igusa varieties
2007 Martin Nowak Modeling the Effects of Population Structure and Vaccination Strategy on Infectious Diseases
2007 Barry Mazur The Exceptional Zero Conjecture For Hilbert Modular Form
2007 Peter Kronheimer A slice genus lower bound from sl(n) Khovanov-Rozansky homology
2007 Noam Elkies Minimal Heights and Regulators for Elliptic Surfaces
2007 Joe Harris Enumerative Geometry of Curves with Exceptional Secant Planes
2006 Shing-Tung Yau On Ordinary K3 surfaces over F
2006 Richard Taylor Weight Spectral Sequence and Hecke Correspondence on Shimura Varieties
2006 Shing-Tung Yau On the Geometry of Superstring with Torsion
2006 Peter Kronheimer Estimated Transversality And Rational Maps
2006 Richard Taylor Weights Of Galois Representations Associated To Hilbert Modular Forms
2006 Joe Harris On Zero-Dimensional Schemes with Special Hilbert Functions
2006 Martin Nowak Analysis of Probabilistic Models of Evolution
2006 Noam Elkies K3 surfaces of high rank
2006 Richard Taylor The Weight in Serre Type Conjecture for Tame n-Dimensional Galois Representations
2006 Wilfried Schmid The Degree 4 L-Function of an Automorphic Form of Full Level on the Rank 2 Real Symplectic Group
2006 Joe Harris Extending Families of Curves: Monodromy and Applications
2006 Curtis McMullen Euler Characteristics of Teichm?ller Curves in Genus Two
2005 Cliff Taubes Applications of Chiral Perturbation Theory
2005 Cliff Taubes Pseudoholomorphic Punctured Spheres in the Symplectization of a Quotient
2005 Shing-Tung Yau A Modular Non-Rigid Calabi-Yau Threefold
2005 Joe Harris Moduli Spaces of Curves with Linear Series and the Slope Conjecture
2005 Shlomo Sternberg Morphlets; A Multiscale Representation for Diffeomorphisms
2005 Richard Taylor Kato's Euler System and the Main Conjecture
2005 Richard Taylor Geometricity of Local p-Adic Representations
2004 Joe Harris Special Linear Series in P2
2004 Noam Elkies Elliptic Curves x + y = k with High Rank
2004 Cliff Taubes Perturbations of the D-bar Operator
2004 Barry Mazur Geometric and p-Adic Modular Forms of Half-Integral Weight
2004 Peter Kronheimer Contact Structures and Floer Homology
2004 Curtis McMullen Simple geodesics on hyperbolic surfaces and the volume of the moduli space of curves
2004 Shing-Tung Yau Intersection Theory on the Moduli Space of Stable Bundles via Morphism Spaces
2004 Peter Kronheimer A Spectrum Valued RQFT from the Seiberg-Witten Equations
2004 Curtis McMullen Complex Projective Structures
2004 Joe Harris Degenerations of scrolls and Del Pezzo Surfaces and Applications to Enumerative Geometry
2003 Benedict Gross The Fourier-Jacobi map and small representations
2003 Peter Kronheimer Floer Homology and Knot Complements
2003 Benedict Gross Central value of Rankin L-series over real quadratic fields
2003 Benedict Gross Quaternion Rings
2003 Barry Mazur Generalization of the Turan and the Erdos-Kac Theorems
2003 Shing-Tung Yau Deformations of G and Spin(7) Structures on Manifolds
2003 Noam Elkies Supersingular primes for rational points on modular curves
2003 Barry Mazur Local and global points on moduli spaces of abelian surfaces with potential quaternionic multiplication
2003 Shing-Tung Yau Li-Yau-Hamilton Estimate for the Ricci Flow
2003 Shing-Tung Yau Flops and Equivalences of derived Categories for Threefolds with only terminal Gorenstein Singularities
2002 Benedict Gross Hecke Algebra action on Siegel Modular Forms
2002 Barry Mazur On u-Invariants of Elliptic Curves over Q
2002 Cliff Taubes Closed Self-Dual Two-Forms on Four-Dimensional Handlebodies
2002 Richard Taylor On Certain Unitary Group Shimura Varieties
2002 Shing-Tung Yau Moduli of J-Holomorphic Curves with Lagrangian Boundry Conditions
2002 Wilfried Schmid A Localization Argument for Characters of Reductive Lie Groups
2002 Yum-Tong Siu Effective Schottky problem
2002 Curtis McMullen Holomorphic families of Rational Maps: Dynamics Geometry and Potential Theory
2002 David Kazhdan Fourier Transform for Quantized Completely Integrable Systems
2001 David Kazhdan Hodge Structure on the Fundemental Group and its Application to p-adic Integrarion
2001 Richard Taylor Modularity of some potentially Barsotti-Tate Galois representations
2001 Barry Mazur On the p-adic L-function of a Modular Form at a Supersingular Prime
2001 Richard Taylor Local Level-Raising for GL
2001 Barry Mazur Modular Varieties and Visibility
2000 Shing-Tung Yau The Global Nonlinear Stability of the Trivial Solution of the Einstein-Maxwell Equations
2000 Barry Mazur On Selmer groups of geometric Galois Representations
2000 Shing-Tung Yau Compact Manifolds with Exceptional Holonomy
2000 Joe Harris Rational Curves on Hypersurfaces in Pn
2000 Shlomo Sternberg Symmetric Space Valued Moment Maps
2000 Peter Kronheimer PU(2) monopoles on Kahler surfaces
2000 Richard Taylor On the Modularity of Certain 2-adic Galois Representations
2000 Noam Elkies New Bounds on Sphere Packings
2000 Raoul Bott Equivariant de Rham Theory and Statinary Phase Expansions
1999 Joe Harris Exact Rates of Convergence for Some Simple Non-Reversible Markocv Chains
1999 Cliff Taubes Configuration space and Monopoles for Yang-Mills-Higgs Theory on R /M
1999 Barry Mazur Flat Regular Models of Elliptic Schemes
1999 Joe Harris Monomial Ideals and Hilbert Schemes
1999 Barry Mazur Jacobians of Curves of Genus One
1999 Joe Harris Moduli Spaces of Curves with Marked Points
1999 Shing-Tung Yau Applications of Affine Differential Geometry to RP(2) Surfaces
1999 Cliff Taubes Dirac Operators on Loop Spaces
1999 Benedict Gross Models over Z for Locally Quasi-Split Algebraic Groups
1999 Shing-Tung Yau Picard-Fuchs Uniformation and Geometric Isomonodromic Deformations
1999 Joe Harris Moduli of curves with level structure
1999 Edward Frenkel Spectral Curves
1998 Richard Melrose (MIT) Microlocal Analysis of the Time-Dependent Schroedinger Operator
1998 Persi Diaconis Eigenvalue Distribution of Random Matrices in Permutation Group and Compact Lie Groups
1998 Shing-Tung Yau Generalized Harmonic Maps and Representations of Discrete Groups
1998 Shing-Tung Yau Topology of Birational Manifolds and Applications to Degeneration
1998 David Mumford Curvature Motions, Medial Axes and Distance Transforms
1998 Joe Harris Calculations on the Moduli Space of Genus Zero Covers
1998 Koszul Property and Bogomolov's Conjecture
1998 Benedict Gross Explicit Hecke Actions on Modular Forms
1998 Benedict Gross Traces of Hecke Operators
1998 Joe Harris Moduli Space of Enriched Stable Curves
1998 Benedict Gross Hecke rings of Groups over Local Fields
1998 Cliff Taubes Reidemeister Torsion in Generalized Morse Theory
1998 Benedict Gross Spin Representations and Lattices
1998 Benedict Gross Exceptional Theta Correspondences
1998 Barry Mazur 2-adic Modular Forms of Minimal Slope
1998 Barry Mazur Hilbert Modular Forms and the Galois Representations Associated to Hilbert-Blumenthal Abelian Varieties
1998 Benedict Gross Variation of capacity for Convex Domains in Euclidean Space
1997 David Kazhdan Integral Motives of Quadrics
1997 Joe Harris Enumerative Geometry of Curves via Degeneration Methods
1997 David Kazhdan Short Time Behavior of Logarithmic Derivatives of the Heat Kernel
1997 Joe Harris Moduli of Trigonal Curves
1997 Ehud Hrushovski Model Theory of Valued D-Fields
1997 Persi Diaconis Random Walks on Groups: Strong Uniform Time Approach
1997 Benedict Gross Abelian L-Functions Twisted by Algebraic Tori A+S=0
1997 Benedict Gross Exceptional Lie Groups and Lie Algebras
1997 Robert MacPherson A Generalization of Springer Theory using Nearby Cycles
1997 Persi Diaconis Probability in the Classical Groups over Finite Fields: Symmetric Functions Stochastic Algorithms and Cycle Indices
1997 Benedict Gross p-adic Gamma Functions
1997 Joe Harris Rational Curves of K3 Surfaces
1996 Persi Diaconis Socks and Boxes: Variations on Daniel Bernoulli's Marriage Problem
1996 Persi Diaconis Weighted Poincare and Exhaustive Techniques for Scaled Metropolis-Hastings Algorithms and Spectral total Variation Convergence Bounds in Infinite Commutable Markov Chain Theory
1996 David Kazhdan Biextension Weil Representations on Derived Categories and Theta Functions
1996 Persi Diaconis Berry-Essen Central Limit Theorem for Markov Chains
1996 Persi Diaconis Random Walks on the Symmetric Group Generated by Conjugacy Classes
1996 Shing-Tung Yau Wall Crossing Formula of Seiberg-Witten Invariants and Symplectic Four Manifolds with b+1=1
1996 Joe Harris Special Cubic Hypersurfaces of Dimension Four
1995 David Kazhdan Cohomology of compact Hyperkaehler Manifolds
1995 Persi Diaconis Methods for Quantifying Rates of Convergence for Random Walks on Groups
1995 Joe Harris The Grothendieck Quot Schemes and Composition Laws for Grassmannians
1995 David Kazhdan Minimal Representations of Exceptional p-adic groups
1995 Shing-Tung Yau On the Convergence of Kahler Metrics
1995 Persi Diaconis Topics in Probability on Compact Lie Groups
1995 Persi Diaconis Walks and Representation Theory
1995 Cliff Taubes A Mayer-Vietoris Principle for Monopoles
1994 Benedict Gross A-divisible Modules
1994 Barry Mazur Canonical Heights and Rational Points on Varieties with Many Elliptic Fibrations
1994 Wilfried Schmid Real Forms of Quantum Groups and Harish-Chandra Modules
1994 Raoul Bott Lie Algebra Cohomology and the Fusion Rules
1994 Joe Harris A Compactification over the Moduli space of Stable curves of the Universal Moduli Space pf Slope-Semistable Vector Bundles
1994 Joe Harris The Moduli Space of (3,3,3) Trilinear Forms
1994 Benedict Gross Multiplicities in restricted Representations of GL (F ) U (F ) and SO(F )
1994 Joseph Bernstein Fusion Categories
1994 Raoul Bott Some explicit Cocycles for Cohomology Classes of Groups of Diffeomorphisms Preserving a G-Structure
1994 Cliff Taubes Symplectic Geometry and the Relative Donaldson Invariants of the Conjugate Projective Plan
1994 Yum-Tong Siu An Effective Polynomial Bound for Base Point Freeness and Point Separation of Adjoint Bundles
1993 Cliff Taubes Essays on Vortices Knots and Monopoles
1993 Raoul Bott Group Actions and Cohomology
1993 David Kazhdan Graded Lie Algebras and conformal Field theories l: The Genus O Case
1993 Persi Diaconis Fast Transforms and Sampling for Compact groups
1993 J. Bernstein Formulas for Generalized Kazhdan-Lusztig Polynomials
1993 Shing-Tung Yau On Modular Invariants and Rigidity Theorems
1993 Shlomo Sternberg Hamiltonian Actions of Lie Groups
1993 Yum-Tong Siu Global Nondeformability of the Complex Hyperquadric
1993 Cliff Taubes On the ends of the monopole Moduli Space
1993 Benedict Gross The 2nd Descent for Certain Families of Mordell-Weil Lattices
1993 Victor Kac Differentiably Simple Lie Superalgebras
1993 Joe Harris On a Compactification of the Universal Picard variety over the Moduli space of Stable Curves
1993 Persi Diaconis Rates of Convergence of Markov Chains Related to Association Schemes
1992 Wilfried Schmid Dolbeault Cohomologies and Zuckerman Modules Associated with Finite Rank Representations
1992 Raoul Bott Extension of Self-Dual Yang-Mills Equations across the Eight Dimension
1992 Barry Mazur Higher-Order Characteristic Classes in Arithmetic Geometry
1992 David Kazhdan Homology of Schemes and covariant Motives
1992 Raoul Bott The Verlinde Formulas and Moduli Spaces of Vector Bundles
1992 Heisuke Hironaka Functional Smoothing of Morphisms in Equal Characteristic 0
1992 Persi Diaconis Rates of Convergence for Gibbs Sampler and other Markov Chains
1992 Barry Mazur Division Points on Semi-Abelian Varieties
1992 Benedict Gross A p-adic Computation of Singular Moduli
1992 Joe Harris Hilbert and Functions of Zero-Dimensional Schemes in Uniform Position
1992 Joe Harris On the Dimension of the Chow Varieties
1991 Cliff Taubes Yang-Mills Connections with Asymptotically Constant Curvature
1991 Barry Mazur Power-free Values of Polynomials
1991 Joe Harris On the Hyperplane sections of a Variety in Projective Space
1991 Joseph Bernstein Affine Kac-Moody Algebras at the Critical Level and Quantum Drinfield-Sokolov Reduction
1991 Benedict Gross Refined Class Formulas for Derivatives of L-Series
1991 Barry Mazur Weierstrass Points on Arithmetic Surfaces
1991 Barry Mazur An Arithmetic Riemann-Roch Theorem for Singular Arithmetic Surfaces
1991 Joe Harris Subvarieties of Abelian Varieties and of Jacobians of Curves
1990 David Mumford Image Segmentation by Variational Methods and Elliptic Boundary Value Problems
1990 John Tate Refined Conjectures of the Birch and Swinnerton-Dyer Type
1990 Cliff Taubes Nonexistence of Almost Handebody Structures on Topological Four Manifold Pairs
1990 Joseph Bernstein Deligne's Conjecture in the Constant Coefficient Case
1990 Shing-Tung Yau Ricci Deformation of the Metric on Complete Noncompact Kahler Manifolds
1990 Barry Mazur Igusa Towers over Hilbert Modular Surfaces
1990 Mark Spivakovsky On Some Conditions of Regularity for Subanalytic Sets
1990 Shing-Tung Yau On Meromorthic Maps between Algebraic Varieties with Log-General Targets
1990 Shing-Tung Yau Construction of Stable Vector Bundle on Surface of Low 2nd Chern Classification
1990 Joe Harris Divisors on Some Moduli Spaces
1990 Persi Diaconis Rates of Convergence of Some Random Processes on Finite Groups
1990 John Tate Spaces of Rational Functions on Curves over Finite Fields
1989 Shing-Tung Yau Semipositive Threefolds and Threefolds with Universal covering C(3)
1989 Persi Diaconis Fast Fourier Analysis for Finite Groups
1989 Benedict Gross Shimura Curves Analogous to X (N)
1989 Benedict Gross Trilinear Forms for GL (2) of a Local Field and Epsilon Factors
1989 Raoul Bott Hilbert's Third Problem and the K Theory of Toric Varieties
1989 Raoul Bott Integral Homology of Real Flag Manifolds and Loop Spaces of Symmetric Spaces
1988 Heisuke Hironaka Newton Polyhedra without Coordinates
1988 Robin Hartshorne Cohomology of Normal bundles of curves in P and other Topics
1988 Raoul Bott Three Topics on Perturbation Analysis of Discrete Event Dynamic Systems
1988 Sing-Tung Yau Kaehler Metrics on Algebraic Manifolds
1988 Limit Multiplications of Cusp Forms
1988 The Behavior of Stability under Equivariant Maps
1988 Yum-Tong Siu Hyperbolic Surfaces
1988 Ordered and unordered Periodic Points of Maps
1987 John Tate Gamma Functions and Gauss Sums for Function Fields
1987 Wilfried Schmid Systems of Hodge Bundles and Uniformization
1987 Benedict Gross Lifting Endomorphisms of Formal Groups
1987 Barry Mazur Iwasawa theory at Multiplicative Primes
1987 Benedict Gross Heights and L-Series
1987 Barry Mazur Arithmetic of p-adic Modular Forms
1987 Arthur Jaffe Degree Theory of Wiener Maps and Supersymmetric Quantum Mechanics
1987 Nancy Ann Lynch (MIT) Topics in Distributed Algorithms
1987 Benedict Gross and Barry Mazur Supersingular Primes of a Given Elliptic Curve over a Number Field
1987 Barry Mazur Deformation theory of Galois Representations
1987 Raoul Bott Excursion in Cyclic Homology of Topological Algebras
1987 Wilfried Schmid Connections between Representations of Lie Groups and Sheaf Cohomology
1986 David Mumford Applications of Equivariant Morse Stratifications
1986 Nondegeneracy of Infinitesimal Invariants Associated to Normal Functions
1986 John Tate p-adic Periods as Moduli for Schottky Curves of Genus Two
1986 Shlomo Sternberg On Supergeometric Structures
1986 Shannon Entropy and the Central Limit Theorem
1986 Philip Griffiths On Local Torelli for Extremal Varieties
1986 Stability and Canonical Metrics in Infinite Dimensions
1986 Local Heightson Families of Abelian Varieties
1985 Heisuke Hironaka Sandwiched Singularities and the Nash Resolution for Surfaces
1985 Bounds on Irregularity of Surfaces of General Type
1985 David Mumford Prym Varieties and the Geodesic Flow on SO(n)
1985 Heisuke Hironaka Rank Conditions on Sub-Varieties of Grassmannians
1985 David Mumford Families of Rational Maps and Iterative Root-Finding Algorithms
1985 David Kazhdan On the Characters of the Representations of Division Algebras over a Weak Field
1985 Barry Mazur On the Canonical Closure of the Universal Elliptic Curve over Xl(n)
1985 Raoul Bott Functional Determinants and Applications to Geometry
1985 Yum-Tong Siu Positivity of the Curvature of the Weil-Petersson Metric on the Moduli Space of Stable Vector Bundles
1985 On Higher Order Conservation Laws
1985 Wilfried Schmid Special K-Types and the Beilinson-Bernstein Realization
1984 Raoul Bott Singularities in Moduli Spaces of Yang-Mills Fields
1984 Characterization of Jacobian Varieties in Terms of Soliton Equations
1984 Barry Mazur The Tate-Shafarevich Group of the Jacobian of a Quotient of the Fermat Curve
1984 Wilfried Schmid Discrete Characters on Non-Riemannian Symmetric Spaces
1984 Barry Mazur Analytic Multiplicity One Theorems for GL (n)
1984 Wilfried Schmid Hyperfunction Solutions of the Zero Rest Mass
1984 Barry Mazur On Zeta Functions Associated with the Space of Binary Cubic Forms with Coefficients in a Function Field
1984 David Mumford Compactification of Siegel Moduli Schemes
1984 Gerald Sacks B-Degrees for Weakly Inadmissible B
1983 Involutive Hyperbolic Differential Systems
1983 Barry Mazur Integral Points on Varieties
1983 David Mumford Construction of Holomorphic Differential Forms on the Moduli Space of Abelian Varieties
1983 David Mumford Hyperelliptic Curves and Solitons
1983 David Kazhdan Etale Cohomology and Arithmetic of Semisimple Groups
1983 David Mumford Topics in Algebraic Geometry
1983 An Analogy of the Penrose Correspondence for Representations of U(p,q) on L2-Cohomology
1983 David Kazhdan Theory of e-representations
1983 Shlomo Sternberg Integral Geometry on Compact Symmetric Spaces
1982 Barry Mazur Dirichlet Series Associated with the Space of Binary Cubic Forms with Coefficients in a Number Field
1982 John Tate The Neron-Tate Height On Elliptic Curves
1982 George Mackey Von Neumann Algebras Associated to Ergodic Actions of Countable Groups
1982 John Tate Algebraic Cycles on Abelian Varieties
1982 Barry Mazur On Certain Covers of the Universal Elliptic Curve
1982 Raoul Bott Homogeneous Connections and Yang-Mils Theory on Homogeneous Spaces
1982 Raoul Bott Hamiltonian Mechanics and Optimal Control
1982 Phillip Griffiths Special Fibers in Families of Plane Curves
1982 Gerald Sacks Independence Results Concerning Some Combinatorial Properties of the Continuum
1982 The Gauss Map and Isometric Embedding
1982 Heisuke Hironaka The Kahler Algebra and Analytic Equivalence of Isolated Hypersurfaces Singularities
1982 Heisuke Hironaka The Division Algorithm and the Hilbert Scheme
1981 Barry Mazur On Congruences Satisfied by Special Values of L-functions
1981 Gerald Sacks Aspects of E-Recursion
1981 On the Arithmetic of CM Elliptic Curves in Zp-Extensions
1981 Barry Mazur On J1(p) and the Arithmetic of the Kernel of the Eisenstein Ideal
1981 Barry Mazur On the Diophantine Arithmetic of Shimura Curves
1981 Raoul Bott Equivariant Morse Theory and Closed Geodesics
1981 Gerald Sacks Axiomatic Definitions of Programming Languages and Logics of Programs
1981 Akihiro Kanamori Iterated Perfect Set Forcing and Degrees of Constructibility
1981 Phillip Griffiths Hodge Theory
1981 John Tate Stark's Conjecture
1980 Phillip Griffiths Variation of Hodge Structure and the Local Torelli Problem
1980 Phillip Griffiths Semistable Degenerations of Enriques and Hyperelliptic Surfaces
1980 Andrew Gleason Stochastic Games: The Minmax Theorem
1980 Shlomo Sternberg Integral Geometry of Plane Complexes
1980 The Kaehler algebra and analytic equivalence of isolated hypersurface singularities
1979 Gerald Sacks Predicate-Oriented Database Search Algorithms
1979 Andrew Gleason Weakened Topologies for Lie Groups
1979 Phillip Griffiths Associated Curves and Plucker Formulas in Grassmannians
1978 Shlomo Sternberg Geometry of the Adjoint Representation of a Complex Semisimple Lie Algebra
1978 Andrew Gleason Zeros and Growth of Entire Functions of Order One and Maximal Type With an Application to the Random Signs Problem
1978 David Mumford Projective Stability of Ruled Surfaces
1978 David Mumford Quasi-Elliptic Surfaces in Characteristic Three
1978 Phillip Griffiths A Bound on the Geometric Genus of Projective Varieties
1978 John Tate Arithmetic on Elliptic Curves with Complex Multiplication
1978 The Applications of Algebraic K-Theory to Intersection Theory
1978 Barry Mazur Some Theorems on Azumaya Algebras
1978 Essential Singularities of Entire Analytic Varieties
1977 Richard Brauer Fusion in Nonabelian Groups of Order p3
1977 John Tate Some Results on Classical Eisenstein Series and Modular Forms over Function Fields
1977 John Tate On the Local Langlands Conjecture for GL(2)
1977 John Tate Orbital Integrals on GL3
1977 The Hodge Theory of Flat Vector Bundles on a Complex Torus
1977 Barry Mazur On p-adic Representations Arising From Descent on Abelian Varieties
1977 Barry Mazur P-adic Eisenstein Series for Function Fields
1977 John Tate A Comparison of the Automorphic Representations of GL (3) and its Twisted Forms
1977 Phillip Griffiths On the Geometry of Grassmannians
1977 Lars Ahlfors Deformations of Lie Groups and Lie Algebras
1977 John Tate Icosahedral Galois Representations
1977 Raoul Bott On the Smooth Cohomology of Groups of Diffeomorphisms
1976 Raoul Bott Refined Chern Classes for Foliations
1976 Heisuke Hironaka Deformations of L-cycles and the Chow Scheme
1976 David Mumford On Arithmetic Quotients of Bounded Symmetric Domains
1976 George Mackey Ergodic Actions of Product Groups
1976 Garrett Birkhoff Symmetry Groups of Partial Differential Equations
1976 Shlomo Sternberg Bernstein Polynomials and the Gauss-Manin Connection
1976 Barry Mazur Congruences Between Modular Forms and Implications for the Hecke Algebra
1976 Heisuke Hironaka Homology and Combinatorics of Ordered Sets
1975 George Mackey Ergodic Group Actions with Generalized Discrete Spectrum
1975 Shlomo Sternberg A Limit Theorem for Conditional Expectations with Applications to Probability Theory and Statistical Mechanics
1975 Phillip Griffiths Holomorphic Pseudogroup Structures on Quasiprojective Varieties
1975 Andrew Gleason Some Aspects of the Four Color Problem
1975 Analytic Theory of Elliptic Curves with Split Multiplicative Reduction over Complete Rings
1975 David Mumford On Degenerations of Algebraic Surfaces
1975 Raoul Bott Cohomology of Hamiltonian and Related Formal Vector Field Lie Algebras
1975 David Mumford Curvature on Algebraic Plane Curves
1975 Raoul Bott Continuous Cohomology of Spaces with Two Topologies
1975 Barry Mazur Diophantine and p-adic Analysis of Elliptic Curves and Modular Forms
1975 George Mackey Haar Measure and Convolution Algebras on Ergodic Groupoids
1975 David Mumford Cohomology of Flag Varieties on Characteristic p
1975 Barry Mazur Linking the Conjectures of Artin-Tate and Birch-Swinnerton-Dyer
1975 Richard Brauer On a Problem of E. Artin
1975 David Mumford Endomorphisms of Abelian Schemes
1975 Lynn Loomis Non-Linear Evolution Equations with Variable Norms
1975 Andrew Gleason Topics in Liftings and Stochastic Processes
1975 Andrew Gleason Small Rings in Critical Maps
1975 Phillip Griffiths Intrinsic Metrics and Measures on Compact Complex Manifolds
1975 David Mumford Polyhedral Reduction Theory in Self-Adjoint Homogeneous Cones
1975 Phillip Griffiths On the Theorem of Frenet
1974 Lynn Loomis Some Aspects of Balayage of Fourier Transforms
1974 David Mumford Deformations of Algebraic Varieties with Gm Action
1974 David Mumford P-adic Schottky Groups
1974 John Tate Two-descent for Elliptic Curves in Characteristic Two
1974 Shlomo Sternberg Ideals of Orbits of Nilpotent Lie Algebras
1974 Raoul Bott The Cohomology of Certain Algebraic Varieties
1974 Richard Brauer On a problem of E. Artin
1974 Frederick Mosteller Weak and Strong Averages in Probability and the Theory of Numbers
1974 Saunders Mac Lane On the Cohomology Theory of Fields
1974 On the 2-primary Part of K2O and on Z2-extensions for Imaginary Quadratic Fields
1974 Stability of the Cut Locus
1973 George MacKey Ergodic Affine Lebesgue G-Spaces
1973 Andrew Gleason Function Algebra Extensions and Analytic Structures
1973 Some Aspects of Balayage of Fourier transforms
1973 Subalgebras of Division Algebras
1973 Galois Action on Division Points of Abelian Varieties with Many Real Multiplications
1973
1973 David Mumford p-Adic Schottky Groups
1973 Raoul Bott Local Isomorphism of Riemannian Hermitian and Combinatorial Manifolds
1973
1973 Transversality Properties of Topologically Stable Mappings
1973 Combinatorial Aspects of Lattice Theory with Applications for the Enumeration of Free Distributive Lattices
1973 Shlomo Sternberg Topics on Universal Algebra
1973 David Mumford Valuative Criteria for Families of Vector Bundles on Algebraic Varieties
1973 Barry Mazur Universal Bounds on the Torsion and Isogenics of Elliptic Curves
1973 Andrew Gleason Interpolating Sequences in Polydisks
1973
1973
1973 Andrew Gleason Construction of Rings in Modular Lattices
1973 George Mackey On the Virtual Groups Defined by Ergodic Actions of R and Z
1973 Convergence from an Algebraic Point of View
1973
1973 Iwasawa Invariants in Z1 extensions of Number Fields
1972 The Bracket Ring and Combinatorial Geometry
1972 George Mackey Liftings in the Category of C* Algebras
1972 Triangulation of 3-Manifolds: A Piecewise Linear Approach
1972 John Coates Elliptic Curves of Prime Conductor
1972 Shlomo Sternberg Invariant Theory and the Cohomology of Infinite Lie Algebras
1972 David Mumford Non-singular Deformations of Space Curves using Determinantal Schemes
1972 John Tate Odd Perfect Numbers are Divisible by at least Seven Distinct Primes
1972 Local Cohomology Dimension of Algebraic Varieties
1972
1972 Andrew Gleason Direct Decompositions of Commutative Monoids
1972 David Mumford Towards Projectivity of the Space of Moduli of Stable Curves of a Given Genus
1972
1972 David Mumford A Study of Three-Dimensional Principally-Polarized Abelian Varieties
1972 Raoul Bott Curvature and the Eigenvalues of the Laplacian for Geometrical Elliptic Complexes
1972 Triangulations of Two Manifolds with Local Properties
1972 Thom Polynomials for Contact Class Singularities
1972 David Mumford Local Moduli of Abelian Varieties
1972 Cauchy Problems with Random Initial Values in the Space of Tempered Distributions
1971 General-Valued Polarities
1971 David Mumford Deformations of Branched Covers and Equisingularity
1971 Ordered Structures and Partitions
1971 Lynn Loomis Complex Involutory Banach Algebras
1971 John Tate The Non-vanishing of L(1) for Certain Elliptic Curves
1971 Round-off Error in the Numerical Solution of Retarded Ordinary Differential Equations
1971
1970 Lars Ahlfors Probabilistic Methods in Combinatorial Theory
1970 Andrew Gleason Properties of Isometrics and Almost Isometrics of Some Function Algebras
1970 Partial Algebras
1970 Raoul Bott Singularities of Maps and Characteristic Classes
1970 Numerical Invariants and Gamma Products of Graphs
1970 David Mumford Contributions to the Theory of Positive Embeddings in Algebraic Geometry
1970 Lars Ahlfors Special Moduli and Theta Relations
1970 Andrew Gleason Homologies and Elations of Finite Projective Planes
1970 Raoul Bott Orderings of Ultrafilters
1969 Andrew Gleason The Structure Space of a Choquet Simplex
1969 David Mumford Ordinary Singularities of Projective Varieties
1969 Barry Mazur On the Structure of Purely Inseparable Field Extensions
1969 Andrew Gleason Individuals in Zermelo-Fraenkel Set Theory
1969
1969 Andrew Gleason Functional Differential Equations with General Perturbation of Argument
1969 Richard Brauer Finite Linear Groups in Six Variables
1969 Mathematical Models of Economic Growth
1969 Numerical Invariants and Multiple Planes
1969
1969 David Mumford The Picard Scheme of a Quotient Problem
1969 Raoul Bott Cobordism
1969 Raoul Bott The Cohomology of the Complement of a Submanifold
1968 John Tate Abelian Varieties over Finite Fields
1968 Compactness in Relational Structures
1968 Barry Mazur Algebraic K of Vector Bundles
1968 On the Flat Cohomology of Finite Group Schemes
1968 P-adic Theta Functions
1968 Shlomo Sternberg Curvature and Metric
1968 The Structure of a Banach Algebra Invariant for Measure Preserving Automorphisms
1968 Nonunitary Representations and Harmonic Analysis of Some Solvable Lie Groups
1968 Shlomo Sternberg The Coholomogy Theory of Transitive Moduales over the Primitive Infinite Lie Algebras
1968 George Mackey Measures on Locally Compact Groups with Certain Transformation Properties
1968 Semigroup Product Formulas and Addition of Unbounded Operators
1968 David Mumford On the Structure of Locally Compact Groups
1968 Commuting Elements in Free Algebras and Related T+E225opics in Ring Theory
1967 Complete Abelian Groups and Direct Sum Decompositions
1967 Richard Brauer Finite Linear Groups in Seven Variables
1967 Lars Ahlfors The External Property of Certain Teichmüller Mappings
1967 John Tate On Automorphisms of Local Fields
1967 Andrew Gleason Approximation Theory on Compact Manifolds and Lie Groups with Applications to Harmonic Analysis
1967 David Mumford Abelian Varieties over a Perfect Field and Dieudonne` Modules
1967 The Conjecture of Birch and Swinnerton-Dyer for Constant Abelian Varieties over Function Fields
1967 Homemorphisms of Sn x Sl
1967 Andrew Gleason Radical Banach Algebras and Formal Power Series
1967 Shlomo Sternberg Overdetermined Systems of Analytic Partial Difference Equations
1967 Cohomology Rings of Commutative Formal Groups
1966 Existence Theorems for Surfaces of Constant Mean Curvature and Perturbations of a Liquid Globule in Equilibrium
1966 Andrew Gleason Powers of Maximal Ideals in Function Algebras
1966 Lynn Loomis Unbounded Normal Operators on Banach Spaces
1966 Raoul Bott Secondary Characteristic Classin K-Theory
1966 David Mumford On Hilbert Schemes
1966 On the Minimum Computation Time of Functions
1966 Topological Methods in the Algebraic Theory of Vector Lattices
1966 Lars Ahlfors Conformal Invariants for Condensers
1965 The Convergence of Difference Approximations to Cauchy Problems in the Space of Tempered Distributions
1965 Quasi-ordinary Singularities of Embedded Surfaces
1965 Raoul Bott Kunneth Formulas for Bordism Theories
1965 Classification of Demushkin Groups
1965 Oscar Zariski Toward a Numerical Theory of Ampleness
1965 Lynn Loomis Types of Completeness in Locally Convex Spaces
1965 Lynn Loomis Decomposition of Centrally Reducible and of Reducible Functionals
1965 Andrew Gleason Functions Resembling Quotients of Measures
1964 Richard Brauer Character Sums and Difference Sets
1964 John Tate Infinitesimal Deformations of Singularities
1964 Raoul Bott Formal Theory of Linear overdetermined Systems of Partial Differential Equations
1964 Richard Brauer Groups and Group Rings
1964 John Tate Curves over Discrete Valuation Rings
1964 Richard Brauer Characters and Systems of Subgroups
1964 Lynn Loomis Modules of Holomorphic Vector-Valued Functions
1964 Richard Brauer P-Solvable Linear Groups
1964 Raoul Bott Classification of certain Types of Spaces
1964 Integral Equations Associated with Hankel Convolutions
1964 Rational Approximations to Generalized Hypergeometric Functions
1964 Convolution Transforms whose Inversion Functions have Complex Roots
1963 Raoul Bott Categories in Homotopy Theory
1963 Enumerating p-Groups
1963 Convex Functions and Dual Extremum Problems
1963 John Tate Curves of Genus 3 over Characteristic 2
1963 Polynomial Bases for Compact Sets in the Plane
1963 Theory of Covering Spaces
1963 John Tate One-parameter Formal Lie Groups over p-adic Integer Rings
1963 Lynn Loomis Almost Periodic Functionals in the Conjugate Space of a Banach Algebra
1963 Lars Ahlfors Elementary Moduli for Teichmüller Space
1963 Andrew Gleason Natural Functors in Topology and Generalizations
1963 The Curves of Genus 3 Defined over Z/2Z
1963 Richard Brauer On Finite Linear Groups whose Order contains a Prime Larger than the Degree
1963 Applications of the Herbrand Theorem
1963 Raoul Bott Spaces of Paths on a Symmetric Space
1963 Richard Brauer On the Subgroups of SL (3,q)
1962 Richard Brauer On Ree's Series of Simple Groups
1962 Cohomology of Artinian Group Schemes over Local Fields
1962 The Shape of Level Loci of Green's Function and other Harmonic Functions
1962 Recursive Well Orderings and Transfinite Progressions
1962 Lynn Loomis Existence and Applications of Dimension Functions in Lattices
1962 Oscar Zariski Existence of the Moduli Scheme for Curves of any Genus
1962 Lars Ahlfors An External Length Problem and the Bilinear Relation on Open Riemann Surfaces
1962 Andrew Gleason Combinatorial Problems of Elementary Abelian Groups
1962 Lynn Loomis Atomic Orthocomplemented Lattices
1962 On the Global Existence of Solutions of Quasi-Linear Parabolic Equations
1962 Shlomo Sternberg Theory of Finite G-Structures
1962 Hierarchies in Recursive Function Theory
1962 On the Representations of C-Algebras
1962 Lars Ahlfors Teichmüller Spaces of Groups of the Second Kind
1962 The Asymptotic Behavior of Some Non-linear Autonomous Systems
1961 On Conformal Maps of Regions of Infinite Connectivity
1961 John Tate Cohomology of Abelian Varieties over Function Fields
1961 Lynn Loomis The Radon-Nikodym Theorem in Dimension Lattices
1961 On Differentials in Function Fields
1961 Raoul Bott Applications of Intersection Theory to Boundary Value Problems
1961 Raoul Bott The Lower Central Series for Free Group Complexes
1961 Some Results in the Cohomology Theory of Finite Groups
1960 Lars Ahlfors A Classification of Noncompact Surfaces
1960 John Moore Extensions and Cohomology Theory of Locally Compact Groups
1960 Minimal and Relatively Minimal Models for the Function Field of an Algebraic Surface
1960 George Mackey Multipliers on Abelian Groups
1960 Oscar Zariski On the Theory of Birational Blowing-Up
1960 A Characterization of Certain Banach Function Spaces
1960 Some Results on a Generalization of the Character Table of a Finite Group
1960 Truncated Policies in Dynamic Programming
1960 The Cohomology Ring of a Finite Group
1960 Oscar Zariski On Enriques' surfaces
1959 George Mackey On General Measure Theory
1959 Andrew Gleason A Characterization of Congruence Groups in Geometries of the Euclidean Type
1959 Richard Brauer On the Irreducible Modulat Representations of Finite Classical Groups
1959 Minimal Metric Spaces
1959 Lars Ahlfors Automorphisms and Coverings of Riemann Surfaces Mappings
1959 Richard Brauer On the Characters of Finite Solvable Groups
1959 Lars Ahlfors Inversion and Representation Theory of the Weierstrass Transform
1958 Richard Brauer The Representation of Finite Groups in Algebraic Number Fields
1958 Computable Functionals
1958 Andrew Gleason Closed Subalgebras of a Commutative Algebra over the Real Numbers
1958 Some aspects on Polynomial Approximation
1958 Mixed Cauchy Problems and C-Semigroups
1958 External Metrics for a Class of Varational Problems Related to Extended Length
1958 Richard Brauer A Generalization of a Theorem of Blichfeldt
1958 Andrew Gleason Some Theories on Transformation Groups
1958 Lynn Loomis Derivations in Commutative Semi-Simple Banach Algebras
1958 On the Homology of Local Rings
1958 Lars Ahlfors The Bilinear Relation on Open Riemann Surfaces
1957 Andrew Gleason On Fibre Spaces and Fibre Bundles
1957 Raoul Bott Regular Curves on Riemann Manifolds
1957 On Canonical Conformal Maps of Multiply Connected Regions
1957 Problems in the Distribution of the Prime Numbers
1956 On Generalized Tchebycheff Polynomials
1956 Andrew Gleason A Global Formulation of the Lie Theory of Transformation Group
1956 Generalized Lambert Transforms
1956 New Simple Lie Algebras
1955 The scattering of electromagnetic radiation by a cyclindrical shell of finite length
1955 Lars Ahlfors Variability Regions for the Univalent Functions and their Derivatives
1955 George Mackey Locally M-Convex Algebras
1955 Lars Ahlfors Conformal Invariants and Prime Ends
1955 On the generalization of the notion of H*-algebra: Complemented algebras
1955 Inversion of the Laplace and Stieltjes transforms utilizing difference operators
1955 Lars Ahlfors Contributions to the Problem of Type
1955 George Mackey Some Inequalities Related to Holder Inequality and Some Contributions to Lattice Theory
1955 Induced potentials
1955 Local uniformization of algebraic surfaces over modular ground fields
1954 Properties of a special set of entire functions and their respective partial sums
1954 Richard Brauer On Finite Groups Related to Permutation Groups of Prime Degrees
1954 Lars Ahlfors External Problems on Riemann Surfaces
1954 On approximation and interpolation by functions analytic in a given region and an application to orthonormal systems
1954 Geometric aspects of integration theory
1953 On distortion at the boundary of a conformal mapping
1953 Overconvergent Taylor Series and the zeros of related polynomials
1953 On some special order statistics from a multinomial distribution
1953 Axially symmetric stokes flows
1953 Lynn Loomis Spectral Theory for a Class of Non-Linear Operators
1953 The totality of curves of genus g
1953 L1-structure in Banach Spaces
1952
1952 A topological analysis of differential equations in the large
1952 Methods in the location of zeros of families of polynomials of unbounded degree in circles sectors and other regions
1952 The Fredholm Theory in Banach Spaces
1952 On the theorem of Bertini on the variable singular points of a linear system
1952
1951 On the harmonic analysis of certain groups and semi-groups of operators
1951 Lars Ahlfors Harmonic Functions on Open Riemann Surfaces
1951 On the Cohomology theory of associative algebras
1951 Some estimates for external distance
1951
1951
1950 Iterative methods for solving partial difference equations of elliptic type
1950 On the characterization of Reynolds operators on the normed algebra of all continuous real-valued functions defined on a compact Hausdorff space
1950 Lynn Loomis On the Generalization of the Notion of H*-Algebra: Complex Algebra
1950 Equivalence concepts on an algebraic curve
1950 Lars Ahlfors Transformations in Reproducing
1950
1950
1949 Lars Ahlfors Properties of Conformal Invariants
1949 Lars Ahlfors Representations of Finite Groups
1949 Hausdorff measure in abstract metric spaces
1948 Tensor fields associated with Lipschitz cochains
1948 The extensibility of local Lie groups of transformations and groups on surfaces
1948 Some problems in conformal mapping
1948 Joseph Walsh On the Degree of Approximation to Harmonic Functions by Harmonic Polynomials
1948 Lars Ahlfors Diophantine Aspects of Poincare Theta Functions
1947 Non-commutative integration and abstract differential equations
1947 Homology with operators and mapping theory
1947 The Moduli of univalence and of p-valence of functions analytic in the unit circle
1947 Some studies of functions of exponential type
1947 Axially symmetric harmonic functions
1947 The third iterate of the Laplace transform
1946 Multiplicative Riemann integration in normed rings
1945 On the solutions of ordinary linear homogeneous differential equations of the second order in the complex domain
1945 Topologies for spaces of transformations
1936-1945 MANY ENTRIES ARE MISSING
1936 Generalized derivatives and approximation by polynomials
1936 On the measure of the critical values of functions
1936 The linear homogeneous group modulo p
1935 Topics in the theory of critical points
1935 On interpolation and approximation to an analytic function by rational functions with preassigned poles
1935 Certain invariants of closed extremals
1935 Extensions of partially ordered sets
1935 The minimizing of a singular quadratic functional
1935 I. Interpolation in transforms of the roots of unity II. The Jacobi interpolation series on the lemniscate of convergence
1934 On a class of polynomials which minimize definite integrals
1934 On the planer points of an analytic surface
1934 The index form associated with an extremaloid
1934 A class of completely monotonic functions every positive power of which is also completely monotonic
1934 Topics in the theory of binary forms
1933 Parallelism and equidistance in Riemannian geometry
1933 Invariant methods in the infinitesimal geometry of surfaces
1933 The theory of tables of group characteristics
1932 The coloring of graphs
1932 On the derivations of Newtonian and logarithmic potentials near the acting masses
1932 On the degree of convergence and overconvergence of polynomials of best simultaneous approximation to several functions analytic in distinct regions
1932 On the double pendulum and similar dynamical systems
1932 Sufficient conditions in the problem of the calculus of variations in n-space in parametric form and under general end conditions
1932 Infinite systems if ordinary differential equations with applications to certain second order non-linear partial differential equations of hyperbolic type
1932 On rigid motions in four dimensions with applications to the Laguerre geometry of three dimensions
1932 The boundary values of analytic functions
1931 The Birkhoff fluid theory of electricity
1931 Invariant functions of conservative surface transformations
1931 The approximation of harmonic functions in three dimensions by harmonic polynomials
1930 On the theory of quadratic fields
1930 Contributions to the restricted problem of three bodies
1930 Existence of critical points of harmonic functions of three variables
1930 (a) Geodesics on a two-dimensional Riemannian manifold with periodic coefficients (b) Poincare's rotation number and Morse's type number
1930 (a) on the expansion of harmonic functions in terms of harmonic polynomials (b) On approximation to an arbitrary function of a complex variable by polynomials
1930 Some properties of genesubspaces of a Riemannian space
1930 The problem if the calculations in m-space with end points variable on two manifolds
1930 The cellular division and approximation of regular spreads
1929 On the differential geometry of surfaces in non-Euclidean space
1929 Singular points of second order systems of real differential equations
1929 The problem of n bodies
1929 Exterior motion in the restricted problem of three bodies
1929 Relations between the critical points of a real analytic function of n independent variables
1929 Fourier representations
1928 On the oscillation of harmonic functions
1928 On the location of the roots of the Jacobian of two binary forms and of the derivative of a rational function
1927 On the theory of linear differential equations of infinite order
1926 Ordinary linear homogeneous differential equations of order n and the related expansion problems
1926 Fundamental transformations of surfaces
1926 On rejection to infinity and exterior motion in the restricted Problem of Three Bodies
1926 The group characteristics of the general and special quaternary linear homogeneous groups
1926 Studies on the gyroscope
1925 Boundary value problems of the third order and the allied expansions
1925 Contributions to the theory of Riemann spaces
1924 Some mean-value theorems connected with Cote's Method of Mechanical Quadature
1924 Linear spaces and their fixed points.
1923 (a) on certain differential equations of the second order (b) on certain systems of differential equations containing a parameter
1923 On the theory of discrete varieties
1923 A method of series in elastic with applications (I) to circular plates of constant or variable thickness
1922 Expansion theorems for solution of a Fredholm homogeneous integral equation of the second kind with kernel of non-symmetric type
1922 (a) Developments associated with a boundary problem not linear in the parameter (b) the boundary problems and developments associated with a system of ordinary linear differential equations of the first order
1922 The equilong transformations of Euclidean space
1922 The general theory of the linear partial q-difference equation and of the linear partial difference equation of the intermediate type
1921 The determination of the coefficients in interpolation formulae and a study of the approximate solution of integral equations
1920 On the location of the roots of the Jacobian of two binary forms
1920 (a) Existence Theorems for the General Real Self-Adjoint Linear System of the Second Order (b) Oscillation Theorems for the Real Self-Adjoint Linear System of the Second Order
1919 The Geometry of a Non-Euclidean Line-Sphere Transformation
1919 On the Zeros of Solutions of Linear Differential Equations
1919 (a) On Linear equations with an Infinite Number of Variables (b) on Infinite Systems of Linear Integral Equations (c) Flexural Deflections and Statically Indeterminate Beams
1917 Certain types of geodesic motion on a surface of negative curvature
1917 Linear integro-differential equations with a boundary condition
1917 On rational approximations to an irrational complex number
1917 Curves invariant under point-transformations of special type
1916 Some problems connected with the linear connectivity of manifolds
1916 Systems of pencils of lines in ordinary space
1916 Some integral tests for the convergence and divergence of infinite series
1916 The hypergeometric difference equation
1915 Conformal transformation of curvilinear angles
1915 Problems in the theory of ordinary linear differential equations with auxiliary conditions at more than two points
1915 On the degree of convergence of Birkhoff's series
1914 The calculus of variations as the limit of a problem in minimizing an algebraic sum
1914 Su alcuni caratteri di una serie algebriea e la formola di de Jonquieres per serie qualsiasi
1913 On the functions of a complex variable defined by an ordinary differential equation of the first order and first degree
1913 An analytic treatment of the conic as an element of space of three dimensions
1913 Implicit functions defined by equations with vanishing Jacobian
1913 Oriented circles in space
1912 Problems connected with linear difference equations of the second order with special reference to equations with periodic coefficients
1911 Certain singularities of point-transformations in space of three-dimensions
1910 Vector functions of a point
1910 Volterra's integral equation of the second kind with discontinuous kernel
1909 On some problems in conformal mapping
1909 Certain singularities of transformations of two real variables
1908 On the theory of convergence factors and some of its applications
1908 The invariants of linear differential expressions
1907 A contribution to the theory of trigonometric and zonal harmonic series
1906 Brilliant points
1905 On the problem of analytic extension as applied to the functions defined by power series
1904 An arithmetic treatment of some problems in analysis situs
1903 Binary Families in a triply connected region with especial reference to hypergeometric families
1902 Regular singular points of a system of homogeneous linear differential equations of the first order
1901 On the invariants of quadratic differential forms
1901 The parametric representation of the neighborhood of a singular point of an analytic surface
1898 On linear differential equations of the third and fourth orders in whole solutions exist certain homogeneous relations
1898 The variation of latitude
1897 Systems of revolution and their relation to conical systems in the theory of Lame's products
1897 On the Roots of the hypergeometric and Bessel's Functions
1895 On the Properties of functions defined by the partial equation
1889 Bessel's Functions
1887 Surfaces having the principal radii of curvature at each point numberically equal but of opposite sign
1886 A contribution to the theory of the general equation of the sixth degree
1879 The investigation of the constants of the micrometirc apparatus of the Merz equatorial of the observatory of Harvard College together with the discussion of the observations of the satellites of Mars observed therewith
1873 The heat of the sun

shortest math dissertation

Dissertations and Placements 2010-Present

Emily Dautenhahn Thesis: Heat kernel estimates on glued spaces Advisor: Laurent Saloff-Coste First Position:  Assistant Professor at Murray State University

Elena Hafner Thesis: Combinatorics of Vexillary Grothendieck Polynomials Advisor: Karola Meszaros First Position: NSF Postdoctoral Fellow,, at University of Washington

Sumun Iyer Thesis: Dynamics of non-locally compact topological groups Advisor: Slawomir Solecki First Position: NSF Postdoctoral Fellow at Carnegie Mellon University in Pittsburgh

Sebastian Junge Thesis: Applications of Transferring the Ramsey Property between Categories Advisor: Slawomir Solecki First Position: Lecturer at Texas State University

Nicki Magill Thesis: Infinite Staircases for Hirzebruch Surfaces Advisor: Tara Holm First Position: NSF Postdoctoral Fellow at UC Berkeley

Prairie Wentworth-Nice Thesis: Finite Groups, Polymatroids, and Error-Correcting Codes Advisor: Edward Swartz First Position: Postdoctoral Teaching Fellow at Johns Hopkins University

Fiona Young Thesis: Dissecting an Integer Polymatroid Advisor: Edward Swartz First Position: Pursuing her own start-up in the math education technology space

Kimoi Kemboi Thesis: Full exceptional collections of vector bundles on linear GIT quotients Advisor: Daniel Halpern-Leistner First Position: Postdoc at the Institution for Advanced Study and Princeton

Max Lipton Thesis: Dynamical Systems in Pure Mathematics Advisor: Steven Strogatz First Position: NSF Mathematical Sciences Postdoctoral Fellow at Massachusetts Institute of Technology

Elise McMahon Thesis: A simplicial set approach to computing the group homology of some orthogonal subgroups of the discrete group  Advisor: Inna Zakharevich First Position: Senior Research Scientist at Two Six Technologies

Andrew Melchionna Thesis: Opinion Propagation and Sandpile Models  Advisor: Lionel Levine First Position: Quantitative Researcher at Trexquant

Peter Uttenthal Thesis: Density of Selmer Ranks in Families of Even Galois Representations Advisor: Ravi Kumar Ramakrishna First Position: Visiting Assistant Professor at Cornell University

Liu Yun Thesis: Towers of Borel Fibrations and Generalized Quasi-Invariants Advisor: Yuri Berest First Position: Postdoc at Indiana University Bloomington

Romin Abdolahzadi Thesis: Anabelian model theory Advisor: Anil Nerode First Position: Quantitative Analyst, A.R.T. Advisors, LLC

Hannah Cairns Thesis: Abelian processes, and how they go to sleep Advisor: Lionel Levine First Position: Visiting Assistant Professor, Cornell University

Shiping Cao Thesis: Topics in scaling limits on some Sierpinski carpet type fractals Advisor: Robert Strichartz (Laurent Saloff-Coste in last semester) First Position: Postdoctoral Scholar, University of Washington

Andres Fernandes Herrero Thesis: On the boundedness of the moduli of logarithmic connections Advisor: Nicolas Templier First Position: Ritt Assistant Professor, Columbia University

Max Hallgren Thesis: Ricci Flow with a Lower Bound on Ricci Curvature Advisor: Xiaodong Cao First Position: NSF Postdoctoral Research Fellow, Rutgers University

Gautam Krishnan Thesis: Degenerate series representations for symplectic groups Advisor: Dan Barbasch First Position: Hill Assistant Professor, Rutgers University

Feng Liang Thesis: Mixing time and limit shapes of Abelian networks Advisor: Lionel Levine

David Mehrle Thesis: Commutative and Homological Algebra of Incomplete Tambara Functors Advisor: Inna Zakharevich First Position: Postdoctoral Scholar, University of Kentucky

Itamar Sales de Oliveira Thesis: A new approach to the Fourier extension problem for the paraboloid Advisor: Camil Muscalu First Position: Postdoctoral Researcher, Nantes Université

Brandon Shapiro Thesis: Shape Independent Category Theory Advisor: Inna Zakharevich First Position: Postdoctoral Fellow, Topos Institute

Ayah Almousa Thesis: Combinatorial characterizations of polarizations of powers of the graded maximal ideal Advisor: Irena Peeva First position: RTG Postdoctoral Fellow, University of Minnesota

Jose Bastidas Thesis: Species and hyperplane arrangements Advisor: Marcelo Aguiar First position: Postdoctoral Fellow, Université du Québec à Montréal

Zaoli Chen Thesis: Clustered Behaviors of Extreme Values Advisor: Gennady Samorodnitsky First Position: Postdoctoral Researcher, Department of and Statistics, University of Ottawa

Ivan Geffner Thesis: Implementing Mediators with Cheap Talk Advisor: Joe Halpern First Position: Postdoctoral Researcher, Technion – Israel Institute of Technology

Ryan McDermott Thesis: Phase Transitions and Near-Critical Phenomena in the Abelian Sandpile Model Advisor: Lionel Levine

Aleksandra Niepla Thesis:  Iterated Fractional Integrals and Applications to Fourier Integrals with Rational Symbol Advisor: Camil Muscalu First Position: Visiting Assistant Professor, College of the Holy Cross

Dylan Peifer Thesis: Reinforcement Learning in Buchberger's Algorithm Advisor: Michael Stillman First Position: Quantitative Researcher, Susquehanna International Group

Rakvi Thesis: A Classification of Genus 0 Modular Curves with Rational Points Advisor: David Zywina First Position: Hans Rademacher Instructor, University of Pennsylvania

Ana Smaranda Sandu Thesis: Knowledge of counterfactuals Advisor: Anil Nerode First Position: Instructor in Science Laboratory, Computer Science Department, Wellesley College

Maru Sarazola Thesis: Constructing K-theory spectra from algebraic structures with a class of acyclic objects Advisor: Inna Zakharevich First Position: J.J. Sylvester Assistant Professor, Johns Hopkins University

Abigail Turner Thesis: L2 Minimal Algorithms Advisor: Steven Strogatz

Yuwen Wang Thesis: Long-jump random walks on finite groups Advisor: Laurent Saloff-Coste First Position: Postdoc, University of Innsbruck, Austria

Beihui Yuan Thesis:  Applications of commutative algebra to spline theory and string theory Advisor: Michael Stillman First Position: Research Fellow, Swansea University

Elliot Cartee Thesis: Topics in Optimal Control and Game Theory Advisor: Alexander Vladimirsky First Position: L.E. Dickson Instructor, Department of , University of Chicago

Frederik de Keersmaeker Thesis: Displaceability in Symplectic Geometry Advisor: Tara Holm First Position: Consultant, Addestino Innovation Management

Lila Greco Thesis: Locally Markov Walks and Branching Processes Advisor: Lionel Levine First Position: Actuarial Assistant, Berkshire Hathaway Specialty Insurance

Benjamin Hoffman Thesis: Polytopes And Hamiltonian Geometry: Stacks, Toric Degenerations, And Partial Advisor: Reyer Sjamaar First Position: Teaching Associate, Department of , Cornell University

Daoji Huang Thesis: A Bruhat Atlas on the Wonderful Compactification of PS O(2 n )/ SO (2 n -1) and A Kazhdan-Lusztig Atlas on G/P Advisor: Allen Knutson First Position: Postdoctoral Associate, University of Minnesota

Pak-Hin Li Thesis: A Hopf Algebra from Preprojective Modules Advisor: Allen Knutson First position: Associate, Goldman Sachs

Anwesh Ray Thesis: Lifting Reducible Galois Representations Advisor: Ravi Ramakrishna First Position: Postdoctoral Fellowship, University of British Columbia

Avery St. Dizier Thesis: Combinatorics of Schubert Polynomials Advisor: Karola Meszaros First Position: Postdoctoral Fellowship, Department of , University of Illinois at Urbana-Champaign

Shihao Xiong Thesis: Forcing Axioms For Sigma-Closed Posets And Their Consequences Advisor: Justin Moore First Position: Algorithm Developer, Hudson River Trading

Swee Hong Chan Thesis: Nonhalting abelian networks Advisor: Lionel Levine First Position: Hedrick Adjunct Assistant Professor, UCLA

Joseph Gallagher Thesis: On conjectures related to character varieties of knots and Jones polynomials Advisor: Yuri Berest First Position: Data Scientist, Capital One

Jun Le Goh Thesis: Measuring the Relative Complexity of Mathematical Constructions and Theorems Advisor: Richard Shore First Position: Van Vleck Assistant Professor, University of Wisconsin-Madison

Qi Hou Thesis: Rough Hypoellipticity for Local Weak Solutions to the Heat Equation in Dirichlet Spaces Advisor: Laurent Saloff-Coste First Position: Visiting Assistant Professor, Department of , Cornell University

Jingbo Liu Thesis: Heat kernel estimate of the Schrodinger operator in uniform domains Advisor: Laurent Saloff-Coste First Position: Data Scientist, Jet.com

Ian Pendleton Thesis:  The Fundamental Group, Homology, and Cohomology of Toric Origami 4-Manifolds Advisor: Tara Holm

Amin Saied Thesis: Stable representation theory of categories and applications to families of (bi)modules over symmetric groups Advisor: Martin Kassabov First Position: Data Scientist, Microsoft

Yujia Zhai Thesis:  Study of bi-parameter flag paraproducts and bi-parameter stopping-time algorithms Advisor: Camil Muscalu First Position: Postdoctoral Associate, Université de Nantes 

Tair Akhmejanov Thesis: Growth Diagrams from Polygons in the Affine Grassmannian Advisor: Allen Knutson First position: Arthur J. Krener Assistant Professor, University of California, Davis

James Barnes Thesis:  The Theory of the Hyperarithmetic Degrees Advisor: Richard Shore First position: Visiting Lecturer, Wellesley College

Jeffrey Bergfalk Thesis:  Dimensions of ordinals: set theory, homology theory, and the first omega alephs Advisor: Justin Moore Postdoctoral Associate, UNAM - National Autonomous University of Mexico

TaoRan Chen Thesis: The Inverse Deformation Problem Advisor: Ravi Ramakrishna

Sergio Da Silva Thesis: On the Gorensteinization of Schubert varieties via boundary divisors Advisor: Allen Knutson First position: Pacific Institute for the Mathematical Sciences (PIMS) postdoctoral fellowship, University of Manitoba

Eduard Einstein Thesis:  Hierarchies for relatively hyperbolic compact special cube complexes Advisor: Jason Manning First position: Research Assistant Professor (Postdoc), University of Illinois, Chicago (UIC)

Balázs Elek Thesis:  Toric surfaces with Kazhdan-Lusztig atlases Advisor: Allen Knutson First position: Postdoctoral Fellow, University of Toronto

Kelsey Houston-Edwards Thesis:  Discrete Heat Kernel Estimates in Inner Uniform Domains Advisor: Laurent Saloff-Coste First position: Professor of Math and Science Communication, Olin College

My Huynh Thesis:  The Gromov Width of Symplectic Cuts of Symplectic Manifolds. Advisor: Tara Holm First position: Applied Mathematician, Applied Research Associates Inc., Raleigh NC.

Hossein Lamei Ramandi Thesis: On the minimality of non-σ-scattered orders Advisor: Justin Moore First position:  Postdoctoral Associate at UFT (University Toronto)

Christine McMeekin Thesis: A density of ramified primes Advisor: Ravi Ramakrishna First position: Researcher at Max Planck Institute

Aliaksandr Patotski Thesis:  Derived characters of Lie representations and Chern-Simons forms Advisor: Yuri Berest First position: Data Scientist, Microsoft

Ahmad Rafiqi Thesis:  On dilatations of surface automorphisms Advisor: John Hubbard First position: Postdoctoral Associate, Sao Palo, Brazil

Ying-Ying Tran Thesis:  Computably enumerable boolean algebras Advisor: Anil Nerode First position: Quantitative Researcher

Drew Zemke Thesis:  Surfaces in Three- and Four-Dimensional Topology Advisor: Jason Manning First position: Preceptor in , Harvard University

Heung Shan Theodore Hui Thesis: A Radical Characterization of Abelian Varieties  Advisor: David Zywina First position: Quantitative Researcher, Eastmore Group

Daniel Miller Thesis: Counterexamples related to the Sato–Tate conjecture Advisor: Ravi Ramakrishna First position: Data Scientist, Microsoft

Lihai Qian Thesis: Rigidity on Einstein manifolds and shrinking Ricci solitons in high dimensions Advisor: Xiaodong Cao First position: Quantitative Associate, Wells Fargo

Valente Ramirez Garcia Luna Thesis: Quadratic vector fields on the complex plane: rigidity and analytic invariants Advisor: Yulij Ilyashenko First position: Lebesgue Post-doc Fellow, Institut de Recherche Mathématique de Rennes

Iian Smythe Thesis: Set theory in infinite-dimensional vector spaces Advisor: Justin Moore First position: Hill Assistant Professor at Rutgers, the State University of New Jersey

Zhexiu Tu Thesis: Topological representations of matroids and the cd-index Advisor: Edward Swartz First position: Visiting Professor - Centre College, Kentucky

Wai-kit Yeung Thesis: Representation homology and knot contact homology Advisor: Yuri Berest First position: Zorn postdoctoral fellow, Indiana University

Lucien Clavier Thesis: Non-affine horocycle orbit closures on strata of translation surfaces: new examples Advisor: John Smillie First position: Consultant in Capital Markets, Financial Risk at Deloitte Luxembourg

Voula Collins Thesis: Crystal branching for non-Levi subgroups and a puzzle formula for the equivariant cohomology of the cotangent bundle on projective space Advisor: Allen Knutson FIrst position: Postdoctoral Associate, University of Connecticut

Pok Wai Fong Thesis: Smoothness Properties of symbols, Calderón Commutators and Generalizations Advisor: Camil Muscalu First position: Quantitative researcher, Two Sigma

Tom Kern Thesis: Nonstandard models of the weak second order theory of one successor Advisor: Anil Nerode First position: Visiting Assistant Professor, Cornell University

Robert Kesler Thesis: Unbounded multilinear multipliers adapted to large subspaces and estimates for degenerate simplex operators Advisor: Camil Muscalu First position: Postdoctoral Associate, Georgia Institute of Technology

Yao Liu Thesis: Riesz Distributions Assiciated to Dunkl Operators Advisor: Yuri Berest First position: Visiting Assistant Professor, Cornell University

Scott Messick Thesis: Continuous atomata, compactness, and Young measures Advisor: Anil Nerode First position: Start-up

Aaron Palmer Thesis: Incompressibility and Global Injectivity in Second-Gradient Non-Linear Elasticity Advisor: Timothy J. Healey First position: Postdoctoral fellow, University of British Columbia 

Kristen Pueschel Thesis: On Residual Properties of Groups and Dehn Functions for Mapping Tori of Right Angled Artin Groups Advisor: Timothy Riley First position: Postdoctoral Associate, University of Arkansas

Chenxi Wu Thesis: Translation surfaces: saddle connections, triangles, and covering constructions. Advisor: John Smillie First position: Postdoctoral Associate, Max Planck Institute of

David Belanger Thesis: Sets, Models, And Proofs: Topics In The Theory Of Recursive Functions Advisor: Richard A. Shore First position: Research Fellow, National University of Singapore

Cristina Benea Thesis: Vector-Valued Extensions for Singular Bilinear Operators and Applications Advisor: Camil Muscalu First position: University of Nantes, France

Kai Fong Ernest Chong Thesis: Face Vectors and Hilbert Functions Advisor: Edward Swartz First position: Research Scientist, Agency for Science, Technology and Research, Singapore

Laura Escobar Vega Thesis: Brick Varieties and Toric Matrix Schubert Varieties Advisor: Allen Knutson First position: J. L. Doob Research Assistant Professor at UIUC

Joeun Jung Thesis: Iterated trilinear fourier integrals with arbitrary symbols Advisor: Camil Muscalu First position: Researcher, PARC (PDE and Functional Analysis Research Center) of Seoul National University

Yasemin Kara Thesis: The laplacian on hyperbolic Riemann surfaces and Maass forms Advisor: John H. Hubbard Part Time Instructor, Faculty of Engineering and Natural Sciences, Bahcesehir University

Chor Hang Lam Thesis: Homological Stability Of Diffeomorphism Groups Of 3-Manifolds Advisor: Allen Hatcher

Yash Lodha Thesis: Finiteness Properties And Piecewise Projective Homeomorphisms Advisor: Justin Moore and Timothy Riley First position: Postdoctoral fellow at Ecole Polytechnique Federale de Lausanne in Switzerland

Radoslav Zlatev Thesis: Examples of Implicitization of Hypersurfaces through Syzygies Advisor: Michael E. Stillman First position: Associate, Credit Strats, Goldman Sachs

Margarita Amchislavska Thesis: The geometry of generalized Lamplighter groups Advisor: Timothy Riley First position: Department of Defense

Hyungryul Baik Thesis: Laminations on the circle and hyperbolic geometry Advisor: John H. Hubbard First position: Postdoctoral Associate, Bonn University

Adam Bjorndahl Thesis: Language-based games Advisor: Anil Nerode and Joseph Halpern First position: Tenure Track Professor, Carnegie Mellon University Department of Philosophy

Youssef El Fassy Fihry Thesis: Graded Cherednik Algebra And Quasi-Invariant Differential Forms Advisor: Yuri Berest First position: Software Developer, Microsoft

Chikwong Fok Thesis: The Real K-theory of compact Lie groups Advisor: Reyer Sjamaar First position: Postdoctoral fellow in the National Center for Theoretical Sciences, Taiwan

Kathryn Lindsey Thesis: Families Of Dynamical Systems Associated To Translation Surfaces Advisor: John Smillie First position: Postdoctoral Associate, University of Chicago

Andrew Marshall Thesis: On configuration spaces of graphs Advisor: Allan Hatcher First position: Visiting Assistant Professor, Cornell University

Robyn Miller Thesis: Symbolic Dynamics Of Billiard Flow In Isosceles Triangles Advisor: John Smillie First position: Postdoctoral Researcher at Mind Research Network

Diana Ojeda Aristizabal Thesis: Ramsey theory and the geometry of Banach spaces Advisor: Justin Moore First position: Postdoctoral Fellow, University of Toronto

Hung Tran Thesis: Aspects of the Ricci flow Advisor: Xiaodong Cao First position: Visiting Assistant Professor, University of California at Irvine

Baris Ugurcan Thesis: LPLP-Estimates And Polyharmonic Boundary Value Problems On The Sierpinski Gasket And Gaussian Free Fields On High Dimensional Sierpinski Carpet Graphs Advisor: Robert S. Strichartz First position: Postdoctoral Fellowship, University of Western Ontario

Anna Bertiger Thesis: The Combinatorics and Geometry of the Orbits of the Symplectic Group on Flags in Complex Affine Space Advisor: Allen Knutson First position: University of Waterloo, Postdoctoral Fellow

Mariya Bessonov Thesis: Probabilistic Models for Population Dynamics Advisor: Richard Durrett First position: CUNY City Tech, Assistant Professor, Tenure Track

Igors Gorbovickis Thesis: Some Problems from Complex Dynamical Systems and Combinatorial Geometry Advisor: Yulij Ilyashenko First position: Postdoctoral Fellow, University of Toronto

Marisa Hughes Thesis: Quotients of Spheres by Linear Actions of Abelian Groups Advisor: Edward Swartz First position: Visiting Professor, Hamilton College

Kristine Jones Thesis: Generic Initial Ideals of Locally Cohen-Macaulay Space Curves Advisor: Michael E. Stillman First position: Software Developer, Microsoft

Shisen Luo Thesis: Hard Lefschetz Property of Hamiltonian GKM Manifolds Advisor: Tara Holm First position: Associate, Goldman Sachs

Peter Luthy Thesis: Bi-parameter Maximal Multilinear Operators Advisor: Camil Muscalu First position: Chauvenet Postdoctoral Lecturer at Washington University in St. Louis 

Remus Radu Thesis: Topological models for hyperbolic and semi-parabolic complex Hénon maps Advisor: John H. Hubbard First position: Milnor Lecturer, Institute for Mathematical Sciences, Stony Brook University

Jenna Rajchgot Thesis: Compatibly Split Subvarieties of the Hilbert Scheme of Points in the Plane Advisor: Allen Knutson First position: Research member at the Mathematical Sciences Research Institute (fall 2012); postdoc at the University of Michigan

Raluca Tanase Thesis: Hénon maps, discrete groups and continuity of Julia sets Advisor: John H. Hubbard First position: Milnor Lecturer, Institute for Mathematical Sciences, Stony Brook University

Ka Yue Wong Thesis: Dixmier Algebras on Complex Classical Nilpotent Orbits and their Representation Theories Advisor: Dan M. Barbasch First position: Postdoctoral fellow at Hong Kong University of Science and Technology

Tianyi Zheng Thesis: Random walks on some classes of solvable groups Advisor: Laurent Saloff-Coste First position: Postdoctoral Associate, Stanford University

Juan Alonso Thesis: Graphs of Free Groups and their Measure Equivalence Advisor: Karen Vogtmann First position: Postdoc at Uruguay University

Jason Anema Thesis: Counting Spanning Trees on Fractal Graphs Advisor: Robert S. Strichartz First position: Visiting assistant professor of mathematics at Cornell University

Saúl Blanco Rodríguez Thesis: Shortest Path Poset of Bruhat Intervals and the Completecd-Index Advisor: Louis Billera First position: Visiting assistant professor of mathematics at DePaul University

Fatima Mahmood Thesis: Jacobi Structures and Differential Forms on Contact Quotients Advisor: Reyer Sjamaar First position: Visiting assistant professor at University of Rochester

Philipp Meerkamp Thesis: Singular Hopf Bifurcation Advisor: John M. Guckenheimer First position: Financial software engineer at Bloomberg LP

Milena Pabiniak Thesis: Hamiltonian Torus Actions in Equivariant Cohomology and Symplectic Topology Advisor: Tara Holm First position: Postdoctoral associate at the University of Toronto

Peter Samuelson Thesis: Kauffman Bracket Skein Modules and the Quantum Torus Advisor: Yuri Berest First position: Postdoctoral associate at the University of Toronto

Mihai Bailesteanu  Thesis: The Heat Equation under the Ricci Flow Advisor: Xiaodong Cao First position: Visiting assistant professor at the University of Rochester

Owen Baker  Thesis:  The Jacobian Map on Outer Space Advisor: Karen Vogtmann First position: Postdoctoral fellow at McMaster University

Jennifer Biermann  Thesis: Free Resolutions of Monomial Ideals Advisor: Irena Peeva First position: Postdoctoral fellow at Lakehead University

Mingzhong Cai  Thesis: Elements of Classical Recursion Theory: Degree-Theoretic Properties and Combinatorial Properties Advisor: Richard A. Shore First position: Van Vleck visiting assistant professor at the University of Wisconsin at Madison

Ri-Xiang Chen  Thesis: Hilbert Functions and Free Resolutions Advisor: Irena Peeva First position: Instructor at Shantou University in Guangdong, China

Denise Dawson  Thesis: Complete Reducibility in Euclidean Twin Buildings Advisor: Kenneth S. Brown First position: Assistant professor of mathematics at Charleston Southern University

George Khachatryan Thesis: Derived Representation Schemes and Non-commutative Geometry Advisor: Yuri Berest First position: Reasoning Mind

Samuel Kolins  Thesis: Face Vectors of Subdivision of Balls Advisor: Edward Swartz First position: Assistant professor at Lebanon Valley College

Victor Kostyuk Thesis: Outer Space for Two-Dimensional RAAGs and Fixed Point Sets of Finite Subgroups Advisor: Karen Vogtmann First position: Knowledge engineering at Reasoning Mind

Ho Hon Leung  Thesis: K-Theory of Weight Varieties and Divided Difference Operators in Equivariant KK-Theory Advisor: Reyer Sjamaar First position: Assistant professor at the Canadian University of Dubai

Benjamin Lundell  Thesis: Selmer Groups and Ranks of Hecke Rings Advisor: Ravi Ramakrishna First position: Acting assistant professor at the University of Washington

Eyvindur Ari Palsson  Thesis: Lp Estimates for a Singular Integral Operator Motivated by Calderón’s Second Commutator Advisor: Camil Muscalu First position: Visiting assistant professor at the University of Rochester

Paul Shafer  Thesis: On the Complexity of Mathematical Problems: Medvedev Degrees and Reverse Advisor: Richard A. Shore First position: Lecturer at Appalachian State University

Michelle Snider  Thesis: Affine Patches on Positroid Varieties and Affine Pipe Dreams Advisor: Allen Knutson First position: Government consulting job in Maryland

Santi Tasena Thesis: Heat Kernel Analysis on Weighted Dirichlet Spaces Advisor: Laurent Saloff-Coste First position: Lecturer professor at Chiang Mai University, Thailand

Russ Thompson  Thesis: Random Walks and Subgroup Geometry Advisor: Laurent Saloff-Coste First position: Postdoctoral fellow at the Mathematical Sciences Research Institute

Gwyneth Whieldon Thesis: Betti Numbers of Stanley-Reisner Ideals Advisor: Michael E. Stillman First position: Assistant professor of mathematics at Hood College

Andrew Cameron Thesis: Estimates for Solutions of Elliptic Partial Differential Equations with Explicit Constants and Aspects of the Finite Element Method for Second-Order Equations Advisor: Alfred H. Schatz First position: Adjunct instructor of mathematics at Tompkins Cortland Community College

Timothy Goldberg Thesis: Hamiltonian Actions in Integral Kähler and Generalized Complex Geometry Advisor: Reyer Sjamaar First position: Visiting assistant professor of mathematics at Lenoir-Rhyne University

Gregory Muller Thesis: The Projective Geometry of Differential Operators Advisor: Yuri Berest First position: Assistant professor at Louisiana State University 

Matthew Noonan Thesis: Geometric Backlund transofrmation in homogeneous spaces Advisor: John H. Hubbard

Sergio Pulido Niño Thesis: Financial Markets with Short Sales Prohibition Advisor: Philip E. Protter First position: Postdoctoral associate in applied probability and finance at Carnegie Mellon University

Academia Insider

How long is a PhD dissertation? [Data by field]

The final piece of the PhD journey is the PhD dissertation. It takes many years to accumulate enough original and new data to fill out a dissertation to the satisfaction of experts in your field. Interestingly, the PhD dissertation length and content vary significantly based on the field you are studying and the publishing conventions.

A PhD can be anywhere from 50 pages to over 450 pages long. This equates to between about 20,000 words to 100,000 words. Most PhD theses are between 60,000 and 80,000 words long excluding contents, citations and references.

A PhD thesis contains different sections including an introduction, methods, results and discussion, conclusions, further work, and references. Each one of these different sections will vary in length depending on the field of study and your particular topic.

Ultimately, a PhD dissertation should contain as many pages and words as it takes to communicate the results of your multi-year investigation.

It is very rewarding to see your thesis come together as you are writing day after day. When I was writing my PhD dissertation I wrote the sections separately and my heart filled with joy when I finally put them all together and compile them into a single PDF document.

Counting the pages should not be the way to determine a PhD dissertation’s value but it certainly helps when your thesis is starting to look substantial in thickness.

How many pages should a PhD dissertation be?

A PhD dissertation should contain as many pages and words as it takes to outline the current state of your field and provide adequate background information, present your results, and provide confidence in your conclusions. A PhD dissertation will also contain figures, graphs, schematics, and other large pictorial items that can easily inflate the page count.

Here is a boxplot summary of many different fields of study and the number of pages of a typical PhD dissertation in the field. It has been created by Marcus Beck from all of the dissertations at the University of Minnesota.

shortest math dissertation

Typically, the mathematical sciences, economics, and biostatistics theses and dissertations tend to be shorter because they rely on mathematical formulas to provide proof of their results rather than diagrams and long explanations.

On the other end of the scale, English, communication studies, political science, history and anthropology are often the largest theses in terms of pages and word count because of the number of words it takes to provide proof and depth of their results.

At the end of the day, it is important that your thesis gets signed off by your review committee and other experts in the field. Your supervisor will be the main judge of whether or not your dissertation is capable of satisfying the requirements of a PhD in your field.

If you want to know more about how long a Masters’s thesis and PhD dissertation is you can check out my other articles:

  • How Long is a Masters Thesis? [Your writing guide]
  • How long is a Thesis or dissertation? [the data]

Can a PhD dissertation be too long?

A PhD thesis should contain enough evidence and discussion to report on the most significant findings of your PhD research.

A PhD dissertation should not contain everything that you have done during your PhD. It should only include the data and information required to convince your PhD examining body that wraps up and tells the full story of particular lines of investigation.

Including random results, thoughts, or superfluous explanation can result in a dissertation that is unfocused. I have heard of music PhD is being described as too verbose and physical sciences PhD dissertations as being unfocused.

Therefore, a PhD thesis can be too long if the information it contains does not form a full and cohesive story.

One of my colleagues during their PhD removed an entire chapter from the thesis after writing it as the supervisor said that it needed more experiments to be a full story. They did not want to spend the next six months gathering the data and simply removed the chapter altogether.

How short can PhD dissertation be?

The shortest PhD dissertations are typically found in mathematics.

George Bernard Danzig was an American mathematical scientist who made contributions to industrial engineering and many other mathematical-related fields. An interesting miscommunication led to 1 of the shortest PhD theses ever.

In 1939 his professor wrote two problems on the blackboard and Danzig thought they were homeless assignments. He stated that they were harder than usual but handed in solutions to the surprise of the professor.

They were, in fact, open mathematical problems in statistics.

His professor said to bind the solution to the two problems together and submit them as his thesis – the total thesis length = 14 pages.

Obviously, most PhD theses and dissertations will be so much longer than that!

My PhD dissertation was 256 pages long. It was full of schematics, diagrams, and tables to demonstrate and communicate my findings.

I would say that most people’s PhD thesis experience will be closer to mine than Prof George Bernard Danzig’s.

Why PhD dissertations are typically so long

PhD dissertations are often over 200 pages long.

One of the primary reasons they are so long is that it is a single document that summarises many years of hard work. Also, summarising the research field to date and making sure that all of your references and citations are included so you avoid plagiarism will bolster the word count of the thesis dramatically.

Here are all of the reasons PhD dissertations tend to be so long.

Many years of work

PhD theses or dissertations contain many years of research and analysis.

In many of my YouTube videos I recommend that a PhD student work towards their PhD thesis by doing at least three hours of focused work every work day.

This amount of work quickly adds up.

Of course, not every bit of work makes it into the PhD dissertation but a lot of it does. It can be difficult to work out what to include or leave out of your thesis.

As a PhD student, I perfected the art of turning one experiment into many different types of grafts and schematics to fully explore the limits of my data. The graphs can take up a lot of space in your PhD thesis and, therefore, bolster the page count significantly.

In depth literature review

One of the most substantial parts of a PhD dissertation is the literature review.

The literature review can take up a huge portion of the early part of your PhD dissertation depending on the amount of data and publications in your field.

Writing an in-depth literature review requires just as much meticulous data analysis and searching as the central part of your dissertation.

Figures and schematics

Some fields end up producing a lot of figures and schematics.

My thesis had many full-page figures of atomic force microscopy experiments with much more explanation on subsequent pages.

shortest math dissertation

As they say, a picture paints a thousand words and a dissertation can really benefit from having many schematics to highlight the important aspects of your findings.

References and citations

The recommended PhD dissertation word count from an institution or university does not include citations, references, or other thesis parts such as summary of abbreviations, table of figures, et cetera.

However, these components of your dissertation can take up many pages and add to the overall thickness of your PhD dissertation.

University formatting rules

University formatting rules will also dictate how you many pages your words take up.

I often get roasted on my YouTube channel for having doublespaced lines and wide margins. Unfortunately, this layout was dictated by my university before printing.

PhD dissertations often end up going into long-term storage and therefore, need to adhere to archival and standardised formatting rules.

Deep in the depths of the University of Newcastle, there is a copy of my thesis on a shelf. The formatting and binding rules mean that my thesis looks like everyone else’s.

Universities will often have their own requirements for PhD dissertation cover colour, quality, and type of paper. Even the quality of the paper can change the thickness of the PhD dissertation significantly.

PhD by publication

It is becoming increasingly common to submit a number of peer-reviewed papers bound together with supplementary information in between instead of a PhD dissertation.

The benefits of this to the researcher and university are:

  • More early career peer-reviewed journals for career advancement
  • an easier review process – they have already been peer-reviewed
  • an early focus on publishing means better research outcomes for the researcher, supervisor, and Department.
  • No mad rush at the end to finish a thesis
  • continually writing peer-reviewed papers throughout your PhD helps with timely analysis and communication of results

Even though this option has been available to PhD students for a number of years, I have only known a handful of students actually submit their PhD via publication.

Nonetheless, having this option will suit some research fields better than others and lead to a more productive PhD.

Wrapping up

This article has been through everything you need to know about the length of a PhD dissertation and the common lengths of PhD dissertations for various fields.

Ultimately, there is no predefined length of a PhD .

A PhD thesis is as long as it needs to be to convince your examiners that you have contributed significantly enough to an academic field to be awarded the title of Dr of philosophy.

Mathematical and analytical theses tend to be shorter and can be as short as 50 pages (with one of the shortest being only 14 pages long). At the other end of the spectrum, PhD students in anthropology and history tend to produce the longest dissertations.

shortest math dissertation

Dr Andrew Stapleton has a Masters and PhD in Chemistry from the UK and Australia. He has many years of research experience and has worked as a Postdoctoral Fellow and Associate at a number of Universities. Although having secured funding for his own research, he left academia to help others with his YouTube channel all about the inner workings of academia and how to make it work for you.

Thank you for visiting Academia Insider.

We are here to help you navigate Academia as painlessly as possible. We are supported by our readers and by visiting you are helping us earn a small amount through ads and affiliate revenue - Thank you!

shortest math dissertation

2024 © Academia Insider

shortest math dissertation

Ohio State navigation bar

  • BuckeyeLink
  • Search Ohio State

PhD Dissertations

In 1909 the department awarded its first PhD to  Grace M. Bareis , whose dissertation was directed by Professor Harry W. Kuhn. The department began awarding PhD degrees on a regular basis around 1930, when a formal doctoral program was established as a result of the appointment of Tibor Radó as a professor at our department. To date, the department has awarded over 800 PhD degrees. An average of approximately 15 dissertations per year have been added in recent times. Find below a list of PhD theses completed in our program since 1952. (Additionally, search Ohio State at  Math Genealogy , which also includes some theses from other OSU departments.)

  
2024Castillo, NicholasOn Rational Approximations, Resurgence and Riemann-Hilbert ProblemsOvidiu Costin
2024Christopherson, AdamWeak-type Regularity of the Bergman Projection on Non-smooth DomainsKenneth Koenig
2024Clause, NathanielNew Invariants and Algorithms for Persistence over PosetsFacundo Mémoli
2024Goldman, KatherineShephard GroupsJingyin Huang
2024Genlik, DenizHolomorphic Anomaly Equations For [C /Z ]Hsian-Hua Tseng
2024Gülen, Aziz BurakAlgebraic-Combinatorial Perspectives on Persistence: Functorial Constructions via Möbius Inversion and Galois ConnectionsFacundo Mémoli
2024Lee, JonghooBrauer Group of Split Toric Variety and Split Toric SchemeRoy Joshua
2024Lee, RayReaction-Diffusion Equations in Spatial EcologyKing-Yeung Lam
2024Newton, ScottRepresentability For Monoid ExtensionsSanjeevi Krishnan
2024Packer, DanielSymmetrical Machine LearningDustin Mixon
2024Sehgal, KritiDynamics of the Hénon–Heiles System and Generalizing the Sokhotski-Plemelj FormulaOvidiu Costin
2024Terek Couto, IvoThe Geometry and Structure of Compact Rank-one ECS ManifoldsAndrzej Derdzinski
2024Xing, HaoNumber Theoretical and Dynamical Properties of Euclidean Lattices and Their SublatticesNimish Shah
2023Charparro Sumalave, Gustavo Lafont, Jean-Francois
2023Chen, Chen Anderson, David
2023Gomez Flores, Mario Memoli, Facundo
2023Pan, Amanda Nguyen, Hoi
2023Super, Shidhesh Tseng, Hsian-Hua
2023Zhou, Ling Memoli, Facundo
2023Carr, Matthew Harper, John
2023Casey, Ian Anderson, David
2023Chen, Quan Penneys, David
2023Mishra, Bhawesh Bergelson, Vitaly
2023Liu, Baian Loper, Alan
2023Su, Wei Hung Xiu, Dongbin
2023Wei, Zhining Luo, Wenzhi
2023Werf Vander, Andrew Kahle, Matthew [Paquette, Elliot]
2023Zhang, Danyu Gogolyev, Andriy
2022Ababneh, Ayat Kahle, Matthew
2022Ackelsberg, Ethan Bergelson, Vitaly
2022Andrejek, Luke Best, Janet
2022Call, Benjamin Thompson, Daniel
2022Campolongo, Elizabeth Taylor, Krystal
2022Chen, Yuhang Tseng, Hsian-Hua
2022Dell, Zachary Penneys, David
2022Duncan, Paul Kahle, Matthew
2022Farhangi, Sohail Bergelson, Vitaly
2022Huston, Peter Penneys, David
2022Jeon, Minyoung Anderson, David
2022Kim, Jimin Kahle, Matthew
2022Leung, Wing Hong Holowinsky, Roman
2022Martínez Figueroa, Francisco Kahle, Matthew
2022Mejia Cordero, Julian Holowinsky, Roman
2022Oh, Josiah Lafont, Jean-Francois
2022Sun, Jiawei Xing, Yulong
2022Tsang, Ling Hei Katz, Eric
2022Vargas Bernal, Esteban Tien, Joseph
2022Wang, Qingsong Memoli, Facundo
2022Wang, Yuda Koenig, Kenneth
2022Yan, Pan Cogdell, James
2022Yang, Ruize Xing, Yulong
2022Zhang, Yilong Clemens, Herbert
2022Zhou, Zixu Dongbin, Xiu
2021Bainbridge, Gabriel Krishnan, Sanjeevi
2021Bello, Jason Sivakoff, David
2021Best, Andrew Bergelson, Vitaly
2021Bruno, Nick Loper, Kenneth Alan
2021Chen, Zhen Xiu, Dongbin
2021Clark, Duncan Harper, John
2021Clum, Charles Mixon, Dustin
2021Ferre Moragues, Andreu Bergelson, Vitaly
2021Harper, Matthew Kerler, Thomas
2021Hernandez Palomares, Roberto Penneys, David
2021Lim, Sunhyuk Memoli, Facundo
2021Patel, Dhir Hiary, Ghaith
2021Schonsheck, Nikolas Harper, John
2021Shah, Aniket Anderson, David
2021Wan, Zhengchao Memoli, Facundo
2021Wang, Tianyu Thompson, Daniel
2021Xie, Yuancheng Kodama, Yuji
2021Zelada Cifuentes, Jose Rigoberto Enrique     Bergelson, Vitaly
2021Zhang, Han Shah, Nimish
2020Antoniou, Austin Loper, Kenneth Alan
2020Beckwith, Alexander Luo, Wenzhi
2020Castillo, Andrew Koenig, Kenneth
2020DeBoer, Neil Carlson, Timothy
2020Horst, Michael Johnson, Niles
2020Kim, Woojin Memoli, Facundo
2020Mernik, Luka McNeal, Jeffery
2020Ohl, Trent   Miller, Christopher
2020Osborne, Matthew Tien, Joseph
2020Singhal, Kritika Memoli, Facundo
2020Wang, Jun Tseng, Hsian-Hua
2020Zhang, Runlin Shah, Nimish
2020Zhang, Yu Harper, John
2019Aggarwal, Keshav Holowinsky, Roman
2019Carnovale, Marc Bergelson, Vitaly
2019Chowdhury, Samir Memoli, Facundo
2019Guo, Sheng Guan, Bo
2019Khalil, Osama Shah, Nimish
2019Meehan, Sean Nguyen, Hoi
2019Okutan, Osman Memoli, Facundo
2019Ritchey, Katherine Kahle, Matthew
2019Xiong, Jue McNeal, Jeffery 
2019Xu, Chao Moscovici, Henri
2019Yang, Pengyu Shah, Nimish
2019Ye, Rongqing Cogdell, James
2018Belfanti, Edward Michael Cogdell, James
2018Blomquist, Jacobson Harper, John
2018Glogic, Irfan Costin, Ovidiu
2018Jo, Yeongseong Cogdell, James
2018Kennedy, Christopher Lafont, Jean-Francois
2018Khan, Gabriel Zheng, Fangyang
2018Lin, Yongxiao Holowinsky, Roman
2018McGregor, Daniel Loper, Kenneth Alan
2018Moore, Daniel Cogdell, James
2018Nash, Evan Kennedy, Gary
2018Newman, John Andrew Kahle, Matthew
2018Nowland, Kevin Holowinsky, Roman
2018Renardy, Marissa  Chou, Ching-Shan
2018Richter, Florian Bergelson, Vitaly
2018Sathaye, Bakul Lafont, Jean-Francois
2018Shin, Yeonjong Xiu, Dongbin
2018Staten, Corey  Johnson, Niles
2018Wang, Yilong Kerler, Thomas
2018Yang, Xige Xue, Chuan
2018Yu, Han Baek Sivakoff, David
2017Adali, Ali Tanveer, Saleh
2017Borland, Alexander Kerler, Thomas
2017Dinitz, Thomas Best, Janet
2017Glasscock, Daniel Bergelson, Vitaly
2017Kim, Tae Eun Tanveer, Saleh
2017Liu, Shenhui Luo, Wenzhi
2017Parsons, Kyle Kahle, Matthew
2017Senay Aras, Betul Chou, Ching-Shan
2017Steward, Michael Loper, Kenneth Alan
2017Swang, Theodore Best, Janet
2017Wang, Yanli Chou, Ching-Shan
2017Xia, Bingyu Anderson, David
2017You, Fenglong Tseng, Hsian-Hua
2016Buenger, Carl  Shah, Nimish
2016Cervantes, José Moscovici, Henri
2016Edholm, Luke  McNeal, Jeffery
2016Le, Giang  Davis, Michael
2016Malen, Greg  Kahle, Matthew
2016Moreira, Joel  Bergelson, Vitaly
2016Schmidt, Benjamin Anderson, David
2016Sun, Weizhou Chou, Ching-Shan
2016Wang, Shi Lafont, Jean-Francois
2016Zhang, Qing Cogdell, James
2016Zheng, Cheng Shah, Nimish
2015Baker, Charles Costin, Ovidiu
2015Gubkin, Steven  McNeal, Jeffery
2015Hsu, Ting-Hao Keyfitz, Barbara
2015Huang, Jihui Kodama, Yuji
2015Lam, Wing Chung Luo, Wenzhi
2015Liu, Yang Moscovici, Henri
2015Nasca, Angelo  Bergelson, Vitaly
2015Noble, Laine Lou, Yuan
2015Qi, Zhi Holowinsky, Roman
2015Robertson, Donald  Bergelson, Vitaly
2015Sui, Zhenan Guan, Bo
2015Talamo, James Gerlach, Ulrich
2015Wang, Xiaohui Golubitsky, Martin
2015Xia, Xiaoyue Costin, Ovidiu
2015Yang, Tao Moscovici, Henri
2015Ying, Hao Keyfitz, Barbara
2015Zhang, Qing Luo, Wenzhi
2014Bosna, Bora Carlson, Timothy
2014Christopherson, John Bergelson, Vitaly
2014Edgren, Neal  McNeal, Jeffery
2014Fotis, Samuel  Luo, Wenzhi
2014Jia, Yuhan Kodama, Yuji 
2014Kim, Jung Eun Best, Janet
2014Miller, Jason  Kennedy, Gary
2014Nicol, Andrew Lafont, Jean-Francois
2014Olmez, Faith Best, Janet
2014Ozcakir, Ozge Tanveer, Saleh
2014Park, Hyejin Costin, Ovidiu
2014Poole, Daniel Pittel, Boris
2014Ravindran, Hari  Luo, Wenzhi
2014Wang, Kun Lafont, Jean-Francois
2014Yang, Lei Shah, Nimish
2014Ye, Zhilin Holowinsky, Roman
2013Acan, Huseyin Pittel, Boris
2013Alexander, Samuel Carlson, Timothy
2013All, Timothy  Sinnott, Warren
2013Banerjee, Sayanti  Best, Janet
2013Chen, Weitao Chou, Ching-Shan
2013Estill, Charles Chmutov, Sergei
2013George, Jennifer  Kerler, Thomas 
2013Gibbins, Aliska Davis, Michael W.
2013Greene, Ryan Davis, Michael W.
2013Joecken, Kyle Lafont, Jean-Francois
2013Kowalick, Ryan Lafont, Jean-Francois
2013Perkins, Rudolph  Goss, David
2013Peterson, Nicholas  Pittel, Boris
2013Rosenblatt, Heather Tanveer, Saleh
2013Son, Younghwan Bergelson, Vitaly
2013Sun, Wei Guan, Bo
2013Teh, Wen Chean Carlson, Timothy
2013Tychonievich, Michael Chris Miller
2013Vutha, Amit Golubitsky, Martin
2013Waller, Bradley Sinnott, Warren
2013Wiser, Justin Golubitsky, Martin
2013Yu, Xun Clemens, Herbert
2012Averill, Isabel  Lou, Yuan
2012Chai, Jingsong Cogdell, James
2012Du, Dong Burghelea, Dan
2012Easwaran, Hiranmoy Bergelson, Vitaly
2012Gard, Andrew  Fangyang Zheng
2012Kim, Raeyong Lafont, Jean-Francois
2012Lynd, Justin Solomon, Ron
2012Sequin, Matthew Kerler, Thomas 
2012Sivaraman, Vaidyanathan Robertson, G. Neil
2012Ustian, Alex  Shah, Nimish
2011Adduci, James Mityagin, Boris
2011Danisman, Yusuf Cogdell, James
2011Hui, Wing San Zheng, Fangyang
2011Ji, Haixia Feinberg, Martin
2011McDougal, Robert  Terman, David
2011Munther, Daniel  Lou, Yuan
2011Polo, Fabrizio Bergelson, Vitaly
2011Ravisankar, Sivaguru McNeal, Jeffery
2011Ross, Christopher Jon Pittel, Boris
2011Samara, Marko March, Peter
2011Valle, Raciel Leary, Ian
2011Whitaker, Erica  Cogdell, James
2011Xie, Zhizhang Moscovici, Henri
2011Zhang, Huaijian Baker, Gregory
2011Zhang, Lizhi Costin, Ovidiu
2010Ahn, Sungwoo Terman, David
2010File, Daniel Whitman Cogdell, James
2010Huang, Min Costin, Ovidiu
2010Im, Jeong Sook Baker, Gregory
2010Joshi, Janhavi McNeal, Jeffery
2010Kadyrov, Shirali Shah, Nimish
2010Khare, Niraj Seress, Akos
2010Kilanowski, Phillip  March, Peter
2010Kim, Kyung-Mi Cogdell, James
2010Lee, Gangyong Rizvi, S. Tariq
2010Lim, Changhoon Guan, Bo
2010Liu, Yu-Han Clemens, Herbert
2010Mance, William Bergelson, Vitaly
2010Peng, Na Edgar, Gerald A.
2010Su, Shu Kao, Chiu-Yen
2010Wang, Jie Clemens, Herbert
2010Wang, Ying Kao, Chiu-Yen
2010Werner, Nicholas  Loper, Alan
2010Ye, Ji Tanveer, Saleh
2010Zeki, Mustafa Terman, David
2010Zeytuncu, Yunus Ergin McNeal, Jeffery
2010Zhang, Yanyan Golubitsky, Martin
2009Altomare, Christian  Robertson, G. Neil
2009Bezugly, Andriy Lou, Yuan
2009Griesmer, John  Bergelson, Vitaly
2009Joshi, Badal  Best, Janet
2009Kurt, Oguz Robertson, G. Neil
2009Kwa, Kiam Heong Gerlach, Ulrich
2009Li, Lingfei Zheng, Fangyang
2009Liu, Sheng-Chi Luo, Wenzhi
2009Luo, Guo Baker, Gregory
2009McSweeney, John  Pittel, Boris
2009Mehta, Nishali Seress, Akos
2009Pikula, Rafal Bergelson, Vitaly
2009Puliyambalath, Naushad  Seress, Akos
2009Qiu, Zhi Costin, Ovidiu
2009Shi, Ronggang Einsiedler, Manfred
2009Swartz, Eric  Seress, Akos
2009Wang, Xueying Terman, David
2009Xie, Chao Baker, Gregory
2009Young, Justin Rallis, Stephen
2009Yu, Yang Baker, Gregory
2009Zhao, Peng Luo, Wenzhi
2008Arms, Scott Sinnott, Warren
2008Ault, Shaun  Fiedorowicz, Z.
2008Balachandran, Niranjan Robertson, G. Neil
2008Hur, Suhkjin Glover, Henry H.
2008Kar, Aditi Chatterji, Indira
2008Khoury, Michael Cogdell, James
2008McClain, Christopher Robertson, G. Neil
2008Nikolov, Martin  Flicker, Yuval
2008Niu, Liang Seress, Akos
2008Schnell, Christian Clemens, Herbert
2008Xiong, Wei March, Peter
2008Xu, Songyun Clemens, Herbert
2008Yang, Keyan Seress, Akos
2008Yeum, Ji-A Pittel, Boris
2007Hambrock, Richard Lou, Yuan
2007Hammett, Adam  Pittel, Boris
2007Lennon, Craig  Pittel, Boris G.
2007Park, Choongseok Terman, David
2007Pavlov, Ronald  Bergelson, Vitaly
2007Pu, Ming March, Peter
2007Qi, Dongwen Davis, Michael
2007Schoenecker, Kevin  Wyman, Bostwick F.
2007Stey, George  McNeal, Jeffery D.
2006Dimitrov, Youri Edgar, Gerald A.
2006Fu, Yun Baker, Gregory
2006Guler, Dincer Zheng, Fangyang
2006Lee, Seung Youn March, Peter
2006McKinley, Scott  March, Peter
2006Oman, Gregory  Loper, Alan
2006Pitale, Ameya Rallis, Steven
2006Tsoi, Man Sandstede, B. & Lou, Y
2006Wang, Hongyuan Zheng, Fangyang
2006Xia, Honggang Luo, Wenzhi
2005Chan, Ping-Shun Flicker, Yuval
2005Ghazaryan, Anna  Sandstede, Bjorn
2005Guloglu, Ahmet  Luo, Wenzhi
2005Kane, Abdoul Terman, David
2005Kaygun, Atabey Moscovici, Henri
2005Kennel, Lauren McNeal, Jeffery
2005Manukian, Vahagn Sandstede, Bjorn
2005Micu, Eliade Mihai Robertson, G. Neil
2005Salminen, Adam  Linckelman, Markus
2005Wolfe, Adam  Seress, Akos
2004Antal, Tamas Moscovici, Henri
2004Argesanu, George  Wyman, Bostwick F.
2004Herbig, Anne-Katrin McNeal, Jeffery D.
2004Iancu, Aniela Karina Wyman, Bostwick F.
2004Liu, Xing Tanveer, Saleh
2004Malyushitsky, Sergey  Harada, Koichiro
2004Otto, Michael Krötz, Bernhard & Stanton, Robert
2004Roman, Cosmin  Rizvi, Syed M.Tariq
2004Wang, Jin Baker, Gregory
2003Bell, Robert William, II Charney, Ruth M.
2003Boros, Dan Davis, Michael W.
2003Golubeva, Natalia  Baker, Gregory R.
2003Gorodnyk, Oleksandr Bergelson, Vitaly
2003Han, Zhongxian Wyman, Bostwick F.
2003Lladser, Manuel  Pemantle, Robin A.
2003Mendris, Robert Nemethi, Andras
2003Wang, Chian-Jen Rallis, Stephen
2003Yablonsky, Eugene Dynin, Alexander
2003Zhou, Xiangqian Robertson, G. Neil
2002Aydin, Nuh Ray-Chaudhuri, Dijen K.
2002Barbacioru, Catalin  Sinnott, Warren M.
2002Conrad, Eric  Milne, Stephen C.
2002Craciun, Gheorghe Feinberg, Martin Robert
2002Fiala, Nick  Seress, Akos
2002Hu, Xiaodong Moscovici, Henri
2002Jalics, Jozsi Zoltan Terman, David
2002McCoy, Ted Edgar, Gerald A.
2002Sachelarie, Vlad Wyman, Bostwick F.
2002Sherer, Scott  Scott, James
2002Slone, Rodney  Lee, R.
2001Beli, Constantin  Hsia, John S.
2001Breitenbucher, Jon  Milne, Stephen C.
2001Korchagina, Inna  Solomon, Ronald
2001Liu, Youjian Fitz, Michael
2001Marchenko, Vadim  Terman, David
2001Pham, Lan  Baker, Gregory R.
2001Pohlman, Matthew  Baker, Gregory R.
2001Stacklin, Thomas  Pittel, Boris G.
2001Xie, Xuming Tanveer, Saleh
2000Arenas-Carmona, Luis  Hsia, John S.
2000Barbu, Adrian  Ash, Avner D.
2000Butkevich, Sergey  Bergelson, Vitaly
2000Cashy, John  Glover, Henry H.
2000Gonciulea, Constantin Davis, Michael W.
2000Gries, Daniel  Mislin, Guido
2000Iskhakov, Igor  Davis, Michael W.
2000Nabavi, Ali Ray-Chaudhuri, Dijen K.
2000Pavlov, Savva  Rallis, Stephen
2000Qian, Jin Ray-Chaudhuri, Dijen K.
2000Ralfs, Arthur  Terman, David
2000Ramsey, John  Oz, Hayrani A.
2000Xie, Xuming Tanveer, Saleh
2000Zhang, Shaobo Myung, In Jae
1999Blackford, Jason  Ray-Chaudhuri, Dijen K.
1999Bucicovschi, Bogdan Burghelea, Dan
1999Genyuk, Julia Edgar, Gerald A.
1999Gorokhovsky, Alexander  Moscovici, Henri
1999Bondareva Humphreys, Natalia  Baishanski, Bogdan M.
1999Liu, Kan Ray-Chaudhuri, Dijen K. & Sehgal, Surinder K.
1999Mihalas, Stelian Robertson, G. Neil
1999Möller, Torsten Crawfis, Roger
1999Nigussie, Yared Robertson, G. Neil
1999Ragozzine, Charles  Ferrar, Joseph C.
1999Roberts, Joel Philip Silverberg, Alice
1999Siap, Irfan Ray-Chaudhuri, Dijen K.
1999Snyder, Brian  Goss, David
1999Szilárd, Ágnes Nemethi, Andras
1999Yang, Zifeng Goss, David
1999Yeh, Jieh-Shan  Ray Chaudhduri, Dijen
1999YU, HOSEOG Rubin, Karl C.
1999ZHANG, LINGHAI Terman, David
1998Golds, Jeffrey  Edgar, Gerald A.
1998Hartenstein, Margaret  Solomon, Ronald
1998HLAVACEK, JAN Baishanski, Bogdan M.
1998Jalics, Miklos  Baker, Gregory R.
1998LARICK, PAUL  Bergelson, Vitaly
1998Lu, Qin Mislin, Guido
1998MAKAROV, MIHAIL  Kappeler, Thomas
1998Mohácsy, Hedvig Ray-Chaudhuri, Dijen K.
1998Renedo, Marco  Bergelson, Vitaly
1998WAYAND, LEE  Davis, Michael W.
1998ZENKOV, DMITRY  BLOCH, ANTHONY M.
19971997BALTEANU, CORNEL FIEDOROWICZ, ZBIGNIEW
1997CHEN, YU QING Glover, Henry H.
1997GALUP, LUIS  Baishanski, Bogdan M.
1997HLAVACEK, AMY  HUNEKE, J. PHILIP
1997Hofmann, Glenn Nagaraja, H.N.
1997JOHNSON, BRYAN  Baishanski, Bogdan M.
1997Kim, Yongdai Baatoszynski, Robert
1997Koperski, Jeffrey  Batterman, Robert
1997LEININGER, VERNE  Milne, Stephen C.
1997MAJOR, EMERY  Burghelea, Dan
1997NANCE, ANTHONY  DOWLING, THOMAS A.
1997STADLER, JONATHAN Milne, Stephen C.
1997VOMPE, DMITRY  Baker, Gregory
1997WEISHAAR, ROBERT  Pittel, Boris
1997YAO, LIHUA March, Peter
1997Ye, Jian Kodama, Yuji
1997ZINOVIEV, DMITRII Flicker, Yuval
1996Altobelli, Joseph  Charney, R.
1996BABIKOV, MARK  Ferrar, Joseph C.
1996BAGDASAROV, SERGEY  Mityagin, Boris 
1996Carlson, Charles  Forest, M. Gregory
1996Chan, Shing-Wai Moscovici, Henri
1996Chan, Wai Kiu Hsia, John
1996CHERN, SHIKAI Moscovici, Henri
1996Das, Manabendra  Edgar, Gerald A.
1996Degenhardt, Sheldon Milne, Stephen C.
1996HUNT, DONALD  CARROLL, FRANCIS W.
1996JEON, INTAE March, Peter
1996Lam, Ching Hung Harada, Koichiro
1996Lame, John Sinnott, Warren
1996MAHARRY, JOHN  Robertson, G. Neil
1996Morje, Prabhav Harada, Koichiro
1996MULLINS, EDMOND N., JR Edgar, Gerald A.
1996POPESCU, CRISTIAN  Rubin, Karl C.
1996POUFINAS, THOMAS Mityagin, Boris 
1996ROSENBERG, STEVEN  Sinnott, Warren
1996SHALACK, JULIE  Ash, Avner D.
1996SZABO, TIBOR Seress, Akos
1996Tsolomitis, Antonis Davis, W. 
1996Tungol, Ronald  Pittel, Boris G. 
1996Wong, ChiKun  Charney, Ruth
1996XIAO, YIMIN TALAGRAND, MICHEL
1996ZHOU, JIANPING Wyman, Bostwick F.
1995Bhatnagar, Gaurav Milne, Stephen C.
1995Dixit-Radiya, Vibha Panda, Dhabaleswar K.
1995Gupta, Sandeep  Huang, C.H.
1995Joung, Haewon Nevai, Paul
1995Kessar, Radha Solomon, Ronald
1995Lee, Cary Dougherty, Randall
1995Lee, Yoonweon Burghelea, Dan
1995Marcsik, John Burghelea, Dan
1995McCutcheon, Randall Bergelson, Vitaly
1995Oh, Jangheon Sinnott, Warren
1995Peery, Thaddeus  Ozbay, Hitay 
1995Pinter, Ferenc  Nevai, Paul
1995Snell, Michael  Baishanski, B.
1995Varga, Jozsef Mityagin, Boris 
1995Wai, Hon-kit Burghelea, Dan
1995Weisz, Iván Seress, Akos 
1995Wu, Kuo-Chi Kodama, Y.
1995Xiang, Qing Ray-Chaudhuri, D.K.
1995Xu, Mingzhi Rubin, Karl
1995Zhang, Jianxiang Baishanski, Bogdan
1994Ashokkumar, C. R. Yedavalli, Rama K.
1994Dharmatilake, Jack  Robertson, G. Neil
1994Giust, Steven  Wyman, Bostwick F.
1994Gonzalez-Aviles, Cristian  Rubin, Karl C.
1994Jiang, Dihua Rallis, Stephen
1994Li, Kuo-tung Rosenblatt, Joseph
1994Manjrekar, Rajesh  Ash, Avner
1994McClure, Mark Edgar, Gerald A.
1994Schwartz, Peter  Rosenblatt, Joseph
1994Shao, You Yu Hsia, John
1994Wu, Xiaohong Dowling, Thomas
1994Zantout, Rached  Zheng, Yuan F.
1993Anderson, Michael  Ferrar, J.
1993Beleznay, F. Foreman, Matthew 
1993Belhadj, Mohamed Aldemir, Tunc 
1993Daquila, Richard Carroll, Francis W. 
1993Elder, Gove  Madan, Manohar L. 
1993Huang, Xiaoming Bojanic, Ranko 
1993Lee, Euiwoo Terman, David 
1993Leou, Ying-Tyug Friedman, Harvey 
1993Ouyang, Mingqing Neumann, Walter 
1993Sofer, Adriana S. Ash, Avner 
1993Szabó, Tamás  Divis, Zita
1993Tang, Shu-Leung Gold, Robert
1993Yang, Tzu-Yi Glover, Henry H.
1993Zha, Xiaoya Dowling, Thomas
1993Zhu, Tianbao Ray-Chaudhuri, D.K.
1992Ahmed, Shamim Klein, Charles A.
1992Bishop, Gregory  Carlson, Timothy
1992Cao, Jianzhong Forest, M.G.
1992Gethner, Ellen Parson, L. Alayne 
1992Haaland, Inger  Bergelson, Vitaly 
1992Icaza Perez, Maria  Hsia, John S.
1992Kane, Stephen Mityagin, Boris
1992Kim, Jeongjin Ray-Chaudhuri, D.K.
1992Krandick, Werner Collins, George E.
1992Lang, Cheng-Lien Terman, David
1992Leclerc, Anthony  Moore, Ramon E.
1992Ling, Tianwen Friedman, Harvey M.
1992Liu, Kecheng Foreman, Matthew
1992Narayani, Lakshmi  Ray-Chaudhuri, D.K.
1992Raqab, Mohammad  Nagaraja, Haikady N.
1992Reyes, Noli  Baishanski, B.
1992Shaw, Hong-Min Ray-Chaudhuri, D.K.
1992Sheu, Shin-pyng Forest, M. Gregory
1992Spieler, Barry  Charney, Ruth M.
1992Zhao, Yue Robertson, Neil
1991Banaszak, Grzegorz Sinnott, Warren
1991Blanchard, John  Mityagin, Boris 
1991Brozovic, Douglas  Solomon, Ronald
1991Craighead, Robert  Carroll, F. W.
1991Dale, Wilbur Nolan Smith, Malcolm C.
1991Donahue, Michael  Mityagin, Boris
1991Johnson, Jeremy  Collins, George E.
1991Manoharan, Palanivel Burghelea, Dan
1991Mariasoosai, William Baishanski, Bogdan M.
1991Némethi, András Moscovici, Henri
1991O'Ryan Lermanda, Manuel Shapiro, Daniel
1991Prabaharan, Kanagarajah Sucheston, L. 
1991Reinhold-Larsson, Karin B. Rosenblatt, Joseph 
1991Voon, Shu-Nan Glover, Henry
1991Wang, Qi Forest, M.G.
1991Xiong, Chuyu Overman, Edward
1991Yan, Zhongde Edgar, Gerald
1990Chen, Lin Yesha, Y.
1990Chilakamarri, Kiran Babu Robertson, Neil
1990Druschel, Kimberly  Davis, M.
1990Forrest, Alan Hunter Bergelson, Vitaly 
1990Gajda, Wojciech Davis, M.W.
1990Iwakata, Yasushi Dowling, Thomas
1990Lari-Lavassani, Ali Lu, Yung-Chen
1990Lee, Doobum Burghelea, Dan
1990Lovri, Miroslav Derdzinski, A.
1990Prieto-Cox, Juan  Hsia, John S.
1990Rodriguez Villegas, Fernando Sinnott, Warren
1990Song, Yongjin Fiedorowics, Zbigniew
1990Szabo, Laszlo Sucheston, Louis
1990Tam, Laying Baishanski, Louis
1990Xia, Yining Glover, Henry H.
1990Yu, Jenn-Hwa Edgar, Gerald
1989Andaloro, Paul  Ferrar, Joseph C.
1989Anghel, Nicolae Moscovici, Henri
1989Bajnok, Bela Bannai, Eiichi
1989Bunge, John  Nagaraja, H.N.
1989Chen, Hua Burghelea, Dan
1989Kwok, Wing Man Bannai, Eiichi
1989Munemasa, Akihiro Bannai, Eiichi
1989Oporowski, Bogdan  Robertson, Neil
1989Peric, Goran Moscovici, Henri
1989Schram, Erin  Ray-Chaudhuri, D.K.
1989Wierdl, Mate Bergelson, Vitaly
1989Wu, Fangbing Moscovici, Henri
1988Bannai, Etsuko Hsia, John Sollion
1988Fiedler, Joseph Huneke, John P.
1988Ghanaat, Patrick Ruh, Ernst Alfred
1988Jha, Shing-Whu Nevai, Paul
1988Kim, Jae Moon Gold, Robert
1988Moussong, Gabor Davis, Michael W.
1988Reeder, Mark Stephen Avner, Dolnick Ash
1988Rzedowski Calderón, Martha Madan, Manohar Lal
1988Villa-Salvador, Gabriel Madan, Manohar Lal
1988Wimelaratna, Ramasinghege Davis, William Jay
1987Ali, Sayel  Baishanski, B.M.
1987Han, Sang-Geun Sinnott, Warren
1987Lang, Mong-lung Bannai, Eiichi
1987Ray, Phillip  Ferrar, J.C.
1987Song, Sung Yell Bannai, Eiichi
1987Vitray, Richard  Robertson, Neil
1986Batra, Sharat Wigen, Philip E. 
1986Bezdek, Andras Glover, Henry
1986Char, Shobha  Burghelea, Dan
1986Lee, Jong-Eao  Carroll, Francis
1986Manickam, Nachimuthu Bannai, Eiichi
1986Miklós, Dezsö Ray-Chaudhuri, D.K.
1986Sali, Attila Bannai, Eiichi
1986Weaver, Robert  Robertson, Neil
1985Bauldry, William  Nevai, Paul
1985Brackebusch, Ruth  Edgar, Gerald A. 
1985Burdick, Bruce  Huneke, Philip
1985Butts, Eric  Davis, William
1985Childress, Nancy  Gold, Robert
1985Cho, Chong-Man Johnson, William B. 
1985Choi, Sul-Young Bannai, Eiichi
1985Guan, Puhua Ash, Avner
1985Huang, Tayuan Bannai, Eiichi
1985Kim, Myung-Hwan Hsia, John S.
1985Narang, Kamal Harada, K.
1985Seress, Akos Ray-Chaudhuri, D.K.
1985Singer, Phyllis  Allen, Harry P.
1985Song, Hi Ja Davis, William J.
1984Brink, James  Gold, Robert
1984Chidume, Charles  Davis, William J.
1984Frangos, Nicholas Sucheston, Louis
1984Grove, John W., (John Whitaker) Davis, William
1984Hemmeter, Joseph Bannai, Eiichi
1984Hong, Yiming Bannai, Eiichi
1984Ku, Jong-Min Ferrar, Joseph C.
1984Sheen, Rong-Chyu Nevai, Paul
1984Wajima, Masayuki Harada, Koichiro
1984Woldar, Andrew  Solomon, Ronald
1982Bonan, Stanford  Nevai, Paul
1982Carothers, Neal  Davis, William
1982Cheng, Fuhua Baishanski, Bogdan
1982Costello, Patrick  Hsia, John S.
1982D'Mello, Joseph  Cronheim, Arno
1982Gross, Francis   Ksienski, Aharon A.
1982Johnson, Sandra  Glover, Henry H.
1982Kirschenbaum, Marc Glover, Henry H.
1982Mahoney, Carolyn  Dowling, Thomas A.
1982Oprea, John  Burghelea, Dan
1982Shan, Chin-Chi Baishanski, Bogdan
1982Shih, Ching-Hsien Robertson, Neil
1981Benham, James  Hsia, John S.
1981Brickell, Ernest  Ray-Chaudhuri, D.K.
1981DeLaurentis, John  Edgar, Gerald
1981Flinn, Patrick  Davis, William
1981Moon, Aeryung Bannai, Eiichi
1981Young, Elmer  Glover, Henry
1979Gearhart, Thomas  Levine, Norman
1979Huffman, William  Drobot, S.
1979Kahn, Jeffry  Ray-Chaudhuri, D.K.
1979Lichtin, Benjamin  Lu, Yung-Chen
1979Roth, Robert  Ray-Chaudhuri, D.K.
1979Valentini, Robert  Cronheim, Arno
1979Woltermann, Michael  Sehgal, Surinder K.
1978Anacker, Steven  Dowling, T.A.
1978Brewster, Stephen  Homer, William
1978Decker, Richard  Glover, Henry
1978Ford, David  Zassenhaus, Hans
1978Ko, Hai-Ping Dowling, T.A.
1978Lichtin, Benjamin  Lu, Yung Chen
1978Lovett, Jane  Ferrar, Joseph
1978Wang, Shinmin  Dowling, T.A. 
1978Yang, Liow-Jing Woods, Alan C.
1977Barnes, Martha  Wilson, Richard
1977LeFever, John  Ray-Chaudhuri, D.K. 
1977Liu, Chung-Der Baishanski, Bogdan
1977Nemzer, Daniel  Ray-Chaudhuri, D.K.
1976Alspach, Dale  Johnson, William B.
1976Astbury, Kenneth  Sucheston, L. 
1976Catlin, Paul Allen  Dowling, T.A. 
1976Chakravati, Kamal  Dowling, T.A.
1976Chang, Kuang-I Dowling, T.A.
1976Ching, Wai-Sin  Bostwick, F. Wyman 
1976Denig, William  Dowling, T.A. 
1976Gbur, Mary Flahive Divis, Bohuslav
1976Markot, Robert  Bannai, Eiichi
1976Pal, Sat Leitzel, James R.C.
1976Yoder, Jeffery  Mickle, Earl J.
1976Baker, Ronald  Dowling, T.A. 
1975Burell, Benjamin  Zibler, Joseph A.
1975Chan, Agnes  Ray-Chaudhuri, D.K.
1975Dennis, John  Mickle, Earl J.
1975Donaldson, John  Zassenhaus, Hans
1975Dor, Leonard  Johnson, William B.
1975Earnest, Andrew  Hsia, John S.
1975Johnson, Robert  Drobot, Stefan
1975Madden, Daniel  Madan, Manohar L.
1975Ploeger, Bernard  Baishanski, Bogdan
1975Scrandis, Ann  Zassenhaus, Hans
1975Sze, Michael Ming Chih Sucheston, L.
1975Trushin, David  Allen, Harry P.
1975Wang, Chin San Robertson, Neil
1974Assa, Steven  Harada, Koichiro
1974Dunham, William  Huneke, Philip
1974Howell, Russell  Bojanic, Ranko
1974Kuntz, Amy  Sucheston, Louis
1974Mayer, David  Brown, Harold
1974O'Neill, Larkin  Huneke, Philip
1974Higgins, RadaOn The Asymptotic Behavior Of Certain SequencesBojanic, Ranko
1973Bieberich, Richard  Baishanski, Bogdan
1973Coon, Lawrence  Riner, John W.
1973Hansen, Henry  Woods, Alan
1973Klippert, John  Eustice, Dan
1973Mertens, Robert  Ross, Arnold E.
1973McLean, Jeffery  Yaqub, Jill C.D.S.
1973Peterson, Roger  Hsia, John S.
1973Sommers, Dean  Crosswhite, F. Joe
1973Sprague, Alan  Ray-Chaudhuri, D.K.
1973Ulrey, Michael  Ahlswede, Rudolf
1973Vijayan, Kulakkatt  Ray-Chaudhuri, D.K.
1973Wang, Paul Tiing Kerr, Douglas S.
1973Winkler, William Sucheston, L.
1973Wong, Kwok Chi Brown, Robert
1972Agashe, Pushpa  Levine, N.
1972Delany, Matthew  Zassenhaus, Hans
1972Ekong, Victor  Bojanic, R.
1972Hovis, Robert  Levine, N.
1972Jurick, Robert  Trimble, Harold C.
1972Lee, You-Hwa  Bojanic, R. 
1972Merklen, Héctor  Zassenhaus, Hans
1972Molnar, Edward  Mislin, Guido
1972Pomaredo, Rolando  Janko, Z.
1972Raber, Neal  Cronheim, A.
1972Smith, Fredrick  Janko, Z.
1972Zahroon, Fike  Trimble, Harold
1971Datta, Biswa  Ray-Chaudhuri, D.K.
1971Nelson Engle, Jessie  Mickle, E.
1971Falk, Daniel Zassenhaus, Hans
1971Heiberg, Charles  Baishanski, Bogdan
1971Milles, Stephen  Trimble, Harold
1971McClure, Clair  Trimble, Harold
1971Karamanoukian, Zaven  Woods, A.C.
1971Lundgren, J. Richard Janko, Z. 
1971Pujara, Lakhpat  Dean, David
1971Richard, Howard  Trimble, Harold
1971Rosenblum, Lawrence  Woods, Alan
1971Sehnert, James  Woods, Alan
1971St. Andre, Richard Levine, N.
1971Terrell, Thomas  Kregnel, U.
1971Veith, Wilbur  Carroll, F.W.
1970Anderson, Osiefield Trimble, Harold
1970Biddle, James  Cronheim, A. 
1970Dudgeon, Charles  Whitney, D. Random
1970Gemma, James  Ahlswede, Rudolf
1970Hanigan, Francis  Uotila, Urho A.
1970Hill, David  Dean, David
1970Hogan, Guy  Cronheim, A.
1970Johnson, Charles  Zassenhaus, Hans
1970Klein, Albert  Levine, N.
1970Logan, J. David Drobot, S.
1970McFarland, Robert  Zassenhaus, Hans
1970Sachdeva, Usha Sucheston, Louis
1970Stager, William  Levine, Norman
1970Sonn, Jack Zassenhaus, Hans
1970Yanosko, Kenneth  Janko, Z.
1969Fong, Humphrey Sek-Ching Sucheston, L.
1969Haines, David  Levine, Norman
1969Hale, Douglas  Davis, William
1969Hern, Thomas  Shapiro, J.M.
1969Heuvers, Konrad  Drobot, Stefan
1969Hull, David Shapiro, J.M.
1969Kimble, Kenneth  Drobpt, Stefan
1969Krier, Nicholas  Yaqub, J.
1969Kunes, Laurence  Shapiro, J.M.
1969Liang, Joseph Jen-Yin Zassenhaus, Hans
1969Mathis, Robert  Saltzer, Charles
1969Meeks, Joseph Mickle, E.
1969Scott, Frank  Levine, Norman
1969Wee, Leben Li Levine, Norman
1969Wilson, R. M. Ray-Chaudhuri, D.K.
1968Block, Henry  Shapiro, J.M.
1968Bonar, Daniel  Carroll, F.W.
1968Brown, John  Kapp, Wolfgang
1968Girard, Dennis  Baishanski, Bogdan
1968Keck, David  Reichelderfer, P.V.
1968Koehl, Frederick  Carroll, F.W.
1968Konvisser, Marc  Kappe, Wolfgang
1968Lu, Yu-Mei Yu Trimble, Harold C.
1968Nachman, Louis  Levine, N. 
1968Parker, Donald  Kappe, Wolfgang
1968Phillips, Paul  Trimble, Harold C.
1968Plybon, Benjamin  Drobot, Stefan
1968Pu, Huay-min Huoh Helsel, R.G.
1968Queen, Clifford  Zassenhaus, Hans
1968Riggle, Timothy  Trimble, Harold C.
1968Shook, Thurston  Levine, N.
1968Sternbach, Leonard Dean, David W.
1968Whitford, Leslie  Baishanski, Bogdan
1967Caufield, Patrick  Levine, N.
1967DeVore, Ronald Bojanic, R.
1967Kimbleton, Stephen  Shapiro, J.M.
1967Klimko, Eugene  Sucheston, Louis
1967Klimko, Lawrence  Sucheston, Louis
1967Olson, John  Zassenhaus, Hans
1966Brown, Harold  Zassenhaus, Hans
1966Caid, Larry  Davis, William
1966Deever, David  Albian, Alexander
1966Holden, Lyman  Trimble, Harold C.
1966Hopkins, Mark  Mickle, E.
1966Nikolai, Paul  Saltzer, Charles
1966Steinlage, Ralph  Mickle, E.
1965Aggarwal, Satish  Bambah, R.P.
1965Breiter, Thomas  Levine, N.
1965Boonyasombut, Virool Shapiro, J.M.
1965Dumir, V. C. Bambah, R.P
1965Frazier, Thyrsa  Reichelderfer, Paul V.
1965Hans-Gill, R. J. Bamba, R.P.
1965Nelson, Larry  Rado, T.
1965Randels, James  Rado, T. 
1964Anderson, Charles  Kleinfeld, E.
1964Brabenec, Robert  Helse, R.G.
1964Brooks, James  Reichelderfer, Paul V.
1964Chaney, Robin  Reichelderfer, Paul V.
1964Houghton, Charles  Mickle, E. 
1964Maxwell, John  Reichelderfer, Paul V. 
1964Pu, Hwang Wen Helsel, R.G.
1963Coppage, William  Whitney, D.R.
1963Laffer, Walter  Mann, Henry B.
1963Lin, Shen Rado, T. 
1963McWorter, William  Abian, Alexander
1963Norris, Donald  Reichelderfer, Paul V.
1963Outcalt, David  Whitney, D.R.
1963Schaefer, Donald  Mickle, E.
1963Staley, David  Levine, N.
1962Dixon, Robert  Tull, J.P.
1962Duemmel, James  Reichelderfer, Paul V.
1962Hardy, F. Lane Kleinfeld, E.
1962Robison, Donald  Whitney, R.
1962Ryeburn, David Reichelderfer, Paul V.
1961Johnsen, Eugene  Ryser, H.J.
1961Leetch, James  Helsel, R.G.
1961Martino, Joseph  Whitney, D.R.
1960Craft, George  Reichelderfer, Paul V.
1960Weiler, Fred  Reichelderfer, Paul V.
1960Willke, Thomas  Whitney, D.R.
1959Maneri, Carl Kleinfeld, Erwin
1959McCulloh, Leon Mann, Henry B.
1959Menon, Manavazhi Mann, Henry B.
1959Nemitz, William Mickle, Earl
1958Haber, Robert  Ryser, H.J.
1958Silverman, Robert Whitney, D.R.
1958Thompson, Robert  Reichelderfer, Paul V.
1958Tinsley, Marion  Ryser, H.J.
1958Parker, Ernest  Hall, Marshall
1955Lin, Chio-Shih Mann, Henry B.
1955Parrish, Herbert Helsel, R.G.
1954Crowley, Thomas  Helsel, R.G.
1954Fadell, Albert  Rado, T. 
1954Neugebauer, Christoph  Mickle, E.
1954Zemlin, Richard  Hall, Marshall
1953Butts, Hubert  Mann, Henry B.
1953Dean, Richard  Hall, Marshall
1953Edwards, Miles  Mickle, E.
1953Evans, John  Lazar, Nathan
1953Hoy, Walter  Mann, Henry
1953Moranda Paul  Mann, Henry 
1953Sterbenz, Pat  Reichelderfer, Paul V.
1952Fadell, Edward  Reichelderfer, Paul V.
1952Levine, NormanAbsolutely Continuous Product Transformations Of The PlaneHelsel, R.G
1952Martin, E. Wainwright  Hall, Marshall
1952Mendenhall, Robert  Helsel, R.G.
1952Myers, William  Reichelderfer, Paul. V
1952Tinnappel, Harold  Mickle, E.
1951Marsaglia, GeorgeStochastic Processes And Classes Of Random VariablesMann, Henry
1950Hoyoke, ThomasAn Embedding Problem For Transitive Permutation GroupsHall, Marshall
1949Whitney, Donald RansomA Comparison of the Power of Nonparametric Tests and Tests Based on the Normal Distribution Under Nonnormal AlternativesMann, Henry
1949Colquitt, Landon A.On Paths Of Minimum Flight TimeBamforth, Frederic
1947Adney, Joseph Elliott Hall, Marshall
1947Miser, HughGeneralized Conformal Representations of Fréchet SurfacesRado, Tibor
1947William, ScottOn Essentially Absolutely Continuous TransformationsRado, Tibor
1946Woods, CecilA Restricted Class of Convex FunctionsRado, Tibor
1945Ayer, MiriamOn Convergence In LengthRado, Tibor
1943Huskey, HarryContributions to the Problem of GeoczeRado, Tibor
1942Goffman, CasperOn The Converses Of Certain Theorems On The Symmetric Structure Of Sets And FunctionsHenry Blumberg
1942Helsel, RobertA Geometrical Application of Intregal MeansRado, Tibor
1942Krabill, DavidSome Matrices Whose Elements Are Functions Of One VariableBamforth, Frederic
1942Westhafer, RobertSingular Solutions Of Ordinary Differential Equations Of The First OrderBamforth, Frederic
1941Mickle, EarlHamiltonian And Quasi-Hamiltonian Functions Associated With Double Integral Variation ProblemsLa Paz, Lincoln
1941Ringenberg, LawrenceOn Functions Of LawrenceRado, Tibor
1941Schart, WilliamConditions For Solutions Of Certain Differential Equations Which Have Specified PropertiesBamforth, Frederic
1941Young, PaulOn The Approximation of Functions By Integral MeansRado, Tibor
1940Cox, Jr., WilliamOn Cummutative Normal Matrices And Unitary Equivalence Of MatricesKuhn, Harry
1939Hammer, PrestonProjective Geometries Over A Pseudo-FieldRado, Tibor
1939Kato. ChosaburoConfiguration N Sub 3Rado, Tibor
1939Reichelderfer, PaulSome Properties Of Continuous Transformations In The PlaneRado, Tibor
1939Snyder, WalterOn Functions Of SquaresRado, Tibor
1939Tepletsky, BenjaminStability And Periodicity Of Solutions Of Mathieu's EquationBamforth, Frederic
1938Rodabaugh, LouisThe Solution of a Certain Linear Partial Differential Equation of the First OrderBamforth, Frederic
1937Kohlmetz, DorothyCertain Problems Of A Special Character In Convex FunctionsRado, Tibor
1936Bailey, AlsonAn Approach To The Study Of Conic Sections, Based On A Group Of Projective TransformationsRado, Tibor
1936Gleyzal, AndreOn Transfinite Real Numbers, General Orders, Riemannian And Finsler SpacesBlumberg, Henry
1936Hanson, EugeneA Theorem Of Denjoy, Young And Saks. Ii. The Tau LimitBlumberg, Henry
1936Hummel, PaulContinued Fractions And MatricesMacDuffee, Cyrus
1936Southard, ThomasOn Certain Projective Geometries and Their Relation to AlgebraRado, Tibor
1935Jenkins, EmersonThe Composition Of Quadratic FormsMacDuffee, Cyrus
1935Lewis, FredSome Properties Of An Infinite Class Of Collineation GroupsKuhn, Harry
1935Rinehart, RobertSome Properties Of The Discriminant Matrices Of A Linear Associative AlgebraMacDuffee, Cyrus

Stack Exchange Network

Stack Exchange network consists of 183 Q&A communities including Stack Overflow , the largest, most trusted online community for developers to learn, share their knowledge, and build their careers.

Q&A for work

Connect and share knowledge within a single location that is structured and easy to search.

What is expected in a masters thesis of a mathematics student?

What is the level of work expected in the masters thesis of a student of maths?

I know some people's works are worthy of publications while some involve only studying some topic in detail from a book and submitting a summary (since this is akin to a couple of courses, a year worth of work, in that topic in terms of content covered, I would consider this,which I believe is called a literature review thesis , as an extreme opposite of independent research work thesis).

But what is the "average" level of a MS thesis of a mathematics student? Is it usually closer to a literature review thesis or a research work thesis?

In particular, I would also like to know:

How much is it valued (if at all) when one applies for PhD? I have heard that its value is more in Europe than America, which if I were to guess I would say, may be due to absence of GRE like criterion there. Is this true?

Edit : After the wonderful existing answer explaining the case in Germany, I would really like to know the situation in US too. I expect a drastic difference due to the presence of GRE system but would like to know how much importance the thesis has, there.

PS: Please excuse me if one can find answers to some of these questions in already existing questions. I have searched, but couldn't find them. Please provide the links in those cases.

Also, anecdotal details will also be greatly appreciated. Thanks!

  • graduate-admissions
  • mathematics

Community's user avatar

  • @user54981 Of course that would most certainly be there. –  Neeraj Kumar Commented Jun 6, 2016 at 6:01
  • 2 The difference between US/Canada and Europe is mainly due to the fact that in most European institutions, applicants to a PhD program are expected to held a Masters or equivalent degree, and the PhD program is often research only with close to no coursework. Compare to North American institutions where applicants to a PhD program are not expected to hold a Masters, and generally only hold a Bachelors degree, and where the program tends to be longer with a more significant coursework portion. –  Willie Wong Commented Jun 8, 2016 at 13:40
  • Note also that the European application process for entering a PhD program is often quite different from the typical North American one. (Search on this site if you want to know more; I'm sure it has been asked before.) –  Willie Wong Commented Jun 8, 2016 at 13:41
  • @NeerajKumar Not necessarily equations. Almost certainly inequalities such as the element relation. –  Jacob Murray Wakem Commented Jun 8, 2016 at 15:35
  • @JacobWakem I don't understand what the term element relation is but isn't the presence of inequalities dependent on the topic? For example, a thesis in algebraic topology or geometry is most likely to not use any inequalities but one in functional analysis or number theory may have a lot of it.. –  Neeraj Kumar Commented Jun 9, 2016 at 4:03

3 Answers 3

I think this varies a lot. But for Germany your first question can be answers succinctly: In a Master's thesis you should show that you have potential for research .

On the other hand, expectations vary a lot between advisors. But certainly you do not have to prove a new theorem or develop a new theory.

How much is it valued (if at all) when one applies for PhD?

I can only answer for the situation where you apply in Germany. The thesis can be a door opener if it is topic closely related to the field where you want to do a PhD. Also a good mark is important. But also in Germany hiring professors will often contact your advisors or request a reference letter and this is much more important.

I have heard that its value is more in Europe than America, which if I were to guess I would say, may be due to no GRE like criterion. Is this true?

Not sure on this point since I can't provide a comparison with the US and also I am not sure if the situation is uniform with the EU.

Dirk's user avatar

  • Thanks for the answer. As I mentioned, there are the two extremes in the kinds of thesis. In the first I can imagine the potential for research to be clearly visible( since they are doing actual research work) but how about the second case? If the work is only the study of a topic then? I doubt if it would reflect much on the potential to do research. Though one consideration that I can imagine is if the person spends his thesis studying on a certain topic then would it be of any advantage if person applies into that topic for PhD. Are such considerations taken into account? –  Neeraj Kumar Commented May 26, 2016 at 15:37
  • Sorry if this is not getting more concrete, but, e. g., a literature review thesis can or can not show research potential. If your question is: What do I have to do in a Master's thesis do get a PhD position, the answer is "Nobody can tell you in advance." Go ahead and choose a thesis topic you find thrilling and write a good thesis. –  Dirk Commented May 26, 2016 at 16:27

From my knowledge of the US system (I did my graduate work in the US, and am currently a professor in the US), the average level of a masters thesis is relatively low. (That said, it usually does involve at least some original research).

The reason for this is the structure of Ph.D. programs in the US. Usually students are admitted to Ph.D. programs directly as undergraduates, and the first two years of the Ph.D. are similar to an MS program in Europe. Students who complete a Ph.D. don't generally write a masters thesis along the way. Rather, masters theses are usually written by students who decide in their second year not to continue with our Ph.D. program, but would still like to earn some sort of degree for their efforts. These theses are often weak (but sometimes are quite good).

Some students do use an MS as a stepping stone to Ph.D. programs elsewhere; indeed, I personally know students who successfully transferred to much stronger programs. Their MS-level work was much better than average.

In short: The degree itself won't be highly valued in the US, but doing an MS can lead to strong letters from your professors and research advisors, and these will be highly valued.

Anonymous's user avatar

  • Another complication with master's theses, in the U.S., is a perception that the student "will do a PhD thesis anyway" if they go on to a PhD program, and so there is less need for the master's thesis to include challenging research. The motivation for writing a master's thesis becomes different from the motivation for writing a PhD thesis. –  Oswald Veblen Commented Jun 9, 2016 at 21:19

A great resource I have used to understand the quality of final thesis work for my primary focus is the Open Access Theses and Dissertations which has thousands of master's and Ph. D. final publications. Research this website using your topic and you will see what amount of research is involved, differences and similarities between schools, methodologies, etc.

In addition, a great site for further publications is http://Arxiv.org . Many thesis in the U.S. are 'sandwich' publications, involving an assortment of publications published while student is performing research.

J. Roibal - BlockchainEng's user avatar

You must log in to answer this question.

Not the answer you're looking for browse other questions tagged phd graduate-admissions thesis masters mathematics ..

  • Featured on Meta
  • Join Stack Overflow’s CEO and me for the first Stack IRL Community Event in...
  • Bringing clarity to status tag usage on meta sites

Hot Network Questions

  • What sci-fi show was Ernie watching in the show My Three Sons?
  • Please help me identify my Dad's bike collection (80's-2000's)
  • Is the white man at the other side of the Joliba river a historically identifiable person?
  • The quest for a Wiki-less Game
  • What do these expressions mean in NASA's Steve Stitch's brief Starliner undocking statement?
  • Is there mathematical significance to the LaGuardia floor tiles?
  • A probably Fantasy middle-length fiction about a probable vampire during the Blitz
  • Text processing: Filter & re-publish HTML table
  • Does Poincare recurrence show that Gibbs entropy is not strictly increasing?
  • Are there epistemic vices?
  • Is this map real?
  • Is the highlighted part false?
  • Using "provide" with value of a variable
  • DFT warn: LUMO = HOMO
  • Do images have propositional content?
  • The meaning of "sharp" in "sharp sweetness"
  • Subtle racism in the lab: how to respond
  • Why Pythagorean theorem is all about 2?
  • Can I land on the "EuroAirport Basel-Mulhouse-Freiburg" with a German National Visa as first destination (NON-EU Citizen)?
  • 4/4 time change to 6/8 time
  • Why would the GPL be viral, while EUPL isn't, according to the EUPL authors?
  • Expansion in Latex3 when transforming an input and forwarding it to another function
  • sp_blitzlock returns blank data in SQL Managed instance
  • How long should a wooden construct burn (and continue to take damage) until it burns out (and stops doing damage)

shortest math dissertation

shortest math dissertation

How To Write A Dissertation Or Thesis

8 straightforward steps to craft an a-grade dissertation.

By: Derek Jansen (MBA) Expert Reviewed By: Dr Eunice Rautenbach | June 2020

Writing a dissertation or thesis is not a simple task. It takes time, energy and a lot of will power to get you across the finish line. It’s not easy – but it doesn’t necessarily need to be a painful process. If you understand the big-picture process of how to write a dissertation or thesis, your research journey will be a lot smoother.  

In this post, I’m going to outline the big-picture process of how to write a high-quality dissertation or thesis, without losing your mind along the way. If you’re just starting your research, this post is perfect for you. Alternatively, if you’ve already submitted your proposal, this article which covers how to structure a dissertation might be more helpful.

How To Write A Dissertation: 8 Steps

  • Clearly understand what a dissertation (or thesis) is
  • Find a unique and valuable research topic
  • Craft a convincing research proposal
  • Write up a strong introduction chapter
  • Review the existing literature and compile a literature review
  • Design a rigorous research strategy and undertake your own research
  • Present the findings of your research
  • Draw a conclusion and discuss the implications

Start writing your dissertation

Step 1: Understand exactly what a dissertation is

This probably sounds like a no-brainer, but all too often, students come to us for help with their research and the underlying issue is that they don’t fully understand what a dissertation (or thesis) actually is.

So, what is a dissertation?

At its simplest, a dissertation or thesis is a formal piece of research , reflecting the standard research process . But what is the standard research process, you ask? The research process involves 4 key steps:

  • Ask a very specific, well-articulated question (s) (your research topic)
  • See what other researchers have said about it (if they’ve already answered it)
  • If they haven’t answered it adequately, undertake your own data collection and analysis in a scientifically rigorous fashion
  • Answer your original question(s), based on your analysis findings

 A dissertation or thesis is a formal piece of research, reflecting the standard four step academic research process.

In short, the research process is simply about asking and answering questions in a systematic fashion . This probably sounds pretty obvious, but people often think they’ve done “research”, when in fact what they have done is:

  • Started with a vague, poorly articulated question
  • Not taken the time to see what research has already been done regarding the question
  • Collected data and opinions that support their gut and undertaken a flimsy analysis
  • Drawn a shaky conclusion, based on that analysis

If you want to see the perfect example of this in action, look out for the next Facebook post where someone claims they’ve done “research”… All too often, people consider reading a few blog posts to constitute research. Its no surprise then that what they end up with is an opinion piece, not research. Okay, okay – I’ll climb off my soapbox now.

The key takeaway here is that a dissertation (or thesis) is a formal piece of research, reflecting the research process. It’s not an opinion piece , nor a place to push your agenda or try to convince someone of your position. Writing a good dissertation involves asking a question and taking a systematic, rigorous approach to answering it.

If you understand this and are comfortable leaving your opinions or preconceived ideas at the door, you’re already off to a good start!

Private Coaching

Step 2: Find a unique, valuable research topic

As we saw, the first step of the research process is to ask a specific, well-articulated question. In other words, you need to find a research topic that asks a specific question or set of questions (these are called research questions ). Sounds easy enough, right? All you’ve got to do is identify a question or two and you’ve got a winning research topic. Well, not quite…

A good dissertation or thesis topic has a few important attributes. Specifically, a solid research topic should be:

Let’s take a closer look at these:

Attribute #1: Clear

Your research topic needs to be crystal clear about what you’re planning to research, what you want to know, and within what context. There shouldn’t be any ambiguity or vagueness about what you’ll research.

Here’s an example of a clearly articulated research topic:

An analysis of consumer-based factors influencing organisational trust in British low-cost online equity brokerage firms.

As you can see in the example, its crystal clear what will be analysed (factors impacting organisational trust), amongst who (consumers) and in what context (British low-cost equity brokerage firms, based online).

Need a helping hand?

shortest math dissertation

Attribute #2:   Unique

Your research should be asking a question(s) that hasn’t been asked before, or that hasn’t been asked in a specific context (for example, in a specific country or industry).

For example, sticking organisational trust topic above, it’s quite likely that organisational trust factors in the UK have been investigated before, but the context (online low-cost equity brokerages) could make this research unique. Therefore, the context makes this research original.

One caveat when using context as the basis for originality – you need to have a good reason to suspect that your findings in this context might be different from the existing research – otherwise, there’s no reason to warrant researching it.

Attribute #3: Important

Simply asking a unique or original question is not enough – the question needs to create value. In other words, successfully answering your research questions should provide some value to the field of research or the industry. You can’t research something just to satisfy your curiosity. It needs to make some form of contribution either to research or industry.

For example, researching the factors influencing consumer trust would create value by enabling businesses to tailor their operations and marketing to leverage factors that promote trust. In other words, it would have a clear benefit to industry.

So, how do you go about finding a unique and valuable research topic? We explain that in detail in this video post – How To Find A Research Topic . Yeah, we’ve got you covered 😊

Step 3: Write a convincing research proposal

Once you’ve pinned down a high-quality research topic, the next step is to convince your university to let you research it. No matter how awesome you think your topic is, it still needs to get the rubber stamp before you can move forward with your research. The research proposal is the tool you’ll use for this job.

So, what’s in a research proposal?

The main “job” of a research proposal is to convince your university, advisor or committee that your research topic is worthy of approval. But convince them of what? Well, this varies from university to university, but generally, they want to see that:

  • You have a clearly articulated, unique and important topic (this might sound familiar…)
  • You’ve done some initial reading of the existing literature relevant to your topic (i.e. a literature review)
  • You have a provisional plan in terms of how you will collect data and analyse it (i.e. a methodology)

At the proposal stage, it’s (generally) not expected that you’ve extensively reviewed the existing literature , but you will need to show that you’ve done enough reading to identify a clear gap for original (unique) research. Similarly, they generally don’t expect that you have a rock-solid research methodology mapped out, but you should have an idea of whether you’ll be undertaking qualitative or quantitative analysis , and how you’ll collect your data (we’ll discuss this in more detail later).

Long story short – don’t stress about having every detail of your research meticulously thought out at the proposal stage – this will develop as you progress through your research. However, you do need to show that you’ve “done your homework” and that your research is worthy of approval .

So, how do you go about crafting a high-quality, convincing proposal? We cover that in detail in this video post – How To Write A Top-Class Research Proposal . We’ve also got a video walkthrough of two proposal examples here .

Step 4: Craft a strong introduction chapter

Once your proposal’s been approved, its time to get writing your actual dissertation or thesis! The good news is that if you put the time into crafting a high-quality proposal, you’ve already got a head start on your first three chapters – introduction, literature review and methodology – as you can use your proposal as the basis for these.

Handy sidenote – our free dissertation & thesis template is a great way to speed up your dissertation writing journey.

What’s the introduction chapter all about?

The purpose of the introduction chapter is to set the scene for your research (dare I say, to introduce it…) so that the reader understands what you’ll be researching and why it’s important. In other words, it covers the same ground as the research proposal in that it justifies your research topic.

What goes into the introduction chapter?

This can vary slightly between universities and degrees, but generally, the introduction chapter will include the following:

  • A brief background to the study, explaining the overall area of research
  • A problem statement , explaining what the problem is with the current state of research (in other words, where the knowledge gap exists)
  • Your research questions – in other words, the specific questions your study will seek to answer (based on the knowledge gap)
  • The significance of your study – in other words, why it’s important and how its findings will be useful in the world

As you can see, this all about explaining the “what” and the “why” of your research (as opposed to the “how”). So, your introduction chapter is basically the salesman of your study, “selling” your research to the first-time reader and (hopefully) getting them interested to read more.

How do I write the introduction chapter, you ask? We cover that in detail in this post .

The introduction chapter is where you set the scene for your research, detailing exactly what you’ll be researching and why it’s important.

Step 5: Undertake an in-depth literature review

As I mentioned earlier, you’ll need to do some initial review of the literature in Steps 2 and 3 to find your research gap and craft a convincing research proposal – but that’s just scratching the surface. Once you reach the literature review stage of your dissertation or thesis, you need to dig a lot deeper into the existing research and write up a comprehensive literature review chapter.

What’s the literature review all about?

There are two main stages in the literature review process:

Literature Review Step 1: Reading up

The first stage is for you to deep dive into the existing literature (journal articles, textbook chapters, industry reports, etc) to gain an in-depth understanding of the current state of research regarding your topic. While you don’t need to read every single article, you do need to ensure that you cover all literature that is related to your core research questions, and create a comprehensive catalogue of that literature , which you’ll use in the next step.

Reading and digesting all the relevant literature is a time consuming and intellectually demanding process. Many students underestimate just how much work goes into this step, so make sure that you allocate a good amount of time for this when planning out your research. Thankfully, there are ways to fast track the process – be sure to check out this article covering how to read journal articles quickly .

Literature Review Step 2: Writing up

Once you’ve worked through the literature and digested it all, you’ll need to write up your literature review chapter. Many students make the mistake of thinking that the literature review chapter is simply a summary of what other researchers have said. While this is partly true, a literature review is much more than just a summary. To pull off a good literature review chapter, you’ll need to achieve at least 3 things:

  • You need to synthesise the existing research , not just summarise it. In other words, you need to show how different pieces of theory fit together, what’s agreed on by researchers, what’s not.
  • You need to highlight a research gap that your research is going to fill. In other words, you’ve got to outline the problem so that your research topic can provide a solution.
  • You need to use the existing research to inform your methodology and approach to your own research design. For example, you might use questions or Likert scales from previous studies in your your own survey design .

As you can see, a good literature review is more than just a summary of the published research. It’s the foundation on which your own research is built, so it deserves a lot of love and attention. Take the time to craft a comprehensive literature review with a suitable structure .

But, how do I actually write the literature review chapter, you ask? We cover that in detail in this video post .

Step 6: Carry out your own research

Once you’ve completed your literature review and have a sound understanding of the existing research, its time to develop your own research (finally!). You’ll design this research specifically so that you can find the answers to your unique research question.

There are two steps here – designing your research strategy and executing on it:

1 – Design your research strategy

The first step is to design your research strategy and craft a methodology chapter . I won’t get into the technicalities of the methodology chapter here, but in simple terms, this chapter is about explaining the “how” of your research. If you recall, the introduction and literature review chapters discussed the “what” and the “why”, so it makes sense that the next point to cover is the “how” –that’s what the methodology chapter is all about.

In this section, you’ll need to make firm decisions about your research design. This includes things like:

  • Your research philosophy (e.g. positivism or interpretivism )
  • Your overall methodology (e.g. qualitative , quantitative or mixed methods)
  • Your data collection strategy (e.g. interviews , focus groups, surveys)
  • Your data analysis strategy (e.g. content analysis , correlation analysis, regression)

If these words have got your head spinning, don’t worry! We’ll explain these in plain language in other posts. It’s not essential that you understand the intricacies of research design (yet!). The key takeaway here is that you’ll need to make decisions about how you’ll design your own research, and you’ll need to describe (and justify) your decisions in your methodology chapter.

2 – Execute: Collect and analyse your data

Once you’ve worked out your research design, you’ll put it into action and start collecting your data. This might mean undertaking interviews, hosting an online survey or any other data collection method. Data collection can take quite a bit of time (especially if you host in-person interviews), so be sure to factor sufficient time into your project plan for this. Oftentimes, things don’t go 100% to plan (for example, you don’t get as many survey responses as you hoped for), so bake a little extra time into your budget here.

Once you’ve collected your data, you’ll need to do some data preparation before you can sink your teeth into the analysis. For example:

  • If you carry out interviews or focus groups, you’ll need to transcribe your audio data to text (i.e. a Word document).
  • If you collect quantitative survey data, you’ll need to clean up your data and get it into the right format for whichever analysis software you use (for example, SPSS, R or STATA).

Once you’ve completed your data prep, you’ll undertake your analysis, using the techniques that you described in your methodology. Depending on what you find in your analysis, you might also do some additional forms of analysis that you hadn’t planned for. For example, you might see something in the data that raises new questions or that requires clarification with further analysis.

The type(s) of analysis that you’ll use depend entirely on the nature of your research and your research questions. For example:

  • If your research if exploratory in nature, you’ll often use qualitative analysis techniques .
  • If your research is confirmatory in nature, you’ll often use quantitative analysis techniques
  • If your research involves a mix of both, you might use a mixed methods approach

Again, if these words have got your head spinning, don’t worry! We’ll explain these concepts and techniques in other posts. The key takeaway is simply that there’s no “one size fits all” for research design and methodology – it all depends on your topic, your research questions and your data. So, don’t be surprised if your study colleagues take a completely different approach to yours.

The research philosophy is at the core of the methodology chapter

Step 7: Present your findings

Once you’ve completed your analysis, it’s time to present your findings (finally!). In a dissertation or thesis, you’ll typically present your findings in two chapters – the results chapter and the discussion chapter .

What’s the difference between the results chapter and the discussion chapter?

While these two chapters are similar, the results chapter generally just presents the processed data neatly and clearly without interpretation, while the discussion chapter explains the story the data are telling  – in other words, it provides your interpretation of the results.

For example, if you were researching the factors that influence consumer trust, you might have used a quantitative approach to identify the relationship between potential factors (e.g. perceived integrity and competence of the organisation) and consumer trust. In this case:

  • Your results chapter would just present the results of the statistical tests. For example, correlation results or differences between groups. In other words, the processed numbers.
  • Your discussion chapter would explain what the numbers mean in relation to your research question(s). For example, Factor 1 has a weak relationship with consumer trust, while Factor 2 has a strong relationship.

Depending on the university and degree, these two chapters (results and discussion) are sometimes merged into one , so be sure to check with your institution what their preference is. Regardless of the chapter structure, this section is about presenting the findings of your research in a clear, easy to understand fashion.

Importantly, your discussion here needs to link back to your research questions (which you outlined in the introduction or literature review chapter). In other words, it needs to answer the key questions you asked (or at least attempt to answer them).

For example, if we look at the sample research topic:

In this case, the discussion section would clearly outline which factors seem to have a noteworthy influence on organisational trust. By doing so, they are answering the overarching question and fulfilling the purpose of the research .

Your discussion here needs to link back to your research questions. It needs to answer the key questions you asked in your introduction.

For more information about the results chapter , check out this post for qualitative studies and this post for quantitative studies .

Step 8: The Final Step Draw a conclusion and discuss the implications

Last but not least, you’ll need to wrap up your research with the conclusion chapter . In this chapter, you’ll bring your research full circle by highlighting the key findings of your study and explaining what the implications of these findings are.

What exactly are key findings? The key findings are those findings which directly relate to your original research questions and overall research objectives (which you discussed in your introduction chapter). The implications, on the other hand, explain what your findings mean for industry, or for research in your area.

Sticking with the consumer trust topic example, the conclusion might look something like this:

Key findings

This study set out to identify which factors influence consumer-based trust in British low-cost online equity brokerage firms. The results suggest that the following factors have a large impact on consumer trust:

While the following factors have a very limited impact on consumer trust:

Notably, within the 25-30 age groups, Factors E had a noticeably larger impact, which may be explained by…

Implications

The findings having noteworthy implications for British low-cost online equity brokers. Specifically:

The large impact of Factors X and Y implies that brokers need to consider….

The limited impact of Factor E implies that brokers need to…

As you can see, the conclusion chapter is basically explaining the “what” (what your study found) and the “so what?” (what the findings mean for the industry or research). This brings the study full circle and closes off the document.

In the final chapter, you’ll bring your research full circle by highlighting the key findings of your study and the implications thereof.

Let’s recap – how to write a dissertation or thesis

You’re still with me? Impressive! I know that this post was a long one, but hopefully you’ve learnt a thing or two about how to write a dissertation or thesis, and are now better equipped to start your own research.

To recap, the 8 steps to writing a quality dissertation (or thesis) are as follows:

  • Understand what a dissertation (or thesis) is – a research project that follows the research process.
  • Find a unique (original) and important research topic
  • Craft a convincing dissertation or thesis research proposal
  • Write a clear, compelling introduction chapter
  • Undertake a thorough review of the existing research and write up a literature review
  • Undertake your own research
  • Present and interpret your findings

Once you’ve wrapped up the core chapters, all that’s typically left is the abstract , reference list and appendices. As always, be sure to check with your university if they have any additional requirements in terms of structure or content.  

shortest math dissertation

Psst... there’s more!

This post was based on one of our popular Research Bootcamps . If you're working on a research project, you'll definitely want to check this out ...

20 Comments

Romia

thankfull >>>this is very useful

Madhu

Thank you, it was really helpful

Elhadi Abdelrahim

unquestionably, this amazing simplified way of teaching. Really , I couldn’t find in the literature words that fully explicit my great thanks to you. However, I could only say thanks a-lot.

Derek Jansen

Great to hear that – thanks for the feedback. Good luck writing your dissertation/thesis.

Writer

This is the most comprehensive explanation of how to write a dissertation. Many thanks for sharing it free of charge.

Sam

Very rich presentation. Thank you

Hailu

Thanks Derek Jansen|GRADCOACH, I find it very useful guide to arrange my activities and proceed to research!

Nunurayi Tambala

Thank you so much for such a marvelous teaching .I am so convinced that am going to write a comprehensive and a distinct masters dissertation

Hussein Huwail

It is an amazing comprehensive explanation

Eva

This was straightforward. Thank you!

Ken

I can say that your explanations are simple and enlightening – understanding what you have done here is easy for me. Could you write more about the different types of research methods specific to the three methodologies: quan, qual and MM. I look forward to interacting with this website more in the future.

Thanks for the feedback and suggestions 🙂

Osasuyi Blessing

Hello, your write ups is quite educative. However, l have challenges in going about my research questions which is below; *Building the enablers of organisational growth through effective governance and purposeful leadership.*

Dung Doh

Very educating.

Ezra Daniel

Just listening to the name of the dissertation makes the student nervous. As writing a top-quality dissertation is a difficult task as it is a lengthy topic, requires a lot of research and understanding and is usually around 10,000 to 15000 words. Sometimes due to studies, unbalanced workload or lack of research and writing skill students look for dissertation submission from professional writers.

Nice Edinam Hoyah

Thank you 💕😊 very much. I was confused but your comprehensive explanation has cleared my doubts of ever presenting a good thesis. Thank you.

Sehauli

thank you so much, that was so useful

Daniel Madsen

Hi. Where is the excel spread sheet ark?

Emmanuel kKoko

could you please help me look at your thesis paper to enable me to do the portion that has to do with the specification

my topic is “the impact of domestic revenue mobilization.

Submit a Comment Cancel reply

Your email address will not be published. Required fields are marked *

Save my name, email, and website in this browser for the next time I comment.

shortest math dissertation

  • Print Friendly

Get the Reddit app

An online community for discussing issues related to academia, faculty life, research, and institutional structures. This is NOT the place to ask questions about your homework, your particular school or professors, or to get admission advice! Survey posts must be approved by mods in advance, must include contact/IRB info, and must be specific to academia.

Is it normal in your field for a PhD thesis to be less than 100 pages?

The length of a PhD thesis largely depends on the academic discipline and specific requirements of the institution. In the humanities or social sciences, theses often exceed 250 pages because of the extensive qualitative data and literature reviews. Is it normal in your field or subfield for a PhD thesis to be less than 100 pages?

By continuing, you agree to our User Agreement and acknowledge that you understand the Privacy Policy .

Enter the 6-digit code from your authenticator app

You’ve set up two-factor authentication for this account.

Enter a 6-digit backup code

Create your username and password.

Reddit is anonymous, so your username is what you’ll go by here. Choose wisely—because once you get a name, you can’t change it.

Reset your password

Enter your email address or username and we’ll send you a link to reset your password

Check your inbox

An email with a link to reset your password was sent to the email address associated with your account

Choose a Reddit account to continue

Home

Search form

  • Travel & Maps
  • Our Building
  • Supporting Mathematics
  • Art and Oxford Mathematics
  • Equality, Diversity & Inclusion
  • Undergraduate Study
  • Postgraduate Study
  • Current Students
  • Research Groups
  • Case Studies
  • Faculty Books
  • Oxford Mathematics Alphabet
  • Oxford Online Maths Club
  • Oxford Maths Festival
  • It All Adds Up
  • Problem Solving Matters
  • PROMYS Europe
  • Oxfordshire Maths Masterclasses
  • Outreach Information
  • Mailing List
  • Key Contacts
  • People List
  • A Global Department
  • Research Fellowship Programmes
  • Professional Services Teams
  • Conference Facilities
  • Public Lectures & Events
  • Departmental Seminars & Events
  • Special Lectures
  • Conferences
  • Summer Schools
  • Past Events
  • Alumni Newsletters
  • Info for Event Organisers & Attendees
  • Undergraduate Courses
  • Teaching and Learning
  • Part C Students

Dissertations

  • Dissertation Topics Titles 2020-21
  • Dissertation Topics Titles 2021-22
  • Dissertation Topics Titles 2022-23
  • Dissertation Topics Titles 2023-24

Introduction to Dissertation 

OMMS and Part C students are required to undertake a dissertation worth two units as part of their degree programme. This can be either a mathematics dissertation or a statistics dissertation. 

The dissertation will entail investigating a topic in an area of the Mathematical Sciences under the guidance of a dissertation supervisor. This will culminate in a written dissertation with a word limit of 7,500 words, which usually equates to 25-35 pages. It is expected that students embarking on a dissertation will be working on it over Christmas vacation, Hilary Term and Easter vacation for submission in early Trinity Term.

Students completing a dissertation may request a book for consultation if it is held only by the Whitehead Library (and not held in the RSL, their College library or as an e-book) by emailing the Librarian at @email .

The book will be sent to the RSL where it can be consulted for reference, not borrowing.   Please see further information  here . 

Timetable for Dissertations

 
Week 0, FridayDissertation Information Session
Week 0-1Dissertation abstracts published
Week 3, Friday 12:00Deadline for submitting dissertation choices
Week 5, FridayStudents notified of project allocation
Weeks 7 and 81-2 initial meetings with dissertation supervisor
 
Weeks 1-84 or 5 further supervision meetings
Weeks 7 and 8Oral presentations take place
 
Week 1, Monday 12:00Submission deadline

Choosing a topic

Following the Dissertation Information Session, a list of potential dissertation topics will be published below. Each topic will be accompanied by a short abstract outlining the project with details on necessary pre-requisite knowledge and the maximum number of students who will be able to take each topic. You will be asked to complete an online form, ranking 5 of the topics. Please note that Maths Part C students are only permitted to chose a maximum of three statistics topics. You will be notified of which project you have been allocated by the end of week 5.

Oral Presentation

Each student is required to give an oral presentation to their supervisors and at least one other person with some knowledge of the field of the dissertation. These will usually take place in the final two weeks of Hilary Term. The presentation does not count towards the final assessment of the project, however, it will give you an opportunity to practise your presentation skills which will prove useful in your later careers.

Useful Links and Sources of Information

  • Nov 2022 Dissertations Information Session
  • Dissertation_Guidenotes_2023-24.pdf
  • Past project archive
  • Mathematical Institute's LaTeX help
  • University guidance on research and library skills
  • University guidance on referencing
  • oral_presentation_guidelines_2023-24.pdf
  • Guidance for Supervisors 2023-24_1.pdf

The First Notices to Candidates (including information on dissertations) can be found here .

IMAGES

  1. (PDF) The Dramatic and Ultimate Shortening of a Doctoral Dissertation

    shortest math dissertation

  2. The Shortest Mathematics Paper #maths #mathematics #research

    shortest math dissertation

  3. Masters Thesis Ideas Math

    shortest math dissertation

  4. The Shortest Papers Ever Published

    shortest math dissertation

  5. The Shortest Papers Ever Published

    shortest math dissertation

  6. Math Dissertation Length Sample

    shortest math dissertation

VIDEO

  1. Dissertation Formatting and APA Style presented by Dr. Scott Zimmer

  2. Quantitative Dissertation Methodology Section: Video 08

  3. Quantitative Dissertation Methodology Section: Video 07

  4. Quantitative Dissertation Methodology Section: Video 06

  5. The shortest math

  6. Quantitative Dissertation Methodology Section: Video 5

COMMENTS

  1. What is the shortest Ph.D. thesis? [closed]

    What is the shortest Ph.D. thesis? [closed]

  2. John Nash's Super Short PhD Thesis: 26 Pages & 2 Citations

    When John Nash wrote "Non Coop­er­a­tive Games," his Ph.D. dis­ser­ta­tion at Prince­ton in 1950, the text of his the­sis (read it online) was brief. It ran only 26 pages. And more par­tic­u­lar­ly, it was light on cita­tions. Nash's diss cit­ed two texts: John von Neu­mann & Oskar Mor­gen­stern's The­o­ry of Games ...

  3. The World's Shortest PhD Dissertations

    16 pages - Edmund Landau: Neuer Beweis der Gleichung (1899) / New Proof of the Equation (2007) 13 pages - Burt Totaro: Milnor K-Theory is the Simplest Part of Algebraic K-Theory (1992) 9 pages - David Lee Rector: An Unstable Adams Spectral Sequence (1966) Please drop us a line if you know any shorter dissertations than the ones above!

  4. The Shortest Papers Ever Published

    Math can be hard and tedious resulting in very long papers. The 1995 proof of Fermat's last Theorem was 108 pages long. But math can also be short. Lander and Parkin's paper about a conjecture by Euler (related to Fermat's last Theorem), is probably the dream of everyone ever written a paper: It answers an interesting and important ...

  5. Short but Substantial Math Papers

    I am looking for short papers that made a significant impact on the mathematics community. I have already seen: interesting-but-short-math-papers and, What is the Shortest Ph.D. Thesis? on math overflow, but these weren't quite what I was looking for (although the intersection of the set of answers to this question with the set of answers to either of the above links is likely to be non-trivial)

  6. Mathematics PhD dissertations that opened a new field of research

    Nash earned a Ph.D. degree in 1950 with a 28-page dissertation on non-cooperative games. The thesis, written under the supervision of doctoral advisor Albert W. Tucker, contained the definition and properties of the Nash equilibrium, a crucial concept in non-cooperative games. It won Nash the Nobel Memorial Prize in Economic Sciences in 1994.

  7. The Dramatic and Ultimate Shortening of a Doctoral Dissertation in

    The shortest possible do ctoral thesis in mathematics has no sentenc es at all. As a way of a proof we giv e the celebrated example of Euler's discovery of the composite character of the number ...

  8. Length of the average dissertation

    Economics, mathematics, and biostatistics had the lowest median page lengths, whereas anthropology, history, and political science had the highest median page lengths. This distinction makes sense given the nature of the disciplines. I was on the long end of the statistics distribution, around 180 pages. Probably because I had a lot of pictures.

  9. PDF A Refinement of a Theorem of J. E. Littlewood

    Department of Mathematics, California State University-Los Angeles, 5151 State University Drive, Los Angeles, CA 90032 [email protected]. A Refinement of a Theorem of J. E. LittlewoodIn [1], J. E. Littlewood raises the question of how short a doctoral dissertation in mathematics could in principle be,1 and propose.

  10. Harvard Mathematics Department Harvard Department of Mathematics PhD

    Harvard Department of Mathematics PhD Dissertations ...

  11. Comprehensive Guide for Ph.D. thesis in Mathematics?

    It could also be helpful to check this and this short guides and the books by Steven G. Krantz, in particular. A Mathematician's Survival Guide: Graduate School and Early Career Development. A Primer of Mathematical Writing. The first one contains subsection 4.6 which deals specifically with writing a thesis, the second one is on mathematical ...

  12. Harvard Mathematics Department Harvard Department of Mathematics PhD

    Many older dissertations can be found on ProQuest Dissertation and Theses Search which many university libraries subscribe to. Harvard University. Department of Mathematics. Science Center Room 325. 1 Oxford Street. Cambridge, MA 02138 USA. Tel: (617) 495-2171 Fax: (617) 495-5132. Department Main Office Contact.

  13. Dissertations and Placements 2010-Present

    Dissertations and Placements 2010-Present - Cornell Math

  14. How long is a PhD dissertation? [Data by field]

    How long is a PhD dissertation? [Data by field]

  15. Average length of PhD dissertations by major : r/dataisbeautiful

    A math dissertation might as well be Martian with all the symbols. A lot of humanities dissertations might as well be Martian because of all the theory being used which necessitates an esoteric vocabulary. Read a page of Bertrand Russell's stuff on philosophy of language or Heidegger's stuff on phenomenology and, for a person outside the ...

  16. PhD Dissertations

    PhD Dissertations | Department of Mathematics - OSU Math

  17. Mathematics Theses, Projects, and Dissertations

    Mathematics Theses, Projects, and Dissertations

  18. What is expected in a masters thesis of a mathematics student?

    9. I think this varies a lot. But for Germany your first question can be answers succinctly: In a Master's thesis you should show that you have potential for research. On the other hand, expectations vary a lot between advisors. But certainly you do not have to prove a new theorem or develop a new theory.

  19. How To Write A Dissertation Or Thesis

    How To Write A Dissertation Or Thesis (+ Examples)

  20. Guidelines on Time for Review and Assessment of Dissertations

    DEPARTMENT OF MATHEMATICS. The graduate faculty in the Department of Mathematics agrees with the principles in the newly adopted Graduate School-New Brunswick's GUIDELINES ON TIME FOR REVIEW AND ASSESSMENT OF QUALIFYING PAPERS, THESES AND DISSERTATIONS": to maintain a culture of mutual respect between students and faculty members and that this include excellent communication among them.

  21. Is it normal in your field for a PhD thesis to be less than ...

    The shortest thesis I ever saw was like 75 pages, but that student had... issues. They were basically allowed to leave with a PhD, but didn't actually take up a career that needed it. It's much more usual for the thesis to be 200-300 pages. ... My dad's PhD thesis in mathematics was 20 pages, he became a full professor at 29. My master thesis ...

  22. Dissertations

    This can be either a mathematics dissertation or a statistics dissertation. ... Each topic will be accompanied by a short abstract outlining the project with details on necessary pre-requisite knowledge and the maximum number of students who will be able to take each topic. You will be asked to complete an online form, ranking 5 of the topics.

  23. Shortest Mathematics PHD Thesis

    Shortest Mathematics Phd Thesis - Free download as PDF File (.pdf), Text File (.txt) or read online for free. - Writing a mathematics PhD thesis can be an incredibly challenging and time-consuming task that requires precision and dedication, from conducting extensive research to analyzing data and formulating arguments. - For many students, the pressure of producing a high-quality thesis while ...